Você está na página 1de 191

1

Cinemtica (MRU)

Bienvenidos al mundo de la Fsica Pero, qu es la Fsica? La Fsica es una ciencia
bsica, es decir fundamental; es madre de muchas otras ciencias. As, utilizan ideas de la fsica
los qumicos cuando estudian la estructura de las molculas, los paleontlogos cuando
intentan reconstruir el modo de caminar de los dinosaurios, y los climatlogos cuando estudian
cmo las actividades humanas afectan la atmsfera y los ocanos.

La fsica es la base de toda la ingeniera y la tecnologa; ningn ingeniero podra
disear un televisor de pantalla plana, una nave espacial ni siquiera una trampa para ratones,
sin antes haber comprendido las leyes bsicas de la fsica.
El estudio de la fsica es tambin una aventura; ustedes la encontrarn desafiante, a
veces frustrante y en ocasiones dolorosa; sin embargo, con frecuencia les brindar abundantes
beneficios y satisfacciones. La fsica estimular en ustedes el sentido de lo bello, as como
vuestra inteligencia racional.

Si alguna vez se han preguntado por qu el cielo es azul, o cmo hacen las ondas de
radio para viajar por el espacio vaco, o cmo un satlite permanece en rbita, las respuestas la
encontrarn en la Fsica. Sobre todo, apreciarn a la Fsica como un logro sobresaliente de la
inteligencia del hombre en su afn por entenderle mundo en que vivimos y a la humanidad
misma.

Pero como ven, el campo de las cuestiones de las que se ocupa la fsica es muy vasto y
nosotros en este primer curso lo vamos a limitar a una parte de ella, conocida con el nombre de
mecnica.

La mecnica estudia las relaciones mutuas que vinculan a 3 agentes fundamentales de la
naturaleza: las fuerzas, la materia y el movimiento. Y precisamente nuestro estudio comenzar
por esto ltimo: el movimiento; as, en estas primeras 4 clases estudiaremos al movimiento, al
movimiento solo, aislado. Su vinculacin con las fuerzas y la materia se ir desarrollando
despus, a lo largo del curso.

Cinemtica.
Si deseamos un ttulo para los temas que trataremos en estas 4 primeras clases, l ser
cinemtica.

CINEMTICA, es la parte de la mecnica que describe los movimientos.

Etimolgicamente la palabra cinemtica significa estudio de los movimientos.
Cuando un cuerpo viaja, puede hacerlo en lnea recta o describiendo curvas de lo ms diversas.
Llamaremos trayectoria a esa lnea recta o curva constituida por los infinitos puntos del
espacio por donde ha pasado el cuerpo durante su marcha. No cabe duda que los movimientos
ms simples para estudiar son aqullos que tienen una trayectoria rectilnea, y sern los
primeros que vamos a estudiar. Como una recta tiene una sola dimensin, a estos movimientos
se los llama tambin movimientos unidimensionales.

Para que nuestro estudio sea lo ms simple posible, siempre sustituiremos el cuerpo que
se mueve por un punto que lo represente: si estudiamos el movimiento de un auto de carrera,
no estudiaremos los infinitos movimientos que realizan cada uno de los infinitos puntos de la
carrocera, sino que escogeremos un nico punto del auto y describiremos el movimiento de
ese nico punto. A ese nico punto al que quedar reducido un cuerpo a los fines de su estudio,
le daremos el nombre de partcula.

Resumiendo lo expresado hasta ac, en la clase de hoy estudiaremos la cinemtica de
una partcula que describe una trayectoria rectilnea.

En cinemtica tenemos dos variables bsicas que son la posicin (que representaremos
con x) y el tiempo (t), y el propsito final de nuestro estudio consistir en obtener la expresin
matemtica que vincule x con t.

Sistema de referencia, posicin y desplazamiento.
Para poder estudiar cualquier movimiento, es fundamental adoptar previamente un
sistema de referencia; l consiste en uno (o ms) ejes cartesianos, respecto del cual (o de los
cuales) puedan expresarse las coordenadas que ubiquen a nuestra partcula en un instante dado.
Elegir un sistema de referencia significa adoptar 2 cosas: un origen (dnde estar el cero) y un
sentido (cmo graduar el eje). Como estas son elecciones personales, es indispensable que al
comenzar a resolver cualquier problema se haga un esquema donde se muestre claramente el
sistema de referencia que se ha elegido.

Supongamos que hacemos rodar una bola por una superficie horizontal; la bola seguir
una trayectoria rectilnea; el sistema de referencia ser un eje nico x cuya direccin haremos
coincidir con la trayectoria; el esquema ser:





La posicin de la partcula en un instante de tiempo dado queda expresada por el valor
de la coordenada del eje de referencia sobre la cual se encuentra en ese instante.

Si la partcula est dotada de movimiento, la posicin va cambiando con el transcurrir
del tiempo.

Ahora vamos a suponer que leemos las posiciones de la partcula para dos momentos de
tiempo diferentes; por ejemplo t
1
= 2 s y t
2
= 3 s donde siempre t
2
es posterior a t
1


1 CASO: Para t
1
, corresponde un x
1
= 2 m
Para t
2
, corresponde un x
2
= 7 m
Es fcil ver que la partcula viaja hacia la derecha, en el mismo sentido que tiene la
flecha del eje de referencia.

A continuacin introduciremos un concepto muy importante: el desplazamiento (x)

DESPLAZAMIENTO es la diferencia que hay entre 2 posiciones.

Para que este concepto sea verdaderamente til, la diferencia indicada no puede hacerse
de cualquier manera sino que debe hacerse obedeciendo el siguiente orden:

x = x
final
x
inicial
= x
2
- x
1

En nuestro ejemplo, x = (7 2) m = + 5 m.
El desplazamiento ha resultado positivo y la partcula viaja hacia
la derecha.

x es un vector que va de 1 hacia 2 cuyo sentido coincide
con el del eje x.

2 CASO: Para t
1
, corresponde un x
1
= 8 m
Para t
2
, corresponde un x
2
= 5 m
Deducimos que ahora la partcula viaja hacia la izquierda con un sentido de movimiento
opuesto al del eje x.
x = x
2
x
1
= (5 8)m = - 3 m
El desplazamiento es negativo y como vector tiene
sentido opuesto al del eje x.

3 CASO: Para t
1
, corresponde un x
1
= 4 m
Para t
2
, corresponde un x
2
= 4 m
Aqu debemos ser cautelosos; no podemos asegurar que la partcula ha permanecido en
reposo. La coincidencia de las posiciones extremas no aseguran el reposo. Nada sabemos sobre
las posiciones intermedias entre t
1
y t
2
.
x = 0 (vector x nulo) no implica necesariamente reposo.

0 2 4 6 8
X (m)
x (m)
x
1
=2 x
2
=7
x
x (m)
x
2
=5m x
1
= 8m
x
Qu hemos aprendido a travs de estos ejemplos? Que el signo del desplazamiento nos
informa sobre el sentido del movimiento de la partcula por su relacin con el sentido del eje de
referencia.

Velocidad media. (v
med
).

VELOCIDAD MEDIA de una partcula es el cociente entre su desplazamiento y el intervalo
de tiempo en que se produce.


1 2
1 2
t t
x x
t
x
v
med

=
Como se trata de un cociente entre un vector y un escalar, la v
med
es una cantidad
vectorial. Los vectores v
med
y x tienen siempre la misma direccin y sentido.

La v
med
es una cantidad vectorial, y su sentido seala el sentido del movimiento de la
partcula.

La unidad de v
med
en el MKS es m/s.

Ejercicio 1.
Reducir unidades de velocidad.
a) Reducir 72 km/h a m/s [Resp. 20 m/s]
b) Reducir 24 m/s a km/h [Resp: 86,4 km/h]

Ejercicio 2.
Mediciones experimentales efectuadas con una partcula en movimiento rectilneo,
arrojaron los siguientes resultados:

PUNTO A B C D E F
POSICIN (m) 8 12 16 20 16 0
TIEMPO (t) 0 2 4 6 7 11

a) Trazar un diagrama x = f(t) para el movimiento.
b) Calcular las velocidades medias para los intervalos AC, CE y EF.
Solucin.

s
m
s
m
t t
x x
V
A C
A C
AC MED
2
) 0 4 (
) 8 16 (
+ =

=

s
m
s
m
t t
x x
v
C E
C E
medCE
0
) 4 7 (
) 16 16 (
=

=

s
m
s
m
t t
x x
v
E F
E F
EF med
4
) 7 11 (
) 16 0 (
=

=



x (m)
t (s)
0 2 4 6 7 8 10 11 12
20

16

12

8

4
A
B
C
D
E
F
IMPORTANTE: Un diagrama x = f(t) NO muestra la trayectoria de la partcula. La partcula
describe en este problema una trayectoria rectilnea. La trayectoria, (el camino) de la
partcula es el propio eje de referencia x.

Velocidad media y rapidez media.
Las palabras velocidad y rapidez se usan indistintamente en el lenguaje cotidiano; no
obstante en fsica tienen diferente significado. Se definen as:

t
x
tiempo
ento desplazami
velocidad

= = (1)
= =
tiempo
recorrida longitud
rapidez (2)
Mientras en la (1) x (y con l, tambin la velocidad), pueden tener diferentes signos
segn el sentido del movimiento, en la (2) la longitud recorrida (y con ella la rapidez tambin)
son cantidades siempre positivas; no nos informan sobre el sentido del movimiento; tan solo
nos permite saber cun tan rpido se mueve la partcula.

Por lo tanto, la velocidad es un vector y la rapidez es un escalar positivo. Pero
cuidado!: No es la rapidez igual al mdulo del vector velocidad. nicamente rapidez = v


cuando la partcula se ha movido todo el tiempo en el mismo sentido. Sino no.

Podemos comprobar lo anterior, retomando el ejercicio 2 y calculando la rapidez en el
intervalo CE. La longitud recorrida en ese intervalo es de 8 metros y demora 3 s en hacerlo, por
lo que:

s
m
s
m
rapidez
CE
7 , 2
3
8
= =
valor que difiere con v
med

CE
. En cambio, en los intervalos AC y EF donde la partcula se
mueve siempre con el mismo sentido, se comprueba que la rapidez es igual al mdulo del
vector velocidad.

Movimiento rectilneo uniforme (MRU).

MRU es el movimiento que realiza la partcula cuando recorre una trayectoria rectilnea con
velocidad constante.

Grficas posibles:





v (+)
v (-)
A B
x
t
x
0

agudo obtuso
v
t
A
B
Leyes del MRU.
1- La posicin es una funcin lineal del tiempo.
2- La velocidad es constante.
Ecuacin horaria del MRU.
x = x
0
+ vt
Destacar:
1- Por qu se llama ecuacin horaria?
2- En el diagrama x = f(t) la velocidad est representada por la pendiente de la recta, y
ella es (+) (-) segn que el ngulo sea agudo u obtuso.
3- En la ecuacin horaria, x
0
(posicin inicial) y v son constantes y x; t son las
variables.

Problemas de encuentro en MRU.
A- ENTRE MVILES QUE VIAJAN EN UN MISMO SENTIDO.
Un mvil pasa por A con v
1
= 2 m/s. Cinco segundos despus otro mvil pasa por A en
el mismo sentido con v
2
= 3 m/s. Si A es el origen del sistema de referencia, hallar a) la
posicin de encuentro; b) el tiempo de marcha del primer mvil desde A hasta el encuentro. c)
Resolver grficamente.
Solucin.
1 mvil x
E
= v
1
t
2 mvil x
E
= v
2
(t 5)
Resolviendo el sistema de ecuaciones formado, se
obtiene t = 15 s; x
E
= 30 m

B- ENTRE MVILES QUE VIAJAN EN SENTIDOS
OPUESTOS.
Dos mviles parten simultneamente uno al
encuentro del otro, desde dos lugares A y B distantes 1000
m. Ambos lo hacen con MRU. El que sali de A, con v
A
=
3 m/s y el que sali de B con v
B
= 5 m/s. a) Calcular la
posicin de encuentro y el tiempo que dur la marcha. b)
Resolver grficamente.
Solucin.

Mvil A x
E
= v
A
t
Mvil B x
E
= 1000 v
B
t

Resolviendo el sistema de dos ecuaciones con 2 incgnitas formado, se obtiene:
t
E
= 125 s x
E
= 375 m





0 5 10 15
30

20

10
x (m)
t (s)
A x
E
B x (m)
0 1000
v
A

= 3 m/s v
B
= 5 m/s





































0 125 333
t (s)
375
1000
A
B





















2
Cinemtica. MRUV
En la clase pasada se estudi el MRU, un movimiento que posee la particularidad de
transcurrir a velocidad constante. En el da de hoy continuaremos estudiando movimientos
unidimensionales, pero donde a diferencia del MRU, se caracterizan por tener una velocidad
variable en el tiempo. Cuando v cambia de valor en el tiempo, se dice que el movimiento es
variado. Dentro de los movimientos variados, hay uno particularmente importante por el
hecho de ocurrir espontneamente en la naturaleza; es aquel en el que para intervalos iguales
de tiempo (t) le corresponden variaciones iguales de velocidad (v). O sea que el movimiento
es variado, pero las variaciones no ocurren aleatoriamente o imprevisiblemente, sino con
uniformidad. En este caso, el movimiento recibe el nombre de movimiento rectilneo
uniformemente variado. Este es el movimiento que trataremos en la clase de hoy.

Resumiendo, para tener un MRUV se necesita que se cumpla que:
te cons
t
v
tan =


A dicha constante le vamos a poner un nombre; la llamaremos aceleracin y la
indicaremos con a. De modo que:
a
t
v
=


La unidad de la aceleracin en el MKS es m/s
2
. Desarrollando la expresin de arriba y
despejando v, se obtiene la ecuacin horaria de la velocidad en el MRUV:
t a v v
t
v v
a + =

=
0
0

La aceleracin es una cantidad vectorial y su sentido apunta hacia el lado de las
velocidades crecientes.

LEYES DEL MRUV.
1- La posicin es una funcin cuadrtica del tiempo.
2- La velocidad es una funcin lineal del tiempo.
3- La aceleracin es constante.

ECUACIONES HORARIAS DEL MRUV.
De la 1 ley: x = x
0
+ v
0
t + at
2

De la 2 ley: v = v
0
+ at
De la 3 ley: a = cte.
En cada una de las ecuaciones horarias, distinguir las constantes y las variables.
Puede obtenerse otra ecuacin, despejando t en la ecuacin de la velocidad y
sustituyendo en la de la posicin y operando:
v
2
v
0
2
= 2a(x x
0
)
Esta ya no es una ecuacin horaria, pues no figura el tiempo en ella, pero es de gran
utilidad al momento de resolver problemas.

POSIBLES REPRESENTACIONES GRFICAS:

ANALIZAR: 1- qu representa la pendiente en cada grfico.
2- En el grfico x = f(t) las parbolas pueden tener concavidad positiva (como la
A) o concavidad negativa (como la B).

Clasificacin de los MRUV.
En s mismo, el signo de la aceleracin no nos indica si el cuerpo est acelerando o
frenando. Por eso, si en la escuela secundaria les han enseado que cuando la aceleracin es
positiva, el movimiento es acelerado y que cuando es negativa el movimiento es de frenado (o
retardado), mejor perdonar, olvidar y empezar de nuevo.

Clasificaremos los MRUV en dos: MUA (mov. unif. acelerado) y MUF (mov. unif.
frenado). En el primero la rapidez es creciente; en el segundo la rapidez es decreciente. Para
saber cundo se tiene uno u otro, deber aplicarse alguna de las siguientes dos reglas:
1- Si v

aumenta al transcurrir el tiempo, es MUA; caso contrario es MUF.


2- Cuando v y a tienen signos iguales, es MUA; caso contrario es MUF.
En general todo MRUV presenta una etapa donde es MUA y otra donde es MUF.

Ejercicio 1
Trazar los diagramas x = f(t); v = f(t), y a = f(t), en correspondencia mutua, para el
movimiento variado cuyas constantes son: x
0
< 0 v
0
< 0 a > 0
x
v
a
t t
t x
0

v
0

A
B
a=(+)
a=(-)
v(-)
v(+)

Solucin:
Pasos a seguir:
1- Trazar los pares de ejes cartesianos alineados, uno debajo del otro y ubicar en cada
uno a x
0
, v
0
y a, de acuerdo con la consigna. Sabemos que en a = f(t) tendremos
una recta horizontal; en v = f(t) una recta con pendiente y que en x = f(t), una
parbola.
2- Comenzar a razonar a partir del diagrama de la aceleracin, luego pasar al de
velocidad y finalmente llegar al de posicin. En nuestro caso a es positiva: se
dibuja una recta horizontal por arriba del origen. Para pasar al diagrama de
velocidad, tener en cuenta que en este diagrama la aceleracin se presenta como
la pendiente y en base a eso darle a la recta de la velocidad la pendiente que le
corresponda; (en nuestro caso, ascendente). Prestar atencin al sitio donde la
velocidad se anula, en correspondencia con l, en la parbola estar su vrtice.
3- Repetir un razonamiento similar para pasar del diagrama de velocidad al de
posicin. Tambin puede aplicarse la siguiente regla til: si a (+), en el x = f(t)
la parbola presenta concavidad positiva.
4- Para darle al movimiento las denominaciones de MUA y de MUF, ponerle los
signos (+) y (-) a la aceleracin y a la velocidad y aplicar las reglas anteriores.
Respuesta:
Consideremos el movimiento en los puntos A, B y C.

En A la partcula est en x < 0, se mueve en el
sentido x frenando. (v y a tienen signo opuesto): MUF

En B la partcula est en x < 0, instantneamente en
reposo (V
B
= 0) y a punto de moverse en el sentido +x.

En C la partcula contina en x < 0; se mueve en el
sentido +x incrementando su rapidez (v y a tienen el mismo
signo): MUA.

Conclusin: A la izquierda de B el movimiento es un
MUF; a la derecha de B es un MUA.

Ver figuras ilustrativas para cada punto, abajo.




Ejercicio 2.
dem para: x
0
> 0; v
0
> 0; a < 0
Solucin:
x
0

v
0

x
v
a
t
t
t
+
-
+
A B
C
P 0 x P 0 x P 0 x
a a a
v v
B
=0 v
En punto A En punto B En punto C
Razonando de la misma manera que el anterior, se obtienen los siguientes diagramas:
A la izquierda del punto B el movimiento es MUF; a la
derecha del punto B es un MUA.

Hacer un anlisis de las caractersticas del movimiento
en cada uno de los puntos indicados: A, B y C, de manera
similar a como se hizo en el ejercicio 1.

Finalmente aprovechar este ejercicio para mostrar que
cuando la aceleracin es negativa, en el diagrama x = f(t) la
parbola tiene concavidad negativa.

Ejercicio 3.
Encuentro en MRUV.
Un automvil viaja a 144 km/h cuando el conductor
observa que, a 200 m de distancia, un camin viene de frente a
54 km/h. Si ambos comienzan a frenar con a = 3 m/s
2
, hallar:
a) La velocidad de cada uno en el momento del choque.
b) Trazar x = f(t) para ambos mviles.
Solucin:
1- Reduciendo unidades de velocidad, resultan: v
A
= 40 m/s v
C
= 15 m/s.
2- Esquema del problema:
3- Planteo de las ecuaciones de posicin, para el encuentro:
Para el auto: x
E
= 0 + 40t 3t
2

Para el camin: x
E
= 200 15t + 3t
2

Igualando: 40t 1,5t
2
= 200 15t + 1,5t
2

Las soluciones de esta cuadrtica son: t
1
= 13,3 s; t
2
= 5 s
t
1
se descarta porque el x
E
calculado con l
da un valor que est afuera del intervalo 0-
200 m. El clculo de x
E
da 162,5 m

a) v
A
= 40 35 = + 25 m/s
v
C
= -15 + 35 = 0 m/s

b) Observando el esquema, vemos que ambos
movimientos son MUF. La parbola que
corresponde al automvil tiene concavidad
negativa, mientras que la que corresponde al
camin tiene concavidad positiva.





x

x
0




0
v
v
0


0



a
0
t
t
t
A
B
C
Auto Camin
0 200
x (m)
a
A
v
A
v
C
a
C

C
A
162,5
200
x (m)
5
t (s)

Cada libre y tiro vertical.
Dentro de los MRUV, hay dos movimientos particularmente importantes por el hecho
de ocurrir espontneamente en la naturaleza, que son la cada libre y el tiro vertical. El primero
principalmente, ha interesado a filsofos y cientficos desde la antigedad. En el siglo 4 a JC
Aristteles pensaba (errneamente) que los objetos pesados caan con mayor rapidez que los
ligeros, en proporcin a su peso. Diecinueve siglos despus, Galileo afirm que los cuerpos
caan todos con la misma aceleracin, independientemente de su peso.

Los experimentos muestran que si puede omitirse el efecto del aire, Galileo est en lo
cierto. Definiremos estos movimientos del siguiente modo:

Son los movimientos que realiza un cuerpo que se encuentra en las proximidades de la
superficie terrestre bajo la nica influencia de la atraccin de la Tierra (fuerza peso) y en
ausencia de la atmsfera (vaco).

Sobre la definicin reflexionar y explicar:
a) Por qu dice en las proximidades de la superficie terrestre?

b) Por qu debe condicionarse a la ausencia de la atmsfera? Sin atmsfera, todos los
cuerpos, cualquiera sea su peso, caen con la misma aceleracin (constante) que llamaremos
gravedad (g). El valor de g depende slo de la Tierra y es de 9,8 m/s
2
. El vector g

tiene
direccin vertical y sentido hacia abajo, siempre.

c) Para resolver problemas relacionados con estos movimientos cuya trayectoria es vertical,
deber adoptarse un eje de referencia vertical que por costumbre llamaremos y.El origen y el
sentido podrn elegirse libremente.

d) Las ecuaciones horarias de estos movimientos son:
y = y
0
+ v
0y
t + gt
2

v
y
= v
0y
+ gt
v
y
2
v
0y
2
= 2g(y y
0
)
Estas ecuaciones horarias, deben escribirse siempre con todos sus trminos positivos,
tal como se presentaron arriba. Si alguna de las constantes (v
0
; g) fuera negativa, ser al hacer
el reemplazo en la frmula por su valor, que se producir el cambio de signo. Para determinar
fcilmente el signo de estas constantes, siempre hay que dibujar el sistema de referencia que se
ha elegido y observar en l si cada uno de los vectores que representan a dichas constantes
tienen el mismo sentido o el contrario en comparacin con el del eje de referencia.

Ejercicio 4.
Desde un globo, a una altura de 175 m sobre el suelo y ascendiendo con una velocidad
de 8 m/s, se suelta un objeto. Calcular: a) La altura mxima alcanzada por ste. b) La posicin
y la velocidad del objeto al cabo de 5 segundos. c) El tiempo que
tarda en llegar al suelo.
NOTA: Utilizar g = 10 m/s
2

Solucin:
1- Adoptar el eje de referencia.
a) m y
g
v v
y
mx
2 , 178 175
20
64 0
2
) (
0
2
0
2
= +

= +

=
y
0
g y
0
175m v
0y
= 8 m/s

b) y
5
= 175 m + (8 m/s)(5 s) (10 m/s
2
)(5 s)
2
= 90 m
v
5
= 8 m/s (10 m/s
2
)(5 s) = - 42 m/s

c) Al llegar al suelo es y = 0; luego:
0 = 175 m + (8 m/s)t (10 m/s
2
)t
2
t = 6,77 s

Valores medios e instantneos
de velocidad y aceleracin.
Hemos definido la velocidad media como:

1 2
1 2
12
12
t t
x x
t
x
v
m

=
|

\
|

=
Consideremos el caso de una partcula que
realiza un MRUV, un movimiento donde la grfica
de x = f(t) es una parbola; la figura ilustra un caso.

La pendiente de la cuerda que pasa por los
puntos 1 y 2 mide la velocidad media de la partcula
entre esos puntos del recorrido. La velocidad media representa la velocidad que
hipotticamente debera tener una partcula para recorrer con MRU el mismo trayecto
empleando el mismo tiempo. Pero la v
m
no expresa la velocidad real de la partcula, instante a
instante. Ntese que el concepto de v
m
obliga a elegir de antemano dos puntos de la trayectoria:
inicial y final del intervalo x. La v
m
depende de 2 puntos.

Cuando la velocidad de la partcula est cambiando continuamente su valor en el
tiempo, el concepto de velocidad media no resulta til y se lo reemplaza por el de velocidad
instantnea. El valor de la velocidad instantnea solamente depende de un punto.

Retomando el ejemplo del grfico, si se desea obtener la velocidad instantnea en el
punto 1 (v
1
) deberamos acercar el punto 2 al 1 reduciendo el intervalo t. La pendiente de la
curva se ira acercando cada vez ms a la curvatura de la parbola. Finalmente cuando el punto
se confunda con el 1 (y el intervalo t tienda a cero), la cuerda se habr convertido en la
tangente a la parbola en el punto 1. En este momento, el valor de la pendiente de la recta
tangente expresa la velocidad instantnea. Lo descripto se escribe, con la notacin del clculo
infinitesimal as:

1
0
v
t
x
lm
t
=



y finalmente v
dt
dx
= (1)
Con la aceleracin podemos aplicar un razonamiento similar y a partir del concepto de
aceleracin media:

1 2
1 2
12
12
t t
v v
t
v
a
m

=
|

\
|

=
se llega a la aceleracin instantnea:

dt
dv
t
v
a
lm
t
=
|

\
|

=
0
(2)
Una combinacin de estas dos ltimas expresiones nos permite decir tambin que
0 t
1
t
2

x
1

x
2
x
t
1
2
x
t


2
2
dt
x d
a =
Ejercicio 5.
Utilizar las relaciones (1) y (2) para deducir, a partir de la ecuacin de la posicin en el
MRUV, las ecuaciones de la velocidad y la aceleracin.

Obtencin de ecuaciones de velocidad y
posicin, por integracin de a = f(t).
Abundan en la vida diaria ejemplos de movimientos variados donde
la aceleracin no es constante. Esto es lo que ocurre por ejemplo, cuando pisamos el pedal del
acelerador de un automvil; cuanto mayor sea la rapidez del auto, ms lentamente adquirir
rapidez adicional; un auto comn tarda el doble en acelerar de 50 km/h a 100 km/h que en
acelerar de 0 a 50 km/h.

En situaciones como esta, al no ser constante la aceleracin, no son vlidas las
ecuaciones del MRUV y no pueden ser utilizadas en un problema. Sin embargo las relaciones
(1) y (2) continan siendo vlidas. Recordar que matemticamente integrar significa recorrer el
camino inverso al de cuando se deriv.

Las relaciones (1) y (2) permitieron a partir de una ecuacin x = f(t) conocida, encontrar
las ecuaciones de velocidad y aceleracin.

Si se conociera la ecuacin de a = f(t), por integracin se podrn obtener las ecuaciones
de velocidad y posicin. Las relaciones a emplear son:

+ =
2
1

1 2
t
t
dt a v v (3)

+ =
2
1

1 2
t
t
dt v x x (4)
Si en el instante inicial vale t
1
= 0 y para l, llamamos v
0
a la velocidad inicial y x
0
a la
posicin inicial, las ecuaciones de arriba pueden escribirse de manera ms prctica as:

+ =
t
dt a v v
0
0
(5)

+ =
t
dt v x x
0
0
(6)
Este procedimiento es interesante porque afortunadamente en los movimientos con
aceleracin variable la informacin ms accesible que se consigue es la de a = f(t) antes que las
de v = f(t) o de x = f(t).

Ejercicio 6.
Un automvil recorre un tramo rectilneo de una autopista en el sentido +x. En el
instante t = 0, cuando su velocidad es de 10 m/s, pasa al lado de un letrero que est en x = 50
m. Su aceleracin es una funcin del tiempo:
a = 2 m/s
2
(0,10 m/s
2
)t
a) Deducir las expresiones de v = f(t) y de x = f(t)
b) En qu momento es mxima su velocidad?
c) Cunto vale dicha velocidad mxima?
d) Dnde est el automvil cuando alcanza la velocidad mxima?
Slo para
UNLaM
e) Si el auto se detiene, en qu instante y posicin ocurre?
f) Trazar los diagramas cartesianos del movimiento, en correspondencia mutua.
Solucin:
1- Identificar las condiciones iniciales: para t = 0, v
0
= 10 m/s; x
0
= 50 m
a) Ecuacin de la aceleracin:
a = 2 0,1t
Aplicando la (5):

+ =
t
dt a v v
0
0
= v
0
+ ( 0,1t)dt
v = 10 m/s + 2 m/s
2
t 0,05 m/s
3
t
2


Aplicando la (6):

+ =
t
dt v x x
0
0
= x
0
+ (10 2t 0,05t
2
)dt
x = 50 m + 10 m/st + 1 m/s
2
t
2
0,017t
3

b) La velocidad se hace mxima en el momento en que deja de aumentar y comienza a
disminuir. En ese instante, 0 = = a
dt
dv
. Entonces, igualamos a cero la expresin de la
aceleracin: 0 = 2 0,1t t = 20 s

c) Reemplazando en la expresin de la velocidad, a t por el valor de 20 s y calculando, resulta:
v = 30 m/s
d) Haciendo un trabajo similar en la expresin de la posicin, resulta: x = 517 m
e) Cuando el auto se detiene se hace v = 0. Entonces, igualando a cero la expresin de la
velocidad, resulta. 0,05t
2
2t 10 = 0 ecuacin cuadrtica cuya solucin es t = 45 s. (La
otra solucin se descarta por tratarse de un tiempo negativo).

Para conocer la posicin de detencin, se ingresa este valor de t en la ecuacin de
posicin y se calcula. Resulta x = 4044 m



FUNCIN LINEAL






FUNCIN CUADRTICA




FUNCIN CBICA
Inflexin en t = 20 s. (A la izquierda de ese valor,
concavidad positiva; a la derecha, concavidad negativa.

Ejercicio 7.
Utilizar las ecuaciones (5) y (6) para obtener las ecuaciones de v = f(t) y de x = f(t) a
partir de que a = cte en el MRUV.

a


0

v
30

10
0

x
4044

517

50
0
2
20 45 t
t
t





















3
Cinemtica en dos dimensiones
Tiro oblicuo.

Cinemtica vectorial.
En las clases anteriores se estudi el movimiento de una partcula sobre una trayectoria
rectilnea. Como una recta tiene una sola dimensin, basta una sola coordenada (x) para dejar
expresada la posicin en un cierto i9natante. Hoy estudiaremos el caso de partculas que se
mueven sobre un plano, como la tiza cuando se desliza sobre la pizarra, un barco que navega
en el mar o un objeto que da vueltas sobre una mesa. Como un plano tiene 2 dimensiones (x;y)
la posicin de la partcula en cada instante requerir de 2 coordenadas.

La cinemtica en 2 dimensiones continuar empleando los mismos conceptos ya
estudiados, de posicin, desplazamiento, velocidad y aceleracin. Ahora adquiere real
importancia recordar que todos estos conceptos son vectoriales. En el movimiento rectilneo
todos estos vectores tienen una misma direccin: la de la trayectoria. Ahora en cambio, tendrn
distintas direcciones (en el plano e incluso fuera de l).

Vector posicin ( r

)
El primer paso en el estudio de todo movimiento consiste en poder describir la posicin
de la partcula en un cierto instante.

La figura muestra la trayectoria de una partcula en un plano cartesiano (x;y); ella pasa
primero por el punto 1 y ms tarde por el punto 2.

Cuando est en 1, la posicin est dada por el vector posicin ( r

1
). Los vectores
posicin tienen su origen en el origen del sistema de referencia y su
extremo se apoya en el punto cuya posicin se desea expresar. Las
coordenadas cartesianas del punto, son las componentes de r

:
r

= xi + yj (1)
Cuando la partcula llegue al punto 2, su posicin ser
expresada por r

2
.

Vector desplazamiento (r

)
Este vector expresa el cambio de posicin experimentado por
la partcula:
r

= r

2
- r

1
= (x
2
x
1
)i + (y
2
y
1
)j
Grficamente r

es un vector con origen sobre el extremo de r

1
y con extremo sobre
el extremo de r

2
.

Vector velocidad media (
med
v

)
Como ya sabemos,
1 2
1 2
t t
r r
t
r
v
med

(2)
Como es resultado de un cociente entre un vector y un escalar positivo, la velocidad
media es otro vector de la misma direccin y sentido que r

.

Velocidad instantnea (v

)
Es el lmite de la velocidad media cuando el intervalo de
tiempo se aproxima a cero:

dt
r d
t
r
v
lm
t

=
0
(3)
El vector velocidad instantnea en un punto tiene por
direccin la de la tangente a la trayectoria, su sentido es el del
movimiento y su mdulo es la rapidez de la partcula.

La velocidad instantnea se
relaciona con sus componentes
cartesianas as:

y x
v v v

+ =
j v i v v
y x
+ =


Teniendo en cuenta que v
x
= dx/dt y v
y
= dy/dt, queda:
j
dt
dy
i
dt
dx
dt
r d
v + = =

(4)
La rapidez (esto es el mdulo de v

) es:

2 2
y x
v v v v + = =

(5)
La direccin de v

se expresa mediante el ngulo calculable a partir de:



x
y
v
v
tg = (6)

Vector aceleracin media (
m
a

)
1
2
r

1

r

2
0
y
x
1
2
0
y
x
v
1

v
2

0
x
y

v
y
v
x

v

1 2
1 2
t t
v v
t
v
a
m

(7)

El vector
m
a

tiene la misma direccin y sentido


que el v

, direccin que no es la de la trayectoria.



Vector aceleracin instantnea
( a

)
Cuando el punto 2 se aproxima al 1 y tiende a confundirse con l, el tringulo vectorial
auxiliar v v v

, ,
2 1
tiende a tomar esta forma:
y el vector aceleracin instantnea apunta decididamente hacia el lado
de la trayectoria donde se encuentra su centro de curvatura.

Por otro lado,
dt
v d
t
v
a
lm
t

=
0
(8)
Ntese que cualquier movimiento curvo es siempre acelerado, no importa si la rapidez
es constante o no. A los fines de clculo, resulta til esta expresin:
j
dt
y d
i
dt
x d
j
dt
dv
i
dt
dv
a
y
x

2
2
2
2
+ = + =

(9)

Componentes intrnsecas de la aceleracin.
En todo movimiento en dos dimensiones (movimiento curvo) el vector velocidad puede
cambiar por 2 razones: porque cambia su mdulo o porque cambia su direccin.

Como la aceleracin a

mide los cambios que se producen en la velocidad, deber


registrar tanto uno como otro cambio. Para poder apreciar separadamente la magnitud del
cambio de direccin, de la magnitud del cambio de mdulo, se descompone al vector a

en dos
direcciones. Estas direcciones no son las del sistema de referencia, sino dos direcciones ligadas
al movimiento mismo, en el instante considerado; por ello, a estas componentes de la
aceleracin se las llama componentes intrnsecas.

Cules son estas direcciones? Una es la del movimiento
(que es la de la tangente a la trayectoria) y la otra es perpendicular a
la primera (que por ser perpendicular a la tangente, tiene la
direccin del radio de curvatura instantneo; como tal, se dirige al
centro de curvatura).

Las componentes de la aceleracin en estas direcciones se
llaman aceleracin tangencial (
t
a

) y aceleracin normal (
n
a

). Ellas
son las componentes intrnsecas de la aceleracin.

t
a

mide los cambios en la rapidez. Se deduce que


dt
v d
a
t

=
n
a

mide los cambios en la direccin de la velocidad. Se deduce que


r
v
a
n
2
=

donde r es el
radio de curvatura.
1
2
0
y
x
v
1

v
2

v
1
v
2
v
a
m
v
1
v
2
v
a
1
0
y
x
a
t

a
n
a
Ejercicio 1.
Una partcula se mueve en un plano x;y; sus coordenadas varan en el tiempo segn:
x = 2 m (0,25 m/s
2
)t
2

y = (1 m/s)t + (0,025 m/s
3
)t
3

Determinar:
a) Para t
2
= 2 s, cules son las coordenadas de la partcula y cul su distancia al origen?
b) Para el intervalo entre t
0
= 0 y t
2
= 2 s. obtener el vector desplazamiento y el vector
velocidad media.
c) Representar grficamente la trayectoria.
d) Deducir la expresin general del vector velocidad instantnea.
e) Calcular la velocidad instantnea para t
2
= 2 s, expresndola en forma cartesiana y en
forma vectorial.
f) Para el intervalo entre t
0
= 0 y t
2
= 2 s, obtener las componentes de la aceleracin
media.
g) Calcular la aceleracin instantnea para t
2
= 2 s.
Solucin:
a) Se toman las expresiones de x = f(t) e y = f(t) del enunciado, se reemplaza t por el valor 2 s y
se calcula: x
2
= 2 0,25 t
2
= 2 0,25 2
2
= 1 m
y
2
= 1 t + 0,025 t
3
= 1 2 + 0,025 2
3
= 2,2 m
La distancia de la partcula al origen se calcula con Pitgoras:
m d 4 , 2 ) 0 2 , 2 ( ) 0 1 (
2 2
= + =

b) Calculamos previamente las coordenadas de posicin para t
0
= 0:
x
0
= 2 0,25 t
2
= 2 m
y
0
= 1 t + 0,025 t
3
= 0 m

0 2 02
r r r

= = (x
2
i + y
2
j) (x
0
i + y
0
j)
= (x
2
x
0
)i + (y
2
y
0
)j = (1-2)i + (2,2 0)j = -1 i + 2,2 j
j
s
m
i
s
m
s
j i
t
r
v
m
1 , 1 5 , 0
) 0 2 (
2 , 2 1
02
+ =

+
=



c) Para poder trazar la trayectoria, utilizaremos las coordenadas (x;y) de la partcula, en los
instantes t
0
= 0 s; t
1
= 1 s; t
2
= 2 s.
En t
0
= 0 s la partcula est en 0(2 ; 0) m (ya calculado).
En t
1
= 1 s la partcula est en 1(1,75 ; 1,025) m
En t
2
= 2 s la partcula est en 2(1 ; 2,2) m (ya
calculado).

d) v

= v
x
i + v
y
j donde v
x
= dx/dt v
y
= dy/dt
v
x
y v
y
se obtienen derivando las ecuaciones de x = f(t)
e y = f(t) del enunciado; luego v
x
= -0,5t
v
y
= 1 + 0,075t
2

Finalmente: v

= (-0,5t)i + (1 + 0,075t
2
)j

e) Para hallar la v
2
en t
2
0 2 s, se reemplaza en la
expresin general de v

la t por su valor y se calcula:


v

2
= (-0,5 t) i + (1 0,075 t
2
) j
= (-0,5 2) i + (1 0,0752
2)
j
= -1 i + 1,3 j EXPRESIN CARTESIANA

0
2,2
1
d
P
y
x
r
0
r
1

r
2
0
1
2
v
0
v
1

v
2
y (m)
x (m)
2
1
1 2

s
m
v v V
y x
6 , 1 ) 3 , 1 ( ) 1 (
2 2 2 2
2
= + = + =


' 43 52
1
3 , 1
' =

= = tg arc
v
v
tg arc
x
y


A este valor que da la calculadora (que corresponde a un ngulo del 4 cuadrante) se le
deben sumar 180 para que quede expresado en forma correcta. Luego:
= - 5243 + 180 = 12734
Finalmente v

2
= [1,6 m/s; 12734] FORMA VECTORIAL.

f) Las componentes de la a
m
son:
a
m x
=
2
0 2
0 2
5 , 0
0 2
0 1
s
m
t t
v v
t
v
x x x
=


a
m y
=
2
0 2
0 2
15 , 0
0 2
1 3 , 1
s
m
t t
v v
t
v
y yx y
=


donde v
x
= -0,5t v
x 2
= -1 v
x 0
= 0
v
y
= 1 + 0,075t
2
v
y 2
= 1,3 v
y 0
= 1

g) a
x
=
2
5 . 0
) 5 , 0 (
s
m
dt
t d
dt
dv
x
=

=
a
y
=
2
2
15 . 0
) 075 , 0 1 (
s
m
dt
t d
dt
dv
y
=
+
=
Para t
2
= 2 s: a

= a
x
i + a
y
j a

= (-0,5 i + 0,30 j) m/s


2


Tiro oblicuo.
En la cinemtica bidimensional los dos movimientos ms importantes para estudiar son
el tiro oblicuo y el movimiento circular. Esta clase se completar estudiando el primero de
ellos. Tiro oblicuo es el movimiento que realiza cualquier proyectil (bomba abandonada desde
un avin, una bala de revlver o de can o simplemente una pelota.

Se estudiar este movimiento en su forma ms pura, que es tambin la ms simple, para
lo cual se lo despojar de toda perturbacin que lo complique. Por eso se supondr que el
movimiento tiene lugar en el vaco, en ausencia de atmsfera, por las razones expuestas cuando
se estudi la cada libre de los cuerpos. De todos modos conviene insistir en que no tomar en
cuenta la resistencia del aire es algo que se hace no porque su efecto sea despreciable, sino para
evitar complicaciones verdaderamente grandes. En verdad, el efecto de la resistencia del aire
no es para nada despreciable, al punto que entre el clculo de un tiro oblicuo en el vaco y el
mismo tiro transcurriendo en la atmsfera se pueden tener diferencias en valores de hasta un 30
%.

En Dinmica se estudia que toda aceleracin es debida a una fuerza; no hay aceleracin
sin fuerza. Entonces, en aquellas direcciones en que hay una fuerza (o una componente de
fuerza), el movimiento es acelerado, mientras que en aquellas direcciones en que no hay fuerza
(ni componente de fuerza), el movimiento ser rectilneo uniforme.

Supngase el caso de una bala de can recorriendo su trayectoria. Qu fuerzas actan
sobre ella?: solamente la debida a su peso, que es vertical. De acuerdo con el concepto anterior,
en la direccin vertical habr aceleracin: la de la gravedad, vertical y constante, mientras que
en la direccin horizontal al no haber fuerza ni componente de fuerza, la aceleracin ser nula.
Conclusin, si un tiro oblicuo se proyecta sobre los ejes cartesianos, en la direccin del eje y
se tendrs un tiro vertical y/o cada libre, que es un MRUV y en la direccin del eje x se
tendr un MRU.

Esto se aprovecha para plantear los problemas de tiro oblicuo a travs de sus
componentes vertical y horizontal, aplicando en cada caso las ecuaciones horarias que
correspondan.

Es comn que en un problema de tiro oblicuo los datos sean la rapidez inicial de
lanzamiento (v
0
) y el ngulo de tiro (). Si es as, para r esolverlo deben seguirse los siguientes
pasos:
1- Adoptar un sistema de referencia, con su origen (por conveniencia) en el punto de
lanzamiento.
2- Calcular el valor de las componentes de la velocidad inicial segn:
v
0 x
= v
0
cos
v
0 y
= v
0
sen
3- Escribir las ecuaciones de posicin y velocidad segn la direccin de cada eje:

En y (MRUV) En x (MRU)
y = y
0
+ v
0 y
t + g t
2
(1)
v
y
= v
0 y
+ g t (3)
x = v
0 x
t (2)

Empleando convenientemente las ecuaciones de arriba, siempre ser posible resolver un
problema de tiro oblicuo.
NOTA: Cuando = 0, el tiro oblicuo cambia su nombre por el de tiro horizontal. En este
caso, en el sistema de referencia a utilizar conviene invertir el sentido del eje y.

Ecuacin de la trayectoria.
Las ecuaciones (1) y (2) reciben el nombre de ecuaciones paramtricas; son del tipo:
x = f(t) y = f(t)
siendo t el parmetro. Una ecuacin del tipo:
y = f(x)
se llama ecuacin geomtrica o ecuacin de la trayectoria. Para obtenerla a partir de las
ecuaciones paramtricas, se hace lo siguiente:
1- Se escriben las ecuaciones (1) y (2), previo reemplazo de v
0y
y v
0x
por sus
expresiones:
y = y
0
+ v
0
sen t + g t
2

x = v
0
cos t
2- En la ecuacin de abajo se despeja t:
cos
0
v
x
t =
3- Se sustituye esta expresin de t en la ecuacin de arriba. Ordenando, se obtiene:

| |
0
2
2
0
cos 2
y X tg X
v
g
y + +
(



sta es una ecuacin cuadrtica del tipo Ax
2
+ Bx + C donde

2
0
cos 2 v
g
A = B = tg C = y
0

Estas funciones tienen como representacin grfica una parbola. Queda demostrado as
que:
Todo tiro oblicuo tiene por trayectoria a una parbola.

Si se ha adoptado un eje y con sentido hacia arriba, no olvidar que en la ecuacin de
la trayectoria g deber ingresarse con signo negativo.

Ejercicio 2.
Desde la parte superior de una torre se arroja una pelota con velocidad de 10 m/s y un
ngulo de elevacin de 37. Sabiendo que la torre tiene 30 m de altura, calcular:
a) cunto tarda la pelota en llegar al suelo?
b) la distancia entre la base de la torre y el punto de impacto.
c) la rapidez de la pelota en el momento del impacto.
d) obtener la ecuacin de la trayectoria.
e) Hallar los valores de las componentes intrnsecas de la aceleracin, en el punto R de la
trayectoria, 2 segundos despus del lanzamiento.
Solucin:
v
0 x
= v
0
cos = 10cos 7 = 8 m/s
v
0 y
= v
0
sen = 10sen 37 = 6 m/s

a) y = y
0
+ v
0y
t + gt
2

0 = 30 + 6t 5 t
2

Resolviendo: t = 3,12 s; La otra solucin se
descarta por ser t negativa.

b) L = v
x
t = 8 m/s 3,12 s = 25 m

c) v
yP
= v
0y
+ gt = (6 103,12) m/s = -25,2 m/s

s
m
v
P
44 , 26 ) 2 , 25 ( 8
2 2
= + =

d) | |
0
2
2
0
cos 2
y X tg X
v
g
y + +
(


| | m X tg X
s
m
s
m
y 30 37
8 , 0 ) 10 ( 2
10
2
2 2
2
+ +
(
(


=
y = -0,077 (1/m)X
2
+ 0,75 X + 30 m

e) Clculo de las componentes de v
R
:
v
R x
= v
0 x
= 8 m/s
v
R y
= v
0 y
+ gt = 6 102 = -14 m/s 30 57 , 0 = = = tg arc
v
v
tg arc
Ry
Rx

Clculo de las componentes intrnsecas de la aceleracin:
a
n
= gsen = 100,5 = 5,0 m/s
2

a
t
= gcos = 100,86 = 8,6 m/s
2

30
y (m)
x
L
v
0
= 10 m/s
37

R
v
R
v
Rx
v
Ry

90

a
t

a
n

g
Tambin por semejanza de s se puede poner: a
n
= g
R
Rx
v
v
a
t
= g
R
Ry
v
v





































































4
Cinemtica en dos dimensiones
Movimiento circular

Previo: Efectuar las siguientes 2 consideraciones:
1- Rever los productos entre vectores (escalar y vectorial) y la regla de la mano
derecha.
2- Recordar la regla de derivacin de un producto: d(uv) = du v + u dv

Movimiento circular.

Es el que describe una partcula que se mueve teniendo por trayectoria una circunferencia.

La figura muestra un disco girando
alrededor del eje e, que el perpendicular al
disco y pasa por su centro. Un punto como
el P describe un movimiento circular.

La posicin del punto P la da el
vector posicin r

.
En un tiempo t el punto describe el arco
s

, arco que es abarcado por el ngulo

.

Velocidad.
0
r
eje
P
s O
La velocidad del punto P puede expresarse de dos maneras:
a) Velocidad tangencial:
dt
s d
v

= Unidad: m/s
b) Velocidad angular:
dt
d

= Unidad: 1/s = radin/s


Recordar el concepto de radin.
Ambas formas de expresar la velocidad estn relacionadas:
arco = ngulo x radio r d s d

=
r
dt
d
dt
s d

=

r v

=
Momento de ubicar en el grfico los vectores ds, d y r.

Aceleracin:
Como v

es un vector, la velocidad cambia porque se producen cambios en la direccin


y/o cambios en el mdulo. Sean cuales fueren los cambios que sufre v

, es la aceleracin quien
mide la magnitud de los mismos:

dt
r d
dt
v d
a
) (


= =


La derivada de un producto vectorial se hace de la misma manera que la de un producto
comn, slo que debe respetarse el orden de los factores:

dt
r d
r
dt
d
a

+ =


donde

=
dt
d
(aceleracin angular, en 1/s
2
) y v
dt
r d

= (velocidad tangencial, en m/s).



Ubicar

en el grfico, en la misma direccin y sentido que

d .

Luego nos queda: v r a

+ =
donde en el 2 miembro, el primer trmino es
t
a

y el segundo es
n
a

. Ellas son las


componentes intrnsecas de la aceleracin.

Ubicar
t
a

y
n
a

en el grfico, razonando cada producto vectorial con la regla de la


mano derecha. Al ubicarlos, se comprende el motivo de sus nombres.

t
a

mide la rapidez con que v

cambia de mdulo.
n
a

mide la rapidez con que v

cambia de direccin.

t
a

y v

tienen el mismo sentido cuando el mdulo de v


aumenta, y tienen sentidos opuestos cuando el mdulo de v


disminuye.

Para hallar los mdulos de las componentes de la
aceleracin, se hace: /
t
a

/ = r [m/s
2
]
/
n
a

/ = v =
2
r = v
2
/ r [m/s
2
]

r
d
0
P
v

r

0
P
v
d
a
n
a
t

Movimiento circular uniforme (MCU).
Si a
t
= 0 = 0 = cte: el movim. es un MCU. Las
ecuaciones horarias del MCU guardan analoga formal con las ecuaciones del MRU:

x = x
0
+ vt =
0
+ t

PERODO: (T) Es el tiempo empleado por la partcula para dar una vuelta completa.


(

= =
s T empleado tiempo
giro un de angular ento desplazami 1 2

Luego, | | s T

2
=
FRECUENCIA: (f) Es la cantidad de vueltas realizadas por la partcula en la unidad de
tiempo.

(

= =
s T
f
1
2
1

o Hertz (Hz)
Es comn tambin expresar la frecuencia en rpm sigla que expresa
uto
vueltas
f
min
=
La equivalencia entre unidades es: 60 rpm = 1 Hz

Movimiento circular uniformemente variado (MCUV).
En este caso es 0 y tiene un valor constante (valor que puede ser positivo o
negativo). Las ecuaciones horarias del MCUV guardan analoga formal con las del MRUV:

x = x
0
+ v
0
t + at
2
=
0
+
0
t + t
2


v = v
0
+ a t =
0
+ t

v
2
v
0
2
= ax
2

0
2
=


Movimiento relativo.
Hasta ahora hemos hablado del movimiento de las partculas (sea con v = cte, con v
variable o con v = 0; con o sin aceleracin, etc) dando por sobreentendido que lo hacan con
respecto de las rutas, las localidades, en definitiva, respecto de la Tierra.

Sin embargo hay casos en que nosotros, automticamente, usamos otros sistemas de
referencia que no son la Tierra; por ejemplo, le decimos a un nio que viaja en tren a nuestro
lado que se quede quieto. La nica forma en que la criatura puede cumplir con nuestro deseo
sera arrojndose por la ventanilla o correr hacia el furgn de cola a 100 km/h (no era esa
nuestra intencin), y por ms que se tranquilice, no podr hacer otra cosa que seguir
movindose a 100 km/h, junto con el tren y dems pasajeros adultos. En este caso, sin darnos
cuenta, usamos el sistema de referencia fijo al tren, aunque jams oiremos que el tren se
encuentra quieto, dicho por un pasajero, a menos que efectivamente est detenido con respecto
a la Tierra.

La condicin de movimiento o falta de l- es, pues, relativa a los sistemas de
referencia, seamos o no concientes de ello al hacer nuestras afirmaciones. Moverse no es algo
propio de un cuerpo, sino algo que concierne a dos cuerpos.

No es fcil convencer a nadie de que la montura de un potro indmito est en reposo, si
se elige un sistema de referencia adecuado, mientras que la Tierra, con sus rboles e
instalaciones, se sacude hacia uno y otro lado. Efectivamente, aunque tericamente podra
hacerse tal afirmacin, no es frecuente hacerla, ni prctico.

Parece que lo ms prctico consiste en utilizar a la Tierra como referencia universal, si
no fuera porque, entre otras cosas, tambin deseamos decir a veces que la Tierra se mueve.
Ninguna dificultad habr si decimos respecto de qu (del Sol, de las estrellas, etc.). Segn este
enfoque relativista, la discusin entre Galileo y la inquisicin sobre si la Tierra se mueve o no
lo hace, parecera por ahora- carecer en absoluto de significado, al no haber referencia a
sistema patrn alguno. Por entonces consideraban que la quietud o el movimiento eran
atributos de cada cuerpo y no de pares de ellos. Cuando decan se mueve o no se mueve no
mencionaban otro cuerpo. Es ms, dejaban entrever que tenan la idea de un espacio absoluto
que servira como sistema de referencia universal, aunque no lo decan en forma explcita.

Como es fcil imaginar, la descripcin de un movimiento depende del sistema de
referencia que se utilice: el movimiento de la pa fonogrfica sobre un disco, respecto del
disco, sigue una trayectoria en forma de espiral plana, cuya longitud podra estimarse en 300
m, que la pa recorre totalmente a una velocidad variable. En cambio, respecto de la
habitacin, el movimiento de la pa consiste en un arco de crculo de no ms de 25 cm de
recorrido, y con una velocidad de mdulo constante.

No slo la descripcin de la trayectoria cambia con el sistema de referencia; tambin lo
hacen la velocidad y la aceleracin.

La transformacin de Galileo.
1 CASO: DOS SISTEMAS DE REFERENCIA QUE SE MUEVEN CON MRU.
Consideremos dos sistemas de referencia, y que uno de ellos se mueve con respecto al
otro de modo que su origen hace un MRU y sus ejes mantienen siempre la misma orientacin.
A ese movimiento se lo llama traslacin uniforme.

En la figura de la derecha, todos los puntos de B se
mueven con velocidad v

BA
(que es la velocidad de B respecto
de A). Recprocamente, A se mueve con respecto a B con
velocidad:
v

AB
= - v

BA
donde el signo menos
indica que v

AB
es opuesta a v

BA.

POSICIONES:
Supongamos ahora una partcula ubicada en un punto P del plano, de la cual
conocemos su vector posicin r

PB
y su velocidad v

PB
en un instante dado, medidos con
respecto al sistema de referencia B (ver figura en la pgina siguiente).

Medida desde A la posicin es otra, aunque su ubicacin fsica en el plano es la misma,
ya que el vector posicin que ahora seala la partcula P desde O
A
es r

PA
.

y
y
x
x
Sistema A
Sistema B
v
BA
Si observamos la figura siguiente, resulta que:
r

PA
= r

BA
+ r

PB

De esta manera se vinculan las posiciones en ambos sistemas.

Analicemos qu ocurre a medida que transcurre el tiempo.
Si la partcula se mueve con respecto a B y adems el sistema se
mueve con respecto a A, los vectores posicin cambiarn, tal que
para un instante t la posicin del punto P respecto del sistema A
ser:

r

PA
= r

BA
+ r

PB


Podemos hallar los desplazamientos:
r

PA
= r

PB
+ r

BA

con r

PA
= r

PA
- r

PA

r

PB
= r

PB
- r

PB
r

BA
= r

BA
- r

BA


VELOCIDADES:
Si consideramos el intervalo de tiempo empleado para estos desplazamientos, podremos
calcular:

t
r
t
r
t
r
PB
A B
PA


de acuerdo con lo visto anteriormente estas son velocidades medias, pero como ya sabemos
pueden definirse tambin las instantneas:
v

PA
= v

BA
+ v

PB
(1)
Hemos obtenido as la ley de adicin de velocidades, que nos dice cmo se
transforman las velocidades de un sistema a otro.

De acuerdo con lo dicho, cuando tenemos 2 sistemas (como el A y el B) en movimiento
mutuo, es imposible asegurar cul de ellos es el que se mueve (si el otro est fijo) o si ambos se
mueven un poco cada uno. Sin embargo comnmente resulta cmodo convenir arbitrariamente
que uno de ellos est fijo y que todo el movimiento lo realiza el otro. Si convenimos que A es
el sistema fijo y B el mvil, la velocidad de la partcula P respecto de A ( v

PA
) se llama
velocidad absoluta, la v

PB
se llama velocidad relativa y la v

BA
se llama velocidad de
arrastre, de modo que la ecuacin (1) puede interpretarse diciendo:

Veloc. absoluta = veloc relativa + veloc de arrastre. (Suma vectorial)

ACELERACIONES:
Haciendo un procedimiento anlogo al anterior, podramos ver cmo se transforman las
aceleraciones, de un sistema a otro, tomando un intervalo de tiempo t y registrando los
cambios de velocidad de la partcula en ese intervalo:


t
v
t
v
t
v
PB
A B
PA


Si, como dijimos, todos los puntos del sistema B tienen la misma velocidad constante,
entonces: 0 =

t
v
BA
lo cual implica que
B A
a a

= (2)
(A)
(B)
O
A

O
B

r
PA
r
PB
P
v
PB
v
BA
O
A
O
B
(A)
(B)
P
r
PA
r
BA
r
PB
Las aceleraciones son las mismas en ambos sistemas (aunque no lo sean las
respectivas velocidades).

2 CASO: DOS SISTEMAS DE REFERENCIA QUE SE MUEVEN CON MRUV.
Supongamos ahora, en cambio, que el sistema B se mueve de tal manera que su origen
hace un MRUV y sus ejes mantienen siempre la misma orientacin. (Por supuesto que existen
otros movimientos posibles de B con respecto a A, aparte de los ya dichos, pero no sern
tratados, como por ejemplo, rotaciones y traslaciones curvilneas).

Luego podemos usar:
PB BA PA
a a a

+ = (3)
Hemos hablado de cmo se transforman posiciones, velocidades y aceleraciones, pero
nada dijimos del tiempo, aunque cuando tomamos intervalos de tiempo para calcular las
velocidades medias, simplemente dividimos por t todos los t rminos de la expresin de
desplazamientos, sin discriminar entre t
A

y t
B
estamos afirmando que el tiempo transcurre
igualmente desde ambos sistemas de referencia. Aunque parezca extrao, esta afirmacin no es
vlida para velocidades grandes, comparables con la de la luz. Pero para velocidades ordinarias
es correcto decir que t
A

y t
B
, y escribir explcitamente la llamada transformacin de Galileo:

B BA A
r t v r

+ =
t
A

= t
B
que permite calcular posiciones y tiempos en el sistema A conociendo posiciones y tiempos en
el sistema B.

O sea con estas ltimas y con las (1) y (3), podemos describir los movimientos del
chico, camine, corra, salte, se quede quieto o no, dentro de un tren que se desplaza con MRU o
con MRUV, respecto de cualquier pasajero sentado o desde el andn de alguna estacin.

Ejercicio 1.
Un bote cruza un ro de 30 m de ancho con una rapidez de 4 m/s respecto del agua y
orientado de tal forma que, si las aguas estuvieran en reposo, cruzara perpendicularmente a las
orillas. El bote parte de un punto O, ubicado sobre una de las orillas y llega a otro punto B
ubicado sobre la otra orilla, distante 50 m. Cul es la velocidad del agua?
Solucin:
Una forma sencilla para no equivocarse es escribir la
frmula vectorial de transformacin de velocidades de la
siguiente manera:

0 = + +
CA BC AB
v v v

(4)

Notar la rotacin de los subndices: AB, BC, CA. Se
trata de una expresin muy simtrica y fcil de entender y de recordar. En cuanto a A, B y C
son lo que uno desee: el sistema A; el sistema B; la lancha; el cuerpo; lo que haga falta:
siempre funciona bien.

Recordando esta otra expresin: v

DE

=

- v

ED
que nos dice que si
permutamos los subndices debemos invertir el sentido del vector, que la velocidad del objeto
D respecto del E es opuesta a la velocidad de E respecto de D.

Hecha esta introduccin, elegimos: A = agua T = tierra L = lancha

30 m
40 m
50 m
O
C D
v
LA
v
LT
v
AT
Para poder aplicar la igualdad (4) necesitamos poner algo as como:
0 = + +
LA TL AT
v v v

(5)
Pero hemos dibujado v

LT
y no v

TL.
Invertimos pues el sentido del
vector al tiempo que intercambiamos los subndices. Y ahora s aplicamos
la relacin (5).


Por semejanza entre el tringulo OCD y el de velocidades
de la derecha, podemos poner:

s
m
v
m
m
AT
4
30
40
=
Luego: v
AT
= 5,33 m/s



v
LA
v
AT
v
LT
v
LA
v
AT
v
TL






















5
Fuerzas

En Cinemtica hemos estudiado diferentes formas de movimiento; pero, cul es
la causa del movimiento? La causa del movimiento es la fuerza, que con su presencia o
ausencia y con su forma de actuar sobre el cuerpo, da lugar a las diferentes formas de
movimiento. Las fuerzas se originan siempre como consecuencia de la interaccin entre
dos masas materiales. Segn que estas masas materiales que interactan se encuentren
distantes una de otra o estn en contacto mutuo, las fuerzas se clasifican en fuerzas a
distancia o fuerzas por contacto.

El propsito de esta clase es el de introducir el concepto de fuerza y presentar a
las fuerzas ms comunes. Como ejemplo de fuerza a distancia, nos referiremos
principalmente a las fuerzas de gravedad, y como ejemplo de fuerzas por contacto, nos
detendremos particularmente en las fuerzas de rozamiento. De las de gravedad slo
haremos una presentacin somera, ya que diferentes aspectos del tema de Gravitacin
sern motivo de una clase futura.

La gravitacin universal.
Desde tiempos muy remotos el hombre trat de conocer y explicar el
movimiento que siguen los astros del cielo, principalmente porque siempre pens que
los mismos ejercan alguna influencia sobre su propia vida, la de los cultivos, de los
animales, el clima, las cosechas, etc. Entonces, poder predecir las posiciones futuras de
los astros en el cielo, fue muy importante.

El astro ms cercano y que ms se impone por su tamao en el cielo es la Luna;
por eso el movimiento de la Luna fue uno de los ms estudiados. Ella gira alrededor de
la Tierra; si realiza un movimiento circular uniforme (MCU), nosotros, que ya hemos
estudiado este movimiento, sabemos que en l, la rapidez es constante, pero que no
obstante ello, hay aceleracin, porque el vector cambia permanentemente de
direccin. Hemos visto que la aceleracin es centrpeta, o sea de la direccin radial
(Tierra-Luna). Ya saba el hombre que donde hay aceleracin hay fuerza, y que por lo
tanto entre la Tierra y la Luna acta una fuerza tambin centrpeta. Pero, lo que fue un
misterio para el hombre durante muchos siglos era quin pona dicha fuerza?

El movimiento de los planetas haba sido estudiado por Kepler alrededor del
1600 y resumido en 3 leyes que hacen una correcta descripcin cinemtica del
problema. Pero faltaba el por qu?; el cules son las fuerzas responsables de dicha
cinemtica planetaria? Con lo cual el misterio continuaba en pie.

Isaac Newton (1642-1727) viene a coronar esta situacin cuando sentenci que
tales fuerzas son el resultado de una accin totalmente general que tiene lugar entre 2
masas prximas. Newton afirmaba esto, inspirado por una observacin casual: es
conocida la historia, no probada, segn la cual Newton se encontraba a la sombra de un
manzano, cuando observ cmo una manzana se desprenda del rbol y caa al suelo.
As es como, en 1686 enuncia la ley conocida con el nombre de ley de la gravitacin
universal, que dice:

Toda partcula de materia del universo atrae a cualquier otra partcula, con una fuerza
cuyo mdulo es directamente proporcional al producto de las masas de ambas
partculas e inversamente proporcional al cuadrado de la distancia que las separa.

Matemticamente:

= G
m
1
m
2

12
2

donde G es una constante de proporcionalidad llamada constante de gravitacin
universal. Ella no debe confundirse con
(aceleracin de la gravedad). La primera es un
escalar, mientras la segunda es una cantidad
vectorial; adems no es constante ni universal.

La figura interpreta en forma grfica la ley
de la gravitacin universal. Ntese que las fuerzas
F (de gravitacin) son dos y no una, consecuencia
de que el fenmeno que involucra a las masas es
mutuo. Las dos F son iguales en mdulo (calculable con la frmula de arriba), tienen la
misma direccin (la de la recta que pasa por los centros de ambas masas) y sentidos de
atraccin (opuestos, si se los compara). De modo que estas fuerzas obedecen en un todo
a un principio que estudiaremos la prxima clase: el principio de accin y reaccin.

El misterio que desvelaba al hombre haba sido resuelto: esa fuerza centrpeta
que acta sobre la Luna era provocada por la Tierra, y era explicada por la ley de la
gravitacin universal.

Dado lo enorme de las masas de la Tierra y de la Luna, dicha fuerza es
gigantesca y es capaz de obligar a la Luna a que realice un MCU en torno de la Tierra.
m
1
m
2

F F
r
1-2
Ntese entonces que la Luna est permanentemente acelerada hacia la Tierra aunque
nunca vaya a caer sobre ella.

Razonamientos similares explican el movimiento de los planetas alrededor del
Sol; las fuerzas gravitacionales rigen tambin el comportamiento de las estrellas y
galaxias. En un entorno humildemente ms cercano, dos objetos cualesquiera de nuestra
mesa de trabajo interactan tambin, bajo la accin de las fuerzas de gravitacin; sin
embargo estas fuerzas mutuas de atraccin resultan tan pequeas en este caso, que no
hay ningn efecto apreciable; aqu las fuerzas de gravitacin son contrarrestadas por las
fuerzas de rozamiento que actan por contacto y que estudiaremos en el prximo tem.

Para terminar, diremos que el valor de la constante G de gravitacin universal,
slo pudo medirse experimentalmente en el ao 1798. Fue Cavendish quien organiz la
experiencia, de la cual result que G = 6,67x10
-11

m
2

2


Esto quiere decir que Newton muri sin haber podido llegar a saber cul era el
valor de la constante de gravitacin que l mismo haba introducido en su frmula.

Las fuerzas de rozamiento.
Las fuerzas de rozamiento se generan entre las superficies de dos cuerpos en
contacto, y segn sean stos slidos ambos, uno slido y otro lquido, lquidos los dos,
etc, las clases de rozamiento que se producen son distintas. En esta ocasin
estudiaremos el caso del rozamiento que tiene lugar entre dos slidos.

En nuestro mundo cotidiano, los rozamientos son inevitables o muy difciles de
eliminar, a pesar de todos los intentos que se han hecho en ese sentido, ya que la
ausencia de rozamientos significara la eliminacin de desgastes, economa de
combustibles, etc. Ya alrededor del ao 1600 Galileo describe en documentos
encontrados, cmo haba logrado reducir considerablemente el rozamiento en los planos
inclinados con los cuales experimentaba, puliendo cuidadosamente las maderas y
tapizndolas con pergamino. Actualmente se logra en el laboratorio la eliminacin casi
completa del rozamiento, manteniendo separadas las superficies con un colchn de aire.

La figura muestra un cuerpo que llamamos
M que se encuentra apoyado en reposo sobre una
mesa horizontal y que est atado a una cuerda que
pasa por la polea y que tiene en su otro extremo
un platillo de peso despreciable, sobre el cual
podemos depositar pesas.

Consideremos y representemos las fuerzas
que actan sobre el cuerpo M. No olvidar que las fuerzas son siempre el resultado de
interacciones entre dos cuerpos, por lo que cada fuerza aplicada a M tiene otra igual y
contraria en otro cuerpo.
M interacta con la Tierra, y como resultado de ello aparece la fuerza peso ()

;
si M es un cuerpo homogneo,

est aplicada en su centro geomtrico, su direccin es


vertical y su sentido es hacia abajo; su igual y contraria se encuentra en la Tierra.
N
F
R

P
F
M
M tambin interacta con la mesa sobre la cual se apoya y recibe de sta a la
fuerza

que se llama reaccin normal; se llama as porque su direccin siempre es


perpendicular a la superficie de la mesa; su igual y contraria se encuentra en la mesa.

es una fuerza de contacto, por lo que se la debe representar con su punto de aplicacin
ubicado en el punto medio del rea compartida entre las dos superficies.

M recibe tambin a travs de la cuerda la fuerza

producida por las pesas que se


coloquen en el platillo; esta fuerza tiene la direccin de la cuerda que la trasmite.
Finalmente, otra interaccin entre M y la mesa da lugar a que aparezca la fuerza
de rozamiento

(observar dnde y cmo se la representa). Su igual y contraria se


encuentra en la mesa.

El dibujo muestra las 4 fuerzas que recibe M; no se representan sus respectivas
iguales y contrarias porque lo que le suceda al cuerpo M, depender nicamente de estas
4 fuerzas. Si M est inicialmente en reposo y estas fuerzas estuvieran equilibradas,
continuara en reposo. De estas 4 fuerzas, dos son verticales y dos son horizontales; nos
convendr analizarlas por separado.

Son verticales

. La experiencia nos dice que el cuerpo M no tiene


movimiento vertical: no se eleva de la mesa ni cae de ella. Ello nos lleva a afirmar
que las fuerzas verticales estn equilibradas; o sea que, en mdulos P = N. Como iremos
viendo poco a poco

cumple en estos casos un papel de comodn tomando el valor


que haga falta para asegurar el equilibrio en su direccin.

Son horizontales

; por la misma razn, si M no se mueve, debern ser F =


F
R
. Es fcil comprender que si la fuerza trasmitida por la cuerda fuera nula, tambin
sera nula F
R
.

Imagine que Ud se encuentra en el laboratorio, tiene el dispositivo de la figura,
dispone de una caja de pesas y se propone efectuar algunas comprobaciones. M est en
reposo sobre la mesa, seal que las fuerzas estn equilibradas. Para alterar ese equilibrio
toma la pesa de 1 gramo y la deposita suavemente en el platillo. Ahora se va a tener que
mover, piensa. Sin embargo M sigue en reposo; teniendo que justificar esto, Ud dir que
al valer F 1 gramo, F
R
tambin pas a valer 1 gramo, por lo que el equilibrio contina.
Entonces hace un segundo ensayo utilizando una pesa mayor, por ejemplo de 5 gramos.
Probablemente tampoco ahora M se mueva, y deber sacar la conclusin de que F
R
pas
a valer 5 gramos y por eso no se mueve. Posiblemente esta situacin se repita varias
veces ms, con pesas cada vez mayores. Esta fuerza de rozamiento que se desarrolla
mientras el cuerpo M permanece inmvil, se llama fuerza de rozamiento esttico, y lo
indicaremos como F
RS
.
.
Sin embargo, encontrar finalmente

una combinacin de pesas
que har que M se ponga en movimiento.

Esto nos ensea que:

La fuerza de rozamiento esttico no posee un valor nico sino que puede asumir
infinitos valores, aunque existe un valor ltimo o mximo por encima del cual el
cuerpo comenzar a moverse.

A este valor ltimo se lo llama fuerza de rozamiento esttico mximo y puede
ser calculado con:
F
Rs mx
=
s
N

s
es el coeficiente de rozamiento esttico; es adimensional y est comprendido entre 0
y 1. Depende de la naturaleza de las superficies en contacto y de su grado de
pulimentado, de la temperatura y otros factores de difcil ponderacin.

Si ahora Ud repite la experiencia anterior, pero introduciendo como variante la
de empujar con un pequeo golpecito al cuerpo M, despus del agregado de cada pesa,
encontrar que M comenzar a moverse cuando las pesas agregadas alcancen un valor
menor al que se requiri en la experiencia anterior. Este valor lo identificaremos como
el de la fuerza de rozamiento cinemtico (F
Rc
) y puede ser calculado con:
F
Rc
=
c
N

c
es el coeficiente de rozamiento cinemtico; es adimensional y su valor est
comprendido entre 0 y 1.

Comparando las dos ltimas expresiones, y sabiendo por la experiencia que:
F
Rc
< F
Rs mx
se deduce que
c
< .

El efecto de esta desigualdad se
puede apreciar con una experiencia muy
sencilla, se sujeta una banda elstica a
un objeto y se lo remolca por la mesa,
tirando en forma pareja. Avanzar a
tirones, an cuando la mano se desplace
uniformemente. Cada vez que el cuerpo
se detiene, para reiniciar el movimiento
es necesaria ms fuerza que la aplicada hasta entonces.

La causa primordial del rozamiento entre dos superficies en contacto, est en las
inevitables irregularidades de las superficies, que provocan encajes locales entre ambas
superficies. Es necesaria una fuerza mayor al iniciar el movimiento porque hay que
vencer los encajes; una vez logrado eso, el movimiento se puede conservar con una
fuerza menor.

Para velocidades bajas,
c
es constante. Por otra parte la fuerza de rozamiento,
slo depende de N y del valor de
.
. es decir que es independiente del tamao de la
superficie de contacto. Por ltimo diremos que la frmula F
R
= N constituye solo una
aproximacin y no muy buena. Las aplicaciones profesionales ms sencillas requieren
tratamientos ms complejos y rigurosos.

Determinacin experimental de los coeficientes
s
y
c
.
Se utiliza un plano inclinado de ngulo variable y sobre l se apoya el bloque
cuyo se desea determinar. Es comn que en los laboratorios, este plano inclinado lleve
adosado un transportador de ngulos, para facilitar la lectura del valor del ngulo que
va teniendo el plano inclinado. En la prctica lo que se hace es comenzar a levantar
lentamente el plano inclinado hasta el momento en que el bloque comienza a deslizar
por el plano inclinado; justo en ese momento, se lee el valor del ngulo y con l, se
hace un clculo sencillo:
= tg
Una pequea diferencia en la ejecucin de la experiencia, permitir conocer
el esttico o el cinemtico. Si se busca el primero, tan solo hay que e sperar que el
deslizamiento del bloque ocurra espontneamente; si se busca el segundo, en cambio,
F
Rs mx

45
Fuerza de
rozamiento
F
Esttico
Cinemtico
habr que dar un pequeo empujoncito al bloque cada vez que se le da un pequeo
incremento a la pendiente del plano, para ver si, una vez iniciado el movimiento, ste
luego se mantiene como un MRU; cuando ello
ocurra, se toma el valor del ngulo . Este ser
ligeramente menor al que se obtuvo en la
experiencia anterior.

Demostraremos a continuacin por qu es
igual al valor de la tangente de .

Los pasos que seguiremos son los pasos que se recomienda seguir siempre, en
problemas de este tipo, porque constituyen un modo ordenado de razonamiento.

1 PASO. Se hace un diagrama de cuerpo
libre (D.C.L.). Consiste en hacer un esquema
del cuerpo que vamos a estudiar, aislndolo
de los restantes cuerpos con los cuales
interacta (figura de la derecha) y
representar todas las fuerzas que actan
sobre l. En nuestro caso son la fuerza peso
P, la reaccin normal N y la fuerza de
rozamiento F
R
. Con respecto a esta ltima,
su sentido ser el de oponerse al sentido de
movimiento del cuerpo, an cuando ste
todava no haya comenzado a moverse. En nuestro caso, F
R
se dirige hacia arriba.

2 PASO. Se adopta un sistema de ejes cartesianos que utilizaremos como referencia. Al
eje x se le da la direccin y el sentido del movimiento del cuerpo; el eje y ser
perpendicular a l.

3 PASO. Se comparan las direcciones de cada una de las fuerzas representadas, con la
de los ejes de referencia. Si hubiera alguna fuerza cuya direccin no fuera la de ninguno
de los ejes cartesianos, entonces se la descompone en x e y. En nuestro caso,
corresponder descomponer a la fuerza peso, que es vertical. Se dibujarn entonces P
x
y
P
y
. Juntas, P
x
y P
y
reemplazarn en lo sucesivo a P. En este pasa se habr logrado que
todas las fuerzas del D.C.L. tengan las direcciones de uno de los ejes cartesianos.

4 PASO. En el tringulo rectngulo formado al descomponer a P, se identifica dnde
qued ubicado el ngulo (ver dibujo). Se escriben las expresiones de las componentes
P
x
y P
y
aplicando trigonometra:

P
x
= Psen P
y
= Pcos

5 PASO. Para cada una de las direcciones cartesianas, se escribe una sumatoria de
fuerzas; si, como es nuestro caso, el cuerpo no tiene movimiento, dicha sumatoria
se iguala a cero. En la sumatoria se escribirn como positivas las fuerzas cuyo
sentido coincida con el del eje de referencia, y como negativas en caso contrario.
Ponemos:
F
y
= 0. N Py = 0 Luego, N = Py (1)
F
x
= 0. Px F
R
= 0 Px = F
R
(2)

N
F
R

Px
Py
P

x
Sistema de
referencia.
D.C.L.
y
6 PASO. Clculos finales. Deseamos obtener la expresin del coeficiente de
rozamiento . De la frmula F
R
= N, despejando:
=

Reemplazando por las (1) y (2): =

j
=
sn
cos

Finalmente: = tg

Equilibrio en el plano inclinado con rozamiento.
Como un modo de afianzar el procedimiento a seguir dado en el tem anterior, se
propone a continuacin el siguiente ejercicio.

Supongamos tener un cuerpo apoyado sobre
un plano inclinado con rozamiento, al que le
aplicamos en forma gradual una fuerza F de la
manera que se muestra en la figura. Al principio, si
F es pequea, no alcanza para impedir que el
cuerpo baje por el plano inclinado, pero llegar un
valor de F (lo llamaremos F
mnimo
) que detendr al
cuerpo y lo dejar en reposo. Si F continuara
aumentando, el cuerpo permanecer de todos
modos en reposo. Pero llegar un valor de F (lo llamaremos F
mximo
) a partir del cual, el
cuerpo abandonar el estado de reposo y comenzar a subir. El cuadro de abajo ilustra
lo expresado:

Pues bien. Se trata de obtener las expresiones que permitan calcular F
mn
y F
mx

en funcin de los siguientes datos:
1
,
2
, y P.

Deduciremos a continuacin la expresin de F
mn
y dejaremos la de F
mx
para que
practique el alumno. Cul es el sentido de F
R
en el momento en que se aplica el F
mn
?
Observando el cuadro de arriba, para F
mn
el cuerpo est en el lmite entre el reposo y
bajar; el movimiento del cuerpo, ocurra o no todava, ser hacia abajo; entonces la F
R

deber tener sentido hacia arriba. Hacemos el DCL, adoptamos un sistema de referencia
y descomponemos las fuerzas que hagan falta para que finalmente todas las fuerzas del
esquema estn en la direccin de x o de y.
Fy = 0] N + Fy Py = 0 (1)

2

F
BAJA REPOSO SUBE
0
F
mn
F
mx

F
N
P
Px
Py
F
R

F
Fy
Fx
x
y

2

Sistema de referencia
Fx = 0] Px Fx F
R
= 0 (2)

La (1) me permite obtener la expresin de N. N = Py Fy (3)
La (2) me permite obtener la expresin de F
R
F
R
= Px Fx (4)

Dado que F
R
=
s
N reemp. por las (3) y (4): Px Fx =
s
(Py Fy)
Ordenando:
s
Fy Fx =
s
Py Px (5)
Aplicando trigonometra:
Px = Psen
1
Fx = Fcos
2

Py = Pcos
1
Fy = Fsen
2


Reemp. En la (5):
s
Fsen
2
- Fcos
2
=
s
Pcos
1
- Psen
1

Finalmente:
Fmn =
(

os
1
sn
1
)

sn
2
cos
2










3
Dinmica lineal
En las clases pasadas hemos estudiado cinemtica, una parte de la Fsica que
trata los distintos movimientos, y donde hemos aprendido el lenguaje que los describe.
Pero la cinemtica no va ms all; ella no se ocupa de cules son las causas del
movimiento. Por eso, para respondernos a preguntas tales como: cmo puede un
remolcador empujar un transatlntico que es mucho ms pesado que l? o por qu es
ms difcil controlar un automvil en hielo mojado que en concreto seco?, debemos
recurrir a otra parte de la Fsica, llamada dinmica.
La dinmica estudia la relacin entre los movimientos y las fuerzas que los causan.

Newton coron todo el conocimiento que el hombre haba acumulado a lo largo
de varios siglos, sobre esta cuestin y los public por primera vez en el ao 1687
reunidos en tres enunciados concisos, a los que hoy llamamos los principios de la
dinmica o las leyes del movimiento de Newton. Cada uno de ellos tiene un nombre y se
ocupa de una cuestin diferente; ellos son:
1- Principio de inercia.
2- Principio de masa.
3- Principio de accin y reaccin.
Trataremos brevemente cada uno de ellos.
PRINCIPIO DE INERCIA
Este principio se ocupa del caso en que sobre una partcula acte un sistema
equilibrado de fuerzas y qu movimientos son compatibles con esta situacin.
Consideremos los siguientes dos ejemplos: a) Si se coloca sobre una mesa un
cartn y sobre ella una lapicera, si se retira rpidamente el cartn, se comprobar que la
lapicera permanece en la misma posicin respecto de la mesa. b) Cuando estamos
parados en el pasillo de un colectivo, si ste arranca bruscamente, somos impelidos
hacia atrs, tratando de mantener el lugar que ocupbamos con respecto del pavimento.
Ambos ejemplos nos muestran que los cuerpos en reposo tienden a mantenerse
en reposo.
Consideremos a continuacin estos otros dos ejemplos: c) Si lanzamos a rodar
una esfera por una superficie horizontal, comprobamos que, una vez lanzada, el
movimiento se mantiene, sin necesidad de continuar empujndola; esto es ms evidente
cuanto ms pulidas estn las superficies (rozamiento menor); y si pudiramos eliminar
por completo el rozamiento el movimiento de la esfera sera perpetuo. d) Cuando
estamos parados en el pasillo de un colectivo en movimiento, si ste frena bruscamente,
seremos impelidos hacia adelante, como si tratramos de conservar el movimiento que
traamos.
Estos otros ejemplos nos muestran que los cuerpos en movimiento tienden a
conservar su movimiento, bajo la forma de un MRU.
Lo que tienen en comn los cuatro ejemplos anteriores es que en ninguno de
ellos acta una fuerza neta. Las fuerzas presentes estn equilibradas.
Una partcula permanecer en reposo o se mover con MRU, si sobre ella acte un
sistema equilibrado de fuerzas.

-Explicar por qu se habla de un sistema equilibrado de fuerzas y no se dice en
ausencia de fuerzas, que matemticamente sera lo mismo.
- Agregar como ejemplo ultimo el caso del cuerpo atado mediante una cuerda al techo,
y del que cuelga otra cuerda idntica; si se tira de esta ltima suavemente en forma
creciente, se rompe primero la cuerda de arriba; si en cambio se tira bruscamente, la
cuerda que se rompe primero es la de abajo.
PRINCIPIO DE MASA.
Este principio se ocupa del caso en que sobre una partcula acta un sistema de
fuerzas no equilibrado. En este caso este conjunto de fuerzas es equivalente a una fuerza
neta. La experiencia muestra que si esta fuerza neta es constante, la partcula adquiere
un MRUV con una aceleracin de la misma direccin y sentido que la de la fuerza neta.
-Para inducir la relacin entre F
neta
y a plantear la experiencia de aplicar sobre un
misma carrito, sucesivamente fuerzas diferentes, F
1
F
2
, F
3
medidas, y medir en cada
caso la aceleracin adquirida por el cuerpo: a
1
, a
2
, a
3
.
Se comprueba que: constante.
A esta constante se le da el nombre de masa (m). Luego: = m

sta es la expresin matemtica de la segunda ley de Newton; ntese que ella est
planteada para la fuerza neta, no para una fuerza cualquiera. Tambin observar que
y son vectores de la misma direccin y sentido, por lo que el escalar m slo
puede tener signo positivo.
La masa es una propiedad intrnseca del cuerpo, que establece la relacin que
hay entre la fuerza neta aplicada y la aceleracin que el cuerpo adquiere; el valor de esta
relacin es independiente del lugar donde se efecte la experiencia, por lo que m es
universal. Marcar la diferencia con el peso del cuerpo, que no es ninguna propiedad
intrnseca del mismo.
Bajo la accin de la fuerza neta, toda partcula adquiere una aceleracin en la misma
direccin y sentido, y cuya intensidad es proporcional a la de la fuerza. La constante
de proporcionalidad se llama MASA.

Explicar cmo se calcula una masa. y dar su unidad.
PRINCIPIO DE ACCIN Y REACCIN
Una fuerza que acta sobre un cuerpo es siempre el resultado de una interaccin
con otro cuerpo, de modo que las fuerzas siempre vienen de a pares. Es imposible
generar una fuerza solitaria. A una de las fuerzas que componen este par, se la llama
accin y a la otra reaccin.
Este principio establece que:
1- Las fuerzas siempre vienen de a pares; que no hay accin sin reaccin.
2- Accin y reaccin actan sobre una misma recta de accin, por lo que tienen la
misma direccin, sentidos opuestos e igual mdulo.
3- Accin y reaccin actan sobre cuerpos diferentes.


Sistema Internacional de Unidades. (S.I.)
El Sistema Internacional (S.I.) es el sistema de unidades empleado por los
cientficos e ingenieros de todo el mundo. Fue adoptado como tal en 1960. Las unidades
del .I. fueron seleccionadas por la Oficina Internacional de Pesas y Medidas, entidad
establecida en 1875 en Svres, Pars. Actualmente componen el Estandar Internacional
Norma ISO 80.000, que reconoce las cantidades fsicas, unidades de medida y frmulas
que los implican, del S.I.
Cuando se crea un sistema de unidades, lo que se Hce es seleccionar de entre
todas las cantidades fsicas un nmero reducido de ellas, a las que se llamar unidades
de base, y todas las dems se obtienen a partir de ellas; son las unidades derivadas.
Las unidades de base del S.I. son 7 y se corresponden con las siguientes
cantidades fsicas: Longitud (metro); Masa (kilogramo); Tiempo (segundo); Corriente
elctrica (Ampre); Temperatura (Kelvin); Cantidad de sustancia (mol); e Intensidad
luminosa (candela).
La parte de la Fsica que nosotros estudiamos, la Mecnica, slo utiliza como
unidades de base a las tres primeras.
UNIDADES DEL S.I. PARA LA MECNICA
UNIDADES CANTIDAD NOMBRE SMBOLO

DE BASE
Longitud Metro m
Masa Kilogramo kg
Tiempo segundo s

DERIVADAS
Velocidad m/s
Aceleracin m/s
2

Fuerza Newton N

Y le siguen otras unidades derivadas que iremos estudiando a lo largo del ao. Pueden
emplearse mltiplos y submltiplos de estas cantidades, los que se nombran agregando
un prefijo al nombre de la unidad. Los ms usados son:
FACTOR PREFIJO SMBOLO
10
3
kilo K
10
6
mega M
10
9
giga G
10
12
tera T
10
-3
mili m
10
-6
micro
10
-9
nano n
10
-12
pico p

REGLAS
1- Los smbolos de las unidades, con raras excepciones como en el caso del ohm (), se
expresan en caracteres latinos, en general, con minsculas; sin embargo, si dichos
smbolos corresponden a unidades provenientes de nombres propios, su letra inicial es
mayscula. Ejemplos: N de Newton; Pa de Pascal; J de Joule, W de Watt; Hz de Hertz.
2- Los smbolos no van seguidos de punto, ni toman la s para el plural. Por ejemplo: se
escribe 5 kg; no 5 kgs
3- Los nombres de las unidades provenientes de nombres propios de cientficos deben
escribirse con minscula inicial y con idntica ortografa que el nombre de stos. No
obstante se aceptan tambin sus denominaciones castellanizadas de uso habitual siempre
que estn reconocidas por la Real Academia de la Lengua. Por ejemplo: amperio, voltio,
faradio, julio, ohmio, watio.
4- Los nombres de las unidades toman una s en el plural (ej: 10 newtons) excepto las
que terminan en s, x z (lux; hertz)
EXPRESIN DEL NEWTON
De 1N=kg.m/s
2

Problemas:
1- En la figura, entre el cuerpo (1) y la
superficie el
c
= 0,2. Calcular a) la
aceleracin del sistema; b) la tensin
en la cuerda.
Solucin:
Trazado de los DCL:
T F
Rc
= m
1
a
P
2
T = m
2
a
Sistema de 2 ecuaciones con 2 inc: a y T.
Resolviendo, a = 0,4 m/s
2
y T = 96 N

2- Para la figura de la pgina siguiente, hallar las aceleraciones del bloque y de la losa.
(1)
(2)
P

= 400N
P
2
= 100N
a
N
F
Rc
P
1

T
T
P
2

a
DCL 1 DCL 2
Solucin:
Diagrama de cuerpo libre del bloque:
Determinaremos primero si hay movimiento
relativo entre el bloque y la losa.
F
Rs mx
=
s
N
1
= 0,6100 N = 60 N
Como F
Rs mx
< F, el bloque se mueve y F
R
=
c
N
1
= 40 N
a
B
=
Diagrama de cuerpo libre de la losa:
a
L
=



3- Hallar el mdulo de F para que el bloque (1) no caiga,
y la aceleracin del conjunto.
Solucin:
DCL (1): Condicin
para que (1) no caiga:
F
Rs mx
= P
1
Luego:
s
N
1
= P
1
N
1
=

-Clculo de F y de a.
Para obtener ambas incgnitas hay que plantear un
sistema de 2 ecuaciones. La 1 proviene del DCL (1): F N
1
= m
1
a
a = (1)
F = 100N
BLOQUE
m
B
= 10 kg

LOSA
m
L
= 40 kg
= 0 s = 0,6;
N
1
= 100N
F
R

F = 100N
P
B
= 100 N
F
R
= 40 N
N
2

N
1
P
L

= 0
s = 0,8
(2)
100kg
F = ?
(1)
m
1
= 10 kg
F
P
1
= 100 N
N
1

F
Rs mx

La 2 puede obtenerse a partir del DCL (2) o bien de considerar el conjunto de
cuerpos. Si se lo hace de la primera manera:
N
1
= m
2
a a = (2)
Resolviendo el sistema de ecuaciones (1) y (2), se obtiene:
a = 1,25 m/s
2
F = 137,5 N

Si hubiramos considerado al conjunto, la ecuacin hubiera
sido:
F = (m
1
+ m
2
)a (3)
Y el sistema de ecuaciones se hubiera formado con la (1) y la (3).


F
Rs mx
N
1

P
2

N
2





















7
Trabajo

Si se nos descompone el automvil y debemos empujarlo, todos estamos de
acuerdo en decir que eso nos va a costar trabajo. Esta interpretacin cotidiana del
trabajo coincide con su significado fsico. El trabajo involucra dos cosas: fuerzas y
desplazamientos. Si para empujar al automvil aplicamos una fuerza mayor, haremos
ms trabajo; y tambin haremos ms trabajo si el desplazamiento fuera mayor. Podemos
entonces decir que para que haya trabajo, se necesitan dos cosas: tener una fuerza y que
ella traslade su punto de aplicacin.

Si enrollamos una cuerda sobre un eje o polea, y su otro
extremo lo atamos a una pesa, al soltarla ella comenzar a caer; la
fuerza peso P realizar trabajo al desplazar su punto de aplicacin.
Ese trabajo se manifiesta en que el eje comenzar a girar cada vez
ms rpido. Tener un eje en rotacin, igual que un motor, podr ser
aprovechado por nosotros para hacer funcionar por un ratito, a un
juguete por ejemplo. As, el trabajo se nos manifiesta como energa
disponible.

Tener una fuerza y que su punto de aplicacin se desplace, es necesario para que
haya trabajo, pero no es suficiente. Si la fuerza y el desplazamiento tienen direcciones
distintas, ser necesario que la fuerza proyecte alguna componente en la direccin del
desplazamiento.
Representaremos al trabajo, con el smbolo W.
P
La figura muestra una fuerza F

que forma un ngulo con la direccin del


vector desplazamiento

. La proyeccin de F sobre l vale Fcos y el trabajo es:



W = Fl cos (1)

Esta expresin no es otra cosa que el desarrollo
escalar del producto escalar entre dos vectores:

W = F

.

El trabajo (W) es una cantidad escalar; su unidad en el S.I. es Nm, expresin
que recibe el nombre de joule (J).
1 J = 1 N m

La ecuacin (1) tiene limitaciones; slo es aplicable si se cumplen dos
condiciones: una que afecta a la fuerza y otra que afecta al desplazamiento. La primera
es que la fuerza debe mantenerse constante durante la ejecucin del trabajo, ya que de
variar, la frmula no permite tomar en cuenta tales variaciones. La segunda es que el
desplazamiento debe ser rectilneo, ya que de no ser as, variara, y la frmula no
puede tomar en cuenta esas variaciones.

Buscaremos mejorar la frmula (1) para darle generalidad a la expresin del
trabajo, y para eso recurriremos a las tcnicas del clculo infinitesimal. Supongamos
una trayectoria curva que se extiende desde el punto A hasta el punto B, y que es
recorrida por el punto de aplicacin de una fuerza F que va variando de mdulo y de
direccin.

Dividiremos la trayectoria en una gran cantidad de pequeos tramos
consecutivos l
i
como para suponer que cada uno de ellos es rectilneo y que la fuerza F


permanece constante, por lo que el trabajo elemental en cada tramo puede expresarse
con la primitiva frmula:
W
i
= F

i

Y el trabajo total entre los puntos A y B ser:
W
AB
= lim
0
F

= F

B
A



Potencia mecnica (P).
En la expresin del trabajo no interviene el tiempo; sin embargo, para muchas
cuestiones prcticas resulta importante poder confrontar el trabajo a realizar con el
tiempo en que se desea que sea ejecutado. Por eso se define la potencia mecnica como
la razn entre el trabajo realizado y el tiempo empleado:

P =

t

La unidad de potencia en el S.I. es el watt (W), de manera que 1 W =
1u
s

Tambin se utilizan todava algunas unidades antiguas de potencia, tales como el
Caballo de potencia (hp) que equivale a 746 W y el Caballo vapor (CV) que equivale a
736 W)

F
l

Fcos
O
Otras unidades de trabajo.
Si en la frmula de la potencia se despeja trabajo, W = Pt, surgen nuevas
unidades de trabajo:
1 KWh = 1000 W1 h = 1000

S
3600 s = 3,6x10
6
J
1 hph = 746

S
3600 s = 2.685.600 J
1 cvH = 736

S
3600 s = 2.649.600 J

Trabajo de una fuerza conservativa.
Existe un reducido y selecto grupo de fuerzas que, cuando realizan trabajo,
ste goza de algunas propiedades muy particulares. La calidad de su trabajo, marca su
diferencia con las dems fuerzas. A estas fuerzas se las llama conservativas, y una de
ellas es la fuerza peso. Cules son las propiedades particulares que poseen estas fuerzas
es lo que trataremos de descubrir a continuacin.

Como ya hemos anticipado, la fuerza peso es una fuerza conservativa; la
tomaremos para desarrollar el siguiente ejemplo. El rectngulo de la figura representa
una trayectoria cerrada que tiene lugar en un plano vertical y que es recorrido por un
punto material. Supongamos que este punto material parte del punto A y recorre todo la
trayectoria rectangular, pasando por B, C, D, para llegar finalmente otra vez al punto A.
Mientras realiza este recorrido, su fuerza peso (P) se va trasladando y haciendo trabajo.
Cunto vale el trabajo total efectuado por la fuerza P, cuando la partcula ha dado la
vuelta completa?

Lo podemos calcular sumando los 4
trabajos parciales hechos sobre cada lado del
rectngulo. En el dibujo se muestra en cada lado
a la partcula, su peso P

su desplazamiento

y
el valor del ngulo comprendido entre ellos.
Y llamamos h a la altura del rectngulo.
Calculemos:

Lado AB. W
AB
= Phcos 0 = + Ph
Lado BC W
BC
= Phcos 90 = 0
Lado CD W
CD
= Phcos 180 = - Ph
Lado DA W
DA
= Phcos 90 = 0

Sumando, Wj = 0

Esta conclusin es general a cualquier camino cerrado y puede expresarse as:

P

= 0
y enunciarse as:

El trabajo de la fuerza peso a lo largo de una curva cerrada cualquiera, es nulo.

Esto se cumple con cualquier fuerza conservativa F, de modo que,
generalizando:
90
0 180
90
P
P
P
P
l
l
l
l
A
B
C
D
h
F

= 0
Desde el punto de vista del trabajo, las fuerzas pueden clasificarse en conservativas y no
conservativas.

SON CONSERVATIVAS: La fuerza peso; las fuerzas elsticas y las fuerzas
electrostticas.
SON NO CONSERVATIVAS: Las fuerzas de rozamiento, las tensiones en
cuerdas, la reaccin normal, las fuerzas internas.

Propiedades de las fuerzas conservativas.
1- LOS TRABAJOS DE IDA Y REGRESO SOBRE EL MISMO CAMINO.
Sea el camino inversible ABA:

F

ABA
= 0

F

+ F

= 0
BA AB


F

= F

BA AB



El trabajo de una fuerza conservativa a lo largo de una curva AB es igual y de signo
opuesto al trabajo de esa fuerza desde B hasta A, sobre BA.

2- INDEPENDENCIA DEL TRABAJO CON EL CAMINO SEGUIDO.
Sean dos caminos distintos, con los mismos extremos A y B.

F

= F

+ F

= 0
BA AB


F

= F

BA AB


Aplicando la primera propiedad:

F

= F

AB AB
(1)

El trabajo de una fuerza conservativa entre dos puntos, es independiente del
camino seguido; slo depende de los puntos inicial y final.

3- EL TRABAJO COMO DIFERENCIA ENTRE DOS FUNCIONES DE ESTADO.

Calcularemos el trabajo de una fuerza conservativa entre dos puntos A y B, en
funcin de un tercer punto O, tomado como referencia:

Tomando como punto de partida la expresin (1):
A
B
A
O
B
F

= F

AB AB


= F

+ F

B A


Aplicando la primera propiedad:

= F

B A


Aplicando ahora la segunda propiedad, segn la cual el valor de cada una de
estas integrales slo depende de los estados inicial y final:

F

= (A; 0)
AB
(B; 0)

F

= (A) (B)
AB
(2)

El trabajo que realiza una fuerza conservativa entre los puntos A y B es igual a la
diferencia entre los valores que adquiere una funcin escalar f en esos dos puntos.

Esta funcin que en forma genrica hemos nombrado con f, en la mecnica se la
representa con la notacin Ep y se la denomina energa potencial. La energa potencial
es considerada matemticamente como una funcin de estado, por el hecho de que su
valor depende pura y exclusivamente del punto que se est considerando.

La (2) se escribe entonces as: F

= E
A
E
B
B
A
(3)

Si utilizamos el operador para indicar la variaci n, recordando que la
diferencia debe escribirse siempre como final menos inicial: Ep = Ep
B
- Ep
A
la (3)
deber ser escrita as:
Ep = Ep
B
- Ep
A
= - F

B
A
(4)

La palabra potencial significa posicin, y la energa potencial puede ser de dos
clases: gravitatoria o elstica; para distinguirlas usaremos como notacin Ep
g
y Ep
e

respectivamente. La primera es generada por la fuerza peso y su valor depende de las
posiciones relativas entre dos cuerpos que interactan; la segunda es debida a la fuerza
que desarrolla un cuerpo elstico (resorte) cuando su extremo es apartado de su posicin
de equilibrio. Consideraremos ahora la primera de estas dos formas mencionadas.

Energa potencial gravitatoria.
En la deduccin de la tercera propiedad del trabajo realizado por las fuerzas
conservativas, se ha recurrido a un punto O de referencia. Para poder aplicar el concepto
de energa potencial tambin necesitamos de una referencia, y ella ser un nivel. La
fuerza peso que posee cualquier objeto, es producida por la Tierra al interactuar con l.
Para nuestras aplicaciones cotidianas consideraremos que el peso de las cosas se
mantiene constante, lo cual slo es cierto en las cercanas de la Tierra.

Entonces, llamando h
A
a la altura del punto A respecto del nivel de referencia y
h
B
a la altura del punto B respecto del nivel de referencia, y considerando a P constante,
podemos poner:
Ep
A
= Ph
A
= mgh
A


Ep
B
= Ph
B
= mgh
B

y el trabajo de la fuerza peso al trasladarse desde A hasta B ser:
Wp
AB
= Ep
A
Ep
B
= mg(h
A
h
B
)

Ejemplo:
El cuerpo de la figura pesa 5
N. Calcular el trabajo realizado por
dicha fuerza cuando se traslada desde
el punto (1) hasta el punto (2)
Solucin:
Como la fuerza peso es
conservativa, no se necesita conocer
el camino seguido para ir desde (1) hasta (2).

Wp
AB
= Ep
A
Ep
B
= P(h
A
h
B
) = 5 N (10 2)m = 40 J

Fuerzas conservativas y disipativas.
No olvidar que la funcin energa potencial expresada por la (4) slo se cumple
para las fuerzas conservativas. Para que una fuerza sea conservativa, una de dos: o ella
es constante (como es el caso del peso en las cercanas de la superficie terrestre) o bien
es funcin exclusiva de la posicin (como ocurre con la fuerza elstica de un resorte).
Cuando la fuerza depende de alguna otra magnitud diferente a la posicin, ella ser
invariablemente una fuerza disipativa.

Veamos un ejemplo. Supongamos que
arrastramos por un plano horizontal un
bloque, que ofrece al movimiento una
fuerza de rozamiento de mdulo constante.
a) Si la llevamos desde A hasta B, el trabajo realizado por ella W
Fr
es negativo,
porque los sentidos de la fuerza y del desplazamiento son opuestos; no hay manera de
recuperar ese trabajo.
b) Si la llevamos desde A hasta C y luego retrocedemos hasta B, el W
Fr
vale el
triple del calculado en a). Al llegar al punto C de este recorrido, la F
R
cambia su sentido;
por lo tanto F
R


no es constante.

Otro ejemplo de fuerza no conservativa es la ejercida por una persona, cuando
empuja o tira. As, cuando se quiere hacer subir verticalmente un cuerpo pesado hasta
una altura h, si se emplea una polea, aplicamos en la cuerda una fuerza vertical T; si T =
P el cuerpo no se acelera. Pero tambin podemos aplicar una tensin tal que T > P y el
cuerpo subir aceleradamente. Es decir que el valor de la fuerza T aplicada no depende
exclusivamente de la posicin y es por lo tanto no conservativa.

nivel
h
1
= 10 m
h
2
= 2 m
1
2
P
P
A B C
0 5 10
x (m)
Resumiendo, para que una fuerza sea conservativa se necesita que ella sea
funcin exclusiva de la posicin; siendo as, su trabajo resultar independiente de la
trayectoria seguida; su valor depender de las posiciones inicial y final. Sin embargo el
ao prximo en Fsica II se ver que ste es un requisito necesario pero no suficiente.

Trabajo y energa.
Hemos comenzado esta clase hablando de trabajo, pero poco a poco, a medida
que avanzamos la palabra trabajo fue usndose menos, dando lugar a una nueva palabra:
energa. Cul es la diferencia de significado entre ambos trminos?

Plantearemos una analoga, trayendo un ejemplo de la economa. Si hemos
cobrado una cierta suma de dinero, podemos hacer con ella dos cosas: salir de shoping a
gastarla adquiriendo bienes, o abrir una caja de seguridad y guardar el dinero en el
banco. El anlogo a usar, gastar, pagar, es trabajo; el anlogo a guardar, almacenar, es
energa. Diremos entonces que:

ENERGA es la capacidad para ejecutar TRABAJO que posee un sistema de cuerpos.

Trabajo es aquella magnitud por la cual habremos de pagar de algn modo. El
trabajo nos habla de ejecucin, mientras que la energa nos habla de una reserva en
espera. Pero en el fondo se trata de una misma cosa como lo patentiza el hecho de que
tanto trabajo como energa se miden con la misma unidad: el joule.

Podemos dar otra analoga con el caso de cuando uno compra una pila en un
kiosco. Mientras esa pila no sea desembalada y permanezca guardada en un cajn,
diremos que tiene energa. Recin cuando le conectemos un par de cables y hagamos
funcionar con ella a un juguete por ejemplo, emplearemos la palabra trabajo

Las formas de almacenar trabajo (o formas de energa) de la naturaleza son
muchas y muy diversas: elctrica, qumica, magntica, elica, nuclear, trmica, etc. En
este curso slo nos ocuparemos de una de estas formas: la llamada energa mecnica.

No est de ms recordar que la energa puede convertirse de una forma a otra,
pero no puede crearse ni destruirse, por lo que la energa total del universo permanece
constante a travs de los tiempos.

Con relacin a la energa mecnica (Em), ella puede presentarse en tres formas:
Energa potencial gravitatoria (Epg): que depende de la posicin relativa entre
dos masas.
Energa cintica (Ec): que depende del movimiento relativo.
Energa potencial elstica (Epe): que depende de la posicin relativa del
extremo de un resorte o muelle.




























5
energa mecnica

En el mdulo anterior hemos estudiado el concepto de trabajo mecnico, como aquel
que es realizado por una fuerza cuando traslada su punto de aplicacin. Pero si sobre nuestra
partcula se encontraran aplicadas muchas fuerzas, cunto valdr el trabajo total realizado por
todas ellas? De esto nos ocuparemos a continuacin. Se nos presentan dos alternativas segn
que el sistema de fuerzas aplicado a la partcula est equilibrado o no.

Veamos el primer caso; la figura muestra una partcula sobre la cual acta un sistema
equilibrado de 4 fuerzas; rige el principio de inercia y la partcula se desplaza con MRU, o sea
con v = cte:








Estado inicial Estado final
F
i
= 0 (sist, de fzs. equilibrado) W
Fi
= 0
(suma vectorial) (suma escalar)
Al calcular los trabajos realizados por cada fuerza, la suma de todos ellos es siempre
nula. Veamos ahora el segundo caso, cuando el sistema de fuerzas no se encuentra equilibrado.
En tal caso la F
i
no es nula y a su resultado denominaremos fuerza neta. Nos preguntamos
nuevamente cunto vale el trabajo total realizado por todas las fuerzas en este caso.
Obviamente podramos volver a aplicar el procedimiento anterior de calcular el valor del
F
1

F
2

F
3

F
4

F
1

F
2

F
3

F
4

trabajo realizado por cada una de las fuerzas y finalmente sumar todos los trabajos. Pero esto
podra en algunos casos resultar sumamente laborioso o tedioso, por lo que vamos a intentar
encontrar otro camino que, tal como un atajo nos permita obtener lo que buscamos de un
modo ms sencillo.

Grficamente el planteo de este segundo caso sera:








Estado inicial Estado final
F
i
0 O bien: F
i
= F
neta
W
Fi
= ?

Al haber una fuerza neta aplicada a la partcula, rige el principio de masa, que establece
que en su desplazamiento, la partcula realizar un MRUV con a = cte. La pregunta ahora es:
cunto vale la W
Fi
. Justamente de esto se ocupa el teorema del trabajo y la energa cintica.

Teorema del trabajo y la energa cintica.
Para razonar la deduccin de este teorema, utilizaremos la figura anterior, slo que en
vez de representar a todas las fuerzas obrantes sobre la partcula, nicamente representaremos a
la fuerza neta, y agregaremos un sistema de referencia en la misma direccin del
desplazamiento:

De la cinemtica y del MRUV, recogemos la expresin: x a v v = 2
2
1
2
2
de donde:

x
v v
a

=
2
2
1
2
2

Por el principio de masa: F
neta
= ma
Mult. por x ambos miembros: F
neta
x = max
Reemplazando: W
Fneta
= m
2
1
2
2
2
1
2
2
2
1
2
1

2
mv mv x
x
v v
=


Finalmente: W
Fneta
= Ec
2
Ec
1
= Ec (1)

ENUNCIADO: El trabajo efectuado por la fuerza neta sobre una partcula es igual a la
variacin de la energa cintica de la partcula.

F
neta
F
neta

m m
v
1

v
2

a = cte
1 2
X
Eje de referencia
x
1
x
2

F
1

F
2

F
3

F
4

F
1

F
2

F
3

F
4

Sobre el teorema del trabajo y la energa cintica, cabe destacar las siguientes tres
circunstancias:

1- Obsrvese que este teorema nos dice que no importa qu tipo de fuerzas acten sobre la
partcula, el trabajo de todas ellas siempre ser igual a la variacin de la energa cintica.

2- Dado que para deducir este teorema hemos utilizado las leyes de Newton, este teorema
recibe por herencia las limitaciones que tienen las leyes de Newton: slo puede usarse en un
marco de referencia inercial. Mientras cualquiera sea dicho marco inercial este teorema
siempre sers vlido, an cuando de un marco a otro puedan diferir los valores de x, v
1
y v
2
y
por ende, los de W
Fneta
, Ec
1
y Ec
2
.

3- Por ltimo diremos que este teorema ha sido dado para una partcula y que su uso en
sistemas materiales ms complejos obliga a consideraciones ms sutiles que no estudiamos.

La energa cintica.
La expresin que aparece en la deduccin del teorema del trabajo y la energa: mv
2

recibe el nombre de energa cintica que indicamos con Ec.

La energa cintica es una forma que tiene un sistema de almacenar energa, en este
caso, en razn de la rapidez que tienen las partculas que lo forman. Al igual que el trabajo, es
una cantidad escalar, pero a diferencia de ste, ella nunca puede ser negativa y nicamente vale
cero en el caso en que la partcula se encuentre en reposo. Trabajo y energa cintica se
expresan con la misma unidad; en el MKS, el Joule.

Corrigiendo errores frecuentes: es incorrecto decir que la energa cintica es igual a un
medio del producto entre la masa y el cuadrado de la velocidad. La energa cintica nada tiene
que ver con la velocidad ya que de ella slo toma su mdulo pero no su direccin ni su sentido.
Dicho en otras palabrea: un automvil que va a 80 km/h tiene la misma energa cintica, no
importa si se dirige hacia el norte, el sur, el este o el oeste. Al mdulo de la velocidad se lo
denomina rapidez. S es correcto decir que la energa cintica es proporcional al cuadrado de la
rapidez.

Problemas con aplicacin del teorema del trabajo y la
energa cintica.

1- Una caja de 5 kg originalmente en reposo, es elevada hasta una altura de 4 metros por una
fuerza vertical de 80 N. Determinar: a) El trabajo realizado por la fuerza aplicada; b) el trabajo
realizado por la fuerza peso; c) la velocidad final de la caja.
Solucin:
a) W
Fext
= F
ext
h cos 0 = 80 N 4 m 1 = +320 Joule
b) W
P
= P h cos 180 = 50 N 4 m (-1) = -200 Joule
c) WF
neta
= Ec
2
Ec
1
donde Ec
1
= 0. Luego:
W
Fneta
= Ec
2

30 N4 m(+1) = (5 kg)v
2
2
v
2
= 7 m/s




m = 5kg
a
P = 50 N
F
ext
= 80 N
h = 4 m
1
2
2- Se aplica una fuerza horizontal de 25 N a una caja de 4 kg inicialmente en reposo sobre una
mesa horizontal rugosa (
c
= 0,35). Determinar la velocidad de la caja despus de haber sido
empujada una distancia de 3 m.
Solucin:
N = mg = 39,2 N
F
Rc
= N = 0,35 39,2 N = 13,72 N
La fuerza resultante es:
F
neta
= F F
Rc
= (25 13,72) N = 11,28 N
WF
neta
= Ec
2
Ec
1
, donde Ec
1
= 0. Luego:
11,28 N3 m = (4 kg)v
2
2
v
2
= 4,11 m/s

3- Demostrar aplicando el teorema del trabajo y la energa cintica, que en el movimiento
circular uniforme, la fuerza centrpeta no realiza trabajo sobre la partcula.
Solucin:
Todos sabemos ya que en el MCU, la fuerza centrpeta, que tiene la direccin del radio
de la trayectoria no realiza trabajo, porque esta direccin y la del desplazamiento son en todo
momento perpendiculares entre s. Pero ahora probaremos esto mismo por otro camino.

La fuerza centrpeta es la fuerza neta aplicada a la partcula; todas las dems estn
equilibradas entre s. El teorema del trabajo y la energa cintica establece que:
W
Fcentrpeta
= Ec
2
Ec
1

Como en el MCU es = v

cte, es v
2
= v
1
, con lo que Ec
2
= Ec
1
. Por lo tanto es:
W
Fcentrpeta
= 0

La fuerza centrpeta
En lo que va de este curso, hemos conocido varias fuerzas, tales como el peso, la
reaccin normal, la fuerza de rozamiento y la tensin en cuerdas. Al estudiar el movimiento
circular uniforme, apareci la fuerza centrpeta, como una fuerza de mdulo constante,
direccin radial y sentido hacia el centro de la trayectoria. Estamos tentados de pensar que la
fuerza centrpeta es entonces otra fuerza ms, que deber agregarse a la lista anterior. Sin
embargo no es as; la fuerza centrpeta no es una fuerza en s misma; es tan solo un nombre que
se le asigna a cualquier fuerza (o conjunto de fuerzas) que rena los requisitos citados en
cuanto a mdulo, direccin y sentido. As por ejemplo, en el caso de la Luna girando alrededor
de la Tierra, la Fc sobre la Luna, es la fuerza de gravedad; en el caso de un objeto atado con
una cuerda, que se hace girar sobre una superficie horizontal (ver figura), la Fc sobre el objeto
es la tensin T de la cuerda.

El nombre de fuerza centrpeta no es afortunado, puesto
que parece implicar que esta fuerza es de alguna manera diferente
de las dems fuerzas ordinarias, o que el MCU genera de algn
modo una fuerza adicional; nada de esto es correcto. El nombre
de centrpeta se refiere a sus caractersticas de que su direccin est siempre dirigida al centro
de la trayectoria y que su mdulo permanece constante.

Podemos ser ms precisos en nuestro intento de explicar a la Fc. Obsrvese que tanto en
los ejemplos de la Luna como en el del objeto que gira, atado a una cuerda, la Fc es
directamente igual a la F
i
fuerzas aplicadas a la partcula que gira. Y esto es totalmente
general.

m = 4 kg
N
P
F
Rc

F = 25 N
N
P
T
Por lo tanto, la Fc no es una fuerza en s misma; tan solo es la resultante de la totalidad
de las fuerzas aplicadas. Por esta razn la Fc no debe incluirse en un diagrama de cuerpo libre.

El MCU es el resultado de cualquier combinacin de fuerzas que produzca una F
i
de mdulo
constante dirigida hacia el centro de la trayectoria.

Si en un DCL se incluye la
c
a , deber dibujrsela a un lado del cuerpo, para indicar
que no es una fuerza.
EJEMPLO: Para ver el tratamiento de este tema, consideremos
el caso de la rueda de la fortuna, que se mueve con MCU y
supongamos que deseamos obtener la expresin correspondiente
a la reaccin normal N, en dos puntos particulares: en el cenit y
en el nadir. Los respectivos diagramas de cuerpo libre son:









C E N I T N A D I R

Las nicas fuerzas son las N y P, ambas verticales. Se ha agregado un eje de referencia
y, positivo hacia arriba. Si queremos obtener las expresiones de N
C
y N
N
, hacemos:

En el CENIT: F
i
= F
neta
N
C
P = -m v
2
/R
Luego: N
C
= m(g -
R
v
2
)
En el NADIR: F
i
= F
neta
N
C
P = + m v
2
/R
Luego: N
C
= m(g +
R
v
2
)

Ejercicio.
Hacemos girar un balde con agua de modo que d vueltas en un plano vertical; el balde
est atado al extremo de una cuerda de 1m de longitud. Cul es la mnima velocidad que
deber tener el balde para que el agua no caiga cuando ste tiene su
boca dirigida hacia abajo?
Solucin:
La figura muestra el DCL para el caso de un observador
inercial. Se ha adoptado un eje de referencia y, positivo hacia abajo.
N es la reaccin normal que el fondo del balde hace contra el agua.
Tenemos: F
neta
= ma mg + N = ma
En el caso lmite de la velocidad mnima, es N = 0, mg es la fuerza
centrpeta y a es a
c
= v
2
/R.
Luego, mg = mv
2
/R
v =
s
m
g R 16 , 3 10 1 = =
a
a
R
v
v
y
a
y

Nc
P = mg
y
a
y

N
N
P = mg
R
N
mg
y
a
c
La fuerza centrfuga
Aprovechando este ltimo problema que trata de un movimiento circular, vamos a
referirnos a una fuerza que se suele mencionar comnmente, y es la llamada fuerza centrfuga.
(analizar los pares de accin y reaccin en el esquema y
mostrar que la F
centrfuga
no tiene igual y contraria. Que hay
fuerzas newtonianas y no newtonianas y concluir con que la
F
centrfuga
es una falsa fuerza o tambin llamada fuerza ficticia.
Referirse a los marcos de referencia inerciales y no inerciales
y que en este curso solamente se trabajar dentro de marcos
de referencia
inerciales. Por lo tanto debe evitarse su uso y su
mencin en los diagramas de cuerpo libre. Concluir
con el ejemplo del automvil que toma la curva de la
figura).


Principio de la conservacin de la energa mecnica
La energa cintica que terminamos de estudiar es una forma que tiene un sistema de
almacenar energa, en este caso, en razn de la rapidez que tienen las partculas que lo forman.
Pero existen en la naturaleza otras formas mecnicas de almacenar energa en razn de la
posicin relativa de las partes que forman el sistema; hemos estudiado la Ep
g
y pronto veremos
la Ep
e
. Veamos un ejemplo. Supongamos que lanzamos una pelota hacia arriba, y no importa si
est subiendo o bajando, sobre ella acta una nica fuerza: el peso. El peso es el producto de la
interaccin entre la pelota y la Tierra.

Es un error frecuente expresarse diciendo por ejemplo: la energa potencial de la pelota
en el punto 1 es La pelota no tiene energa
potencial; quien la tiene es el sistema pelota-Tierra.
La energa es un fenmeno compartido entre dos
cuerpos; nunca es patrimonio de uno solo. La
energa es como un hijo; todo hijo tiene dos padres;
nadie tiene un padre solo. As como todo hijo es el
producto de la interaccin entre dos padres, la
energa es el producto de la interaccin entre dos
cuerpos.

Si 1 y 2 son los estados inicial y final y teniendo en cuenta que el peso es una fuerza
conservativa, el trabajo realizado por ella es:
W
P
= mgy
1
mgy
2
= Ep
g1
Ep
g2
.

Si aplicamos el teorema del trabajo y la energa: W
Fneta
= Ec
tendremos, reemplazando:
Ep
g1
Ep
g2
= Ec
2
Ec
1
.

Reordenando trminos: Ep
g1
+ Ec
1
= Ep
g2
+ Ec
2
.
Em
1
= Em
2
.
Em = cte
sta es la expresin del principio de la conservacin de la energa mecnica. Llamamos
energa mecnica Em al conjunto de las energas potencial y cintica.

1
2
y
x
y
1

y
2

mg
P
F
centrfuga
F
centrpeta
R
N
Centro de
rotacin
a
Sistema mecnico
Cuando un detective tiene que investigar un crimen, al comienzo piensa que todo el
resto de la humanidad es sospechosa, pero poco a poco encuentra que muchos grupos de
personas, por diferentes motivos, son totalmente ajenas al hecho que investiga, y entonces
procede a descartarlo de su crculo de sospechosos. Y as, varias veces, hasta que finalmente
Ens. crculo de sospechosos quedan slo 4 5 personas. Ahora tiene un panorama claro: sabe
dnde debe investigar y dnde no. Cuando un cientfico debe estudiar un fenmeno o un
proceso, de cualquier naturaleza que sea, procede de la misma manera. Al encerrar todos
aquellos elementos necesarios y slo los necesarios- para su estudio, crea un sistema.

Nosotros, que estudiamos una parte de la fsica llamada mecnica, trabajaremos con
sistemas mecnicos. Los elementos que reuniremos no sern personas, como en el ejemplo del
detective, sino partculas o cuerpos.


Un sistema mecnico es una reunin de partculas o cuerpos seleccionados con el fin de poder
analizar y resolver un problema determinado.

Cules partculas o cuerpos debern incluirse en el sistema, depender de cada
problema. En principio un sistema puede llegar a contener una sola partcula aunque tal
sistema seguramente resulte muy poco til, dado que como ya habrn apreciado a lo largo de
este curso, en esta materia, todas las cosas que se estudian son consecuencia de interacciones
entre dos cuerpos, como es el caso de las fuerzas y de la energa en todas sus formas. Es
importante saber que si por ejemplo se va a plantear alguna
forma de energa, debern estar contenidos en el sistema a los dos
actores responsables de dicha energa. As, en el ejemplo de la
pelota lanzada en el tiro oblicuo de recin, el sistema deber estar
constituido por la pelota y la Tierra.

La figura muestra un sistema constituido con los cuerpos
1 y 2, mientras que el cuerpo 3 ha sido excluido; tambin se
muestran todas las fuerzas de interaccin posibles entre los tres cuerpos. Hay pares de fuerzas
como las F
13
y F
23
que tienen una componente dentro del sistema y la otra afuera; en cambio el
par de fuerzas F
12
tiene ambas componentes dentro del sistema. Esta diferencia da lugar a que
las fuerzas se clasifiquen en externas e internas.

Fuerzas externas son las que resultan de la interaccin entre un cuerpo del sistema y otro que
se encuentra fuera de l.
Fuerzas internas son las que resultan de la interaccin entre dos cuerpos del sistema.

Ntese que esta clasificacin est subordinada a la forma en que haya sido formado el sistema.

Cuando no hay fuerzas externas el sistema recibe el nombre de cerrado o aislado.

Los principios de conservacin.
El estudio de los sistemas cerrados es particularmente importante, pues nos conduce a
una serie de propiedades que nos permitirn abordar la solucin de ciertos problemas de una
manera ms sencilla. As se puede demostrar que en los sistemas cerrados se conservan (o sea
que permanecen constantes) tres magnitudes fsicas: la energa mecnica, la cantidad de
1
2
3
Sistema
mecnico
movimiento (o momento lineal) y el momento cintico (o momento angular). En
correspondencia con lo dicho, existen tres principios de conservacin:
el principio de conservacin de la energa mecnica (E)
el principio de conservacin del momento lineal ( p

)
el principio de conservacin del momento angular ( L

)
Estos principios estn estrechamente ligados con las propiedades bsicas del espacio y
del tiempo. Los tres son principios bien precisos que se cumplen con todo rigor an en la
regin relativista. Los tres principios constituyen potentes recursos para la investigacin.
Sucede con frecuencia que la solucin estricta de las ecuaciones de movimiento es
extremadamente complicada. En tales casos, con la ayuda de los principios de conservacin,
sin resolver las ecuaciones de movimiento, es posible obtener una serie de importantes datos
acerca del transcurso de los fenmenos mecnicos.

El primero de estos principios de conservacin acaba de ser presentado. Pero, cul es
su mbito de cumplimiento, o cules son sus limitaciones? Se puede demostrar que este
principio tiene 2 restricciones: una impuesta a las fuerzas externas y otra impuesta a las fuerzas
internas que realizan el trabajo computable dentro del sistema:

entonces:
se cumple el principio de conservacin de la energa mecnica

Lo podemos enunciar as:

En todo sistema cerrado de cuerpos entre los que slo actan fuerzas conservativas, la energa
mecnica permanece constante.

Su expresin matemtica es: Ep
1
+ Ec
1
= Ep
2
+ Ec
2
.

Si no se cumpliera la primera condicin (que el sistema debe ser cerrado), intervendran
sobre el sistema fuerzas externas capaces de hacer trabajo; precisamente ser:
W
Fext
= Em = Em
2
Em
1
.
Si no se cumpliera la segunda condicin, (si hubiera fuerzas no conservativas haciendo
trabajo computable dentro del sistema, del mismo modo, se podra poner:
W
Fno conserv
= Em = Em
2
Em
1


Estas dos expresiones constituyen una forma modificada del teorema del trabajo y la
energa, y son sumamente tiles al momento de resolver problemas. Se enuncia:

El trabajo realizado por las fuerzas externas o por las fuerzas no conservativas es igual a la
variacin de la energa mecnica total del sistema.
Si
No hay fuerzas
externas
(sistema cerrado)
Las fuerzas internas que
hacen trabajo, son
conservativas

Energa potencial elstica. (Resorte)
Un resorte almacena energa cuando se encuentra deformado. Para poder deformarlo, es
necesario que una fuerza externa acte sobre l, la que al desplazarse realiza trabajo.

LEY DE HOOKE: En 1678 Roberto Hooke observ que el alargamiento
(moderado) de un resorte es proporcional a la fuerza externa aplicada
sobre el extremo del mismo. En verdad no debera ser considerada una
ley, pues es una afirmacin sobre un dispositivo especfico, pero no
expresa un comportamiento general de la naturaleza.

Fext = kx (1)

Los resortes reales no siempre obedecen a la ecuacin (1). k se llama constante elstica
del resorte; su unidad es N/m. Un resorte blando (de juguete) tiene una k de 1 N/m; un
resorte duro como el de la suspensin de un automvil tiene un k = 10
5
N/m.

TRABAJO EFECTUADO SOBRE EL RESORTE
Para estirar un resorte, debemos aplicar un par de fuerzas externas, iguales y opuestas,
cada una en cada extremo del resorte. Si mantenemos fijo el extremo izquierdo, la fuerza
aplicada en este extremo no efecta trabajo. Slo hace trabajo la fuerza externa aplicada en el
extremo derecho mvil del resorte. A medida que incrementamos esta fuerza, el resorte se
estira desde un x
0
= 0 hasta un X final. Podemos calcular el trabajo de dos maneras:

1 MANERA: Analticamente. W =
2
0 0

2
1
X k dx x k dx F
X X
ext
= =


2 MANERA: Grficamente: El rea del tringulo sombreado
representa el trabajo total realizado por la Fext contra el resorte:
W =
( )
2

2
1
2 2
X k
X k X altura x base
= =

Si el resorte tiene espacios entre espiras cuando est relajado,
tambin puede comprimirse; en este caso, tanto Fext como X toman
sentidos opuestos a los del caso de estirar el resorte. Pero el trabajo conserva en ambos casos el
mismo signo.

El trabajo de la fuerza externa sobre el resorte, siempre es POSITIVO, tanto en el estiramiento
como en la compresin del resorte.

TRABAJO EFECTUADO POR EL RESORTE.
Por el principio de accin y reaccin, la Fext que uno le aplica al resorte, genera en el
resorte otra fuerza igual y contraria, que el resorte le aplica a uno. Esta fuerza, que le pertenece
al resorte, se llama fuerza recuperadora; ella hace trabajo sobre uno. Este trabajo es siempre
negativo.

El trabajo de la fuerza recuperadora de un resorte es siempre NEGATIVO, tanto en el
estiramiento como en la compresin.

Ntese que si bien la Fext y la Frec constituyen un par de fuerzas de accin y reaccin,
ellas son de naturaleza diferente. La Fext (que bien podra ser una fuerza humana aplicada
Fext
x
W
Fext
kX
x
X
sobre el resorte, es una fuerza no conservativa, ya que su valor puede ser cualquiera,
independientemente de la posicin. En cambio, la Frec es una fuerza conservativa ya que su
valor est regido por la ley de Hooke, una ley del tipo F = f(x) que establece que el valor de la
fuerza recuperadora es funcin exclusiva de la posicin.

Problemas
A- APLICANDO EL PPIO. DE CONSERV. DE LA ENERGA MECNICA.
El sistema de la figura parte del reposo; despreciando
las masas de la polea y cuerda y todos los rozamientos y
considerando que la cuerda es inextensible, calcular la rapidez
que adquiere la masa B en el momento de llegar al suelo.
Solucin:
1 paso: Definir el sistema: Lo que se muestra en la figura
ms la Tierra.
2 paso: Identificar las fuerzas que intervienen: Externas, no
hay. Internas tenemos los pesos de las masas y la tensin en la cuerda.
3 paso: Determinar si se renen las condiciones para que se cumpla el ppio de conserv. de E.
T no es conservativa. Sin embargo igual se cumple. Discutir.
4 paso: Hacer sendos esquemas para los estados inicial y final.

ESTADO INICIAL (1) ESTADO FINAL (2)










5 paso: Plantear ecuaciones y resolver.
Em
1
= Em
2

m
B
gh = m
A
gh + (m
A
+ m
B
)v
2


s
m
m m
h g m m
v
B A
A B
225 , 1
) ( 2
=
+

=


B- APLICANDO LAS FORMAS MODIFICADAS DEL TEOREMA DEL TRABAJO Y
LA ENERGA
1- Se lanza un cuerpo hacia arriba por el plano inclinado
de la figura, con velocidad inicial v
1
= 20 m/s. Si el
coeficiente de rozamiento es
c
= 0,2, calcular la distancia
que recorrer por el plano inclinado hasta detenerse.
Solucin:
W
Fno conserv
= Em
2
Em
1
(1)
Elaboraremos previamente la
m
B
= 5 kg
m
A
= 3 kg
h = 0,30 m
h
T
T
m
A
g
m
B
g h T
T
m
A
g
m
B
g
30
2 1
h
2

v
1

d
Froz
N
Px
Py
expresin que le corresponde al primer miembro; hacemos un diagrama de cuerpo libre:
N = Py = mgcos Fr = N = mgcos
W
Fr
= (mgcos )dcos 180 = -mgdcos
Y teniendo en cuenta que h
2
= dsen , reemplazando en (1):

-mgdcos = mgdsen mv
1
2
.
Simplificando m y despejando d:
m
sen g
v
d 29
) cos ( 2
2
1
=
+
=



PROBLEMAS CON INTERVENCIN DE LA Ep
elstica
.
1- Un cuerpo de 2000 kg se desplaza por una superficie horizontal y embiste un resorte de k =
10000 N/m y se detiene luego de un recorrido x = 0,3 m. Determinar la velocidad del cuerpo
en el momento del choque.
Solucin:
Ec
1
= Ep
el 2
. mv
2
= kx
2

s
m
m
x k
v 66 , 0

2
= =

2- Un cuerpo de 100 g cae desde una altura de 10 m, sobre un resorte de k = 1000 N/m. Hallar
cunto se comprime el resorte al detener la cada del cuerpo.
Solucin:
Epgr
1
= Epel
2
mg(h + x) = (x)
2

Ordenando: k(x)
2
mgx mgh = 0 (Ecuac. de 2 grado en x)
500(x)
2
- 1x 10 = 0 cuyas races son: +0,14 m -0,14 m

Para pensar:
El nio de la figura est parado sobre patines sin friccin sobre una
superficie horizontal; l empuja la pared con sus manos y se pone en
movimiento hacia la derecha, de modo que adquiere energa cintica. Las
fuerzas N y P no hacen trabajo por tener direccin perpendicular al
movimiento. La fuerza resultante es F, pero ella tampoco realiza trabajo ya
que el nio mantiene sus manos apoyadas contra la pared con lo que F no
desplaza su punto de aplicacin. Si aplicamos el teorema del trabajo y la
energa: W
Fneta
= Ec vemos entonces que el primer miembro es nulo,
mientras que el segundo miembro posee un valor distinto de cero. Qu
sucede aqu? Cmo explica esta discrepancia?

F
N
P










9
Impulso y cantidad de movimiento
Hemos estudiado la segunda ley de Newton: F

= mo. Pero an nos quedan


muchas preguntas relacionadas con fuerzas que no pueden contestarse aplicando
directamente esta ley. Por ejemplo:
a) Si un camin de gran porte choca frontalmente con un auto, qu determina
hacia donde se mueven los restos despus del choque?
b) Cuando uno juega al pool, cmo decide la direccin que debe darle a la bola
blanca, para meter la otra bola en la tronera?
c) Cuando un meteorito choca contra la Tierra, cunto de su energa cintica se
libera en el impacto?
Algo que tienen en comn todas estas preguntas es que implican fuerzas acerca
de las cuales sabemos muy poco: las fuerzas que actan entre el auto y el camin, entre
dos bolas de billar o entre un meteorito y la Tierra. Lo curioso es que en esta clase
veremos que no necesitamos saber nada acerca de estas fuerzas para contestar preguntas
de este tipo.
Nuestro enfoque utiliza dos conceptos nuevos: el de impulso y el de cantidad de
movimiento y una nueva ley de conservacin: la conservacin de la cantidad de
movimiento.
A partir de la segunda ley de Newton, tomada como frmula madre, pueden
deducirse otras expresiones de gran utilidad al momento de resolver problemas. Una de
esas deducciones ya la hemos hecho y obtuvimos el teorema del trabajo y la energa, un
teorema que nos condujo al principio de la conservacin de la energa mecnica. A
continuacin veremos otra importante deduccin a partir de la segunda ley de Newton.
La cantidad de movimiento (p )
Partiendo de la segunda ley de Newton: F

= mo y dado que o =
d
dt
,
podemos poner: F

= m.
d
dt

Si la masa es constante, la podemos introducir dentro de la derivada:
F

=
d( m)
dt

Al producto m: lo llamamos cantidad de movimiento de la partcula y lo
indicamos con el smbolo p . As que p = m: y
F

=
dp
dt
(1)
As, la segunda ley de Newton puede interpretarse ahora como:
La fuerza neta que acta sobre una partcula es igual a la rapidez con que cambia su
cantidad de movimiento.

Ntese que p es una cantidad vectorial cuya direccin y sentido coincide con el
de :. De modo que un auto que viaja hacia el norte a 20 m/s y otro auto idntico que
viaja hacia el este a 20 m/s tienen el mismo | p|pero diferentes p porque sus direcciones
son distintas.
En el MKS la unidad de p es k.m/s
Impulso (I

).
Suponiendo que la fuerza neta que acta sobre la partcula permanece constante,
se define el impulso (I

) de dicha fuerza, como el producto entre dicha fuerza neta y el


t durante el cual acta sobre la partcula:
I

= F

At I

= F
nctu

( t
2
t
1
) (2)
Ntese que I

es una cantidad vectorial cuya direccin y sentido coincide con el


de la F
nctu

.
En el MKS la unidad de I

= N.s. Si desarrollamos la expresin del Newton y


operamos con sus unidades, veremos que N.s = kg.m/s con lo que el impulso y la
cantidad de movimiento se expresan con las mismas unidades.
Teorema del impulso y la cantidad de movimiento.
Si la fuerza neta es constante, la ecuacin (1) puede escribirse as:
F
nctu

=
p
2
- p
1

t
2
- t
1
y F
nctu

. (t
2
t
1
) = p
2
p
1

Y tomando en cuenta la ecuacin (2): I

= p
2
p
1
(3)
La (3) es conocida como Teorema del impulso y la cantidad de movimiento, que
se enuncia as:
El impulso de la fuerza neta sobre una partcula, es igual a la variacin de la cantidad
de movimiento que experimenta la partcula.

Principio de conservacin de la cantidad de movimiento.
Sea un sistema de partculas cerrada, o sea libre de fuerzas externas. Si las
partculas del sistema interactan, las fuerzas mutuas obedecen al principio de accin y
reaccin y la F
internas
= 0.
Por lo tanto, en el sistema cerrado, la F
neta
es nula, y si no hay F
neta
no hay
impulso. La ecuacin (3) predice entonces que:
p
1
= p
2

En todo sistema cerrado, la cantidad de movimiento permanece constante.

Ntese que este principio se refiere a la cantidad de movimiento del sistema, no
a la de cada partcula. Las fuerzas internas pueden cambiar las cantidades de
movimiento individuales de cada partcula. Lo que no puede cambiar es la cantidad de
movimiento total del sistema.
Este principio de conservacin es vlido an en situaciones en las que las leyes
de Newton son inadecuadas, tales como cuerpos que se mueven con una rapidez muy
alta (cercana a la de la luz) u objetos muy pequeos (como las partculas subatmicas).
Finalmente, esquematizando:
En todo
SISIEHA
CERRA0
SE CUMPLE EL
PPI0. E C0NSERIACIN
E IA CANI. E H0IIH.

Este principio, es el segundo de los grandes principios de conservacin que hasta
ahora hemos estudiado. En la Fsica, los principios de conservacin son de importancia
terica y prctica, ya que son sencillos y universales; todos ellos tienen formas
semejantes: en un sistema que est cambiando, existe algn aspecto que permanece
inalterado.
Si comparamos los dos principios de conservacin estudiados, vemos que para
que el cumpla el de la conservacin de la Em, es necesario que se renan
simultneamente dos condiciones (que el sistema sea cerrado y que las fuerzas sean
conservativas), mientras que para que se cumpla el de la conservacin de la cantidad de
movimiento, se requiere de una sola condicin (que el sistema sea cerrado). Al ser este
ltimo principio de conservacin menos exigente que el primero, sus oportunidades de
aplicacin en la prctica son ms numerosas, por lo que lo convierte de hecho en un
principio ms importante que el de la conservacin de la energa mecnica.
Comparacin de cant. de movim. con energa cintica.
El teorema del impulso y de la cantidad de movimiento:
F
nctu

. (t
2
t
1
) = p
2
p
1
dice que la variacin de la cantidad de movimiento de una
partcula depende de la fuerza neta y del tiempo durante el cual ella acta.
El teorema del trabajo y la energa: W
F neta
= Ec
2
Ec
1
dice que la variacin de
la energa cintica de una partcula depende de la fuerza neta y del desplazamiento
necesario para acelerar la partcula.
Veamos un ejemplo. Hay dos pelotas en movimiento; la 1 con m
1
= 0,5 kg y
v
1
= 4 m/s y la 2 con m
2
= 0,1 kg y v
2
= 20 m/s. Cul es ms fcil de atajar?
Solucin -Clculo de sus cantidades de movimiento:
P
1
= m
1
v
1
= 0,5 kg 4 m/s = 2 kg m/s
P
2
= m
2
v
2
= 0,1 kg 20 m/s = 2 kg m/s
Ambas pelotas tienen la misma cantidad de movimiento: p
1
= p
2

-Clculo de sus energas cinticas:
Ec
1
= m
1
v
1
2
= 0,5 kg (4 m/s)
2
= 4 J
Ec
2
= m
2
v
2
2
= 0,1 kg (20 m/s)
2
= 20 J
Las Ec son diferentes; la pelota ms grande es ms lenta.


-Conclusiones: Por ser p
1
= p
2
, ambas requieren el mismo impulso para detenerlas. Pero
detener la bola ms pequea requiere un trabajo 5 veces mayor que detener a la ms
grande. En definitiva, si las atajamos con la mano, tardaremos el mismo tiempo en
detener cualquiera de las pelotas, pero nuestra mano ser empujada una distancia 5
veces mayor hacia atrs, si decidimos atajar la pelota ms pequea. As que nos
conviene atajar la pelota ms grande.

Ejercicios.
1-Se lanza horizontalmente contra una pared vertical una pelota de 0,4 kg; su rapidez
antes de chocar es de 30 m/s y rebota con una rapidez de 20 m/s. El tiempo de contacto
con la pared ha sido de 0,01 s. Calcular: a) las cantidades de movimiento de la pelota
antes y despus del choque; b) la fuerza media ejercida por la pared contra la pelota.
Solucin:
Lo primero que se debe hacer es un bosquejo del problema; luego, adoptar un eje
de referencia. Finalmente calcular.
a) p
1
= m
1
v
1
= -12 kg m/s
p
2
= m
2
v
2
= + 8 kg m/s
I = p
2
p
1
= + 20 Ns
b) F
media
=
I
t
= 2000 N
2- Un tirador sostiene holgadamente un rifle de m
R
= 3 kg de manera que pueda
retroceder libremente en el momento de hacer el disparo. Dispara horizontalmente
contra un blanco, una bala de m
B
= 5 kg con v
B
= 300 m/s. Calcular a) la velocidad de
retroceso del rifle; b) la cantidad de movimiento y la energa cintica de la bala; c) dem
del rifle.
Solucin:
a) p = 0 = m
R
v
R
+ m
B
v
B

v
R
= -v
B

m
B
m
R

= -
0,005
3
300 = -0,5 m/s
b) p
B
= m
B
v
B
= 0,005 x 300 = 1,5 kg m/s
Ec
B
= m
B
v
B
2
= (0,005)(300)
2
= 225 J
c) p
R
= m
R
v
R
= 3 (-0,5) = -1,5 kg m/s
Ec
R
= m
R
v
R
2
= (3)(-0,5)
2
= 0,375 J
3- Un padre y su hijo estn parados en una pista horizontal de hielo, sin friccin. Sus
masas valen 80 kg y 40 kg respectivamente. Se empujan mutuamente y el padre se aleja
con v
P
= 0,5 m/s. a) Cules principios de conservacin se cumplen? b) Con cunta
velocidad se mover el nio?
DESPU
ANTES
:
2

:
1

X
ANTES
DESPUS
Rifle + bala
vR
vB
Solucin:
a) El padre y su hijo forman un sistema cerrado porque las nicas fuerzas externas:
P y N se cancelan entre s. Como fuerzas internas tenemos las de empuje mutuo, que no
son conservativas. Por lo tanto se cumple el principio de conservacin de la cantidad de
movimiento, pero no el de la energa.
b) p (antes) = p (despus) 0 = m
P
v
P
m
H
v
H
v
H
= v
P

m
P
m
H
= 0,5
80
40
= 1 m/s
Em (antes) = 0
Em (despus) = [m
P
v
P
2
+ m
H
v
H
2
] = 30 J
______________________________________________________________________





























10
Choque
El principio de la conservacin de la cantidad de movimiento encuentra su
aplicacin ms valiosa en el estudio del choque.
Se llama CHOQUE al fenmenos fsico que tiene lugar cuando dos cuerpos se
encuentran (e impactan) a consecuencia de su movimiento relativo.
Para estudiar este fenmeno, lo primero que debe hacerse es definir el sistema
que va a ser motivo de estudio; se estudiar nicamente el caso de tener un sistema
aislado, o sea que no intercambia fuerzas con el mundo externo: por lo tanto el impulso
sobre el sistema es nulo y se cumple el principio de la conservacin de la cantidad de
movimiento. Este principio establece que la cantidad de movimiento del sistema antes
del choque es igual a la cantidad de movimiento del sistema despus del choque. Ntese
que este principio se aplica al sistema y NO a cada cuerpo en particular.

Adems, la conservacin de la cantidad de movimiento es una premisa vlida
para cualquier tipo de choque.






Caractersticas
del proceso de
choque
1- Tiene una duracin sumamente breve.
2- Se producen grandes cambios de velocidad.
3- Se producen cambios de forma.
4- Como consecuencia de 1) y 2), va acompaado de aceleraciones
elevadas.
5- Como consecuencia de 4), intervienen fuerzas muy grandes.
Clasificacin de los choques.
Hay varias maneras de clasificar los choques, segn el aspecto desde el cual se
los observe.


Por las caractersticas
del movimiento antes
del choque. (Condic.
Geomtricas).
CUOQUE CENTRAL: Tiene lugar cuando la recta m que
une a ambos centros de masa, es perpendicular al plano
tangente a la superficie de los cuerpos en el punto inicial de
contacto.
Como un caso particular dentro del choque central est el:
CHOQUE NORMAL: que ocurre si :
1
y :
2
estn contenidos
en la recta m citada.
CHOQUE EXCNTRICO: (No se estudia).
La figura de la pgina siguiente ilustra el caso de los choques central normal y
central no normal.
Otra forma de clasificar los choques se basa en las condiciones elsticas de los
materiales. En esta clasificacin hay tres posibilidades:
CHOQUE PERFECTAMENTE ELSTICO: Cuando se recuperan totalmente tanto las
formas como las dimensiones. La energa cintica se conserva.
CHOQUE PERFECTAMENTE PLSTICO: Cuando no hay absolutamente ninguna
recuperacin. No hay fuerzas de restitucin, por lo que las deformaciones se conservan
en su totalidad.
CHOQUE SEMIELSTICO: Es un caso intermedio entre los anteriores. Son los
verdaderos choques reales.

Tambin se suele clasificar a los choques en base al nmero de dimensiones en
que tienen lugar: unidimensionales, bidimensionales o tridimensionales.

Velocidad relativa:
En el estudio del choque, lo que cuenta es la velocidad relativa. Si los cuerpos
que van a chocar son los 1 y 2, que se mueven con velocidades absolutas v
1
y v
2
, la
velocidad relativa v, si el choque es unidimensional, se define como:
v = v
1
v
2
(o al revs)

Energa en el choque.
Antes del choque los cuerpos poseen una determinada cantidad de energa, en
forma de energa cintica. Durante el proceso de choque generalmente, el sistema pierde
una parte de esa energa. Los principales destinos para esa energa que se pierde son:
1- la ejecucin de trabajos de deformacin.
2- en aumentos locales de la temperatura por la friccin, que terminan con una entrega
de calor al medio ambiente.
3- en la generacin y propagacin de ondas de ruido.
De manera que: Ec
despus
< Ec
antes

Existe una clase extrema de choque, donde Ec
despus
= Ec
antes
. Es el choque
perfectamente elstico; es la clase de choques que tienen lugar entre las molculas de un
gas.
Planteo matemtico del choque.
En todo choque es posible distinguir siempre dos etapas. La primera comienza
con el primer contacto entre ambos cuerpos y le siguen deformaciones crecientes en
ellos; esta etapa concluye cuando tales deformaciones alcanzan su mximo. Las fuerzas
que intervienen en esta etapa se llaman fuerzas de interaccin. Ellas son fuerzas internas
al sistema constituido por los dos cuerpos que chocan, y por lo tanto la cantidad de
movimiento permanece constante.
Antes del
choque
Despus
del choque
m
m
m
m
t
t
Choque central normal Choque central no normal
v
1
v
2

v
1
v
2


La segunda etapa comienza cuando los cuerpos alcanzan su mxima
deformacin (velocidad relativa nula); las fuerzas que intervienen en esta etapa se
llaman fuerzas recuperadoras y tienden a eliminar o al menos reducir las deformaciones
producidas en los cuerpos.

El esquema que sigue ilustra estas etapas para el caso de una bola (cuerpo 1) que
choca contra una pared fija (cuerpo 2); se dan las velocidades relativas entre ambos en
cada momento. Se utiliza una prima para los valores despus del choque. Se calculan los
impulsos de cada etapa (frenante y de restitucin respectivamente) y finalmente se llega
a la expresin del coeficiente de restitucin k como cociente entre ambos impulsos.
Comienza la 1 etapa Finaliza la 1 etapa Finaliza la 2 etapa
Comienza la 2 etapa
v
rel
= v
1
v
2
= v
1
v
rel
= v
1
v
2
= 0 v
rel
= v
1
v
2
= v
1


I = Impulso frenante I = Impulso de restitucin.

I = m(Vrel
final
Vrel
inic
) I = m(Vrel
final
Vrel
inic
)

I = m(0 v
1
) = -mv
1
I = m(v
1
0) = +mv
1


Coeficiente de restitucin k:

k =
I
I
=
Impulso Jc rcstitucin
Impulso {rcnontc
=
:
1
:
1


En esta deduccin se ha considerado el caso particular de un choque contra una
pared inmvil, por una cuestin de sencillez. Pero si ambos cuerpos se mueven, las
velocidades relativas seran:

ANTES: v = v
1
v
2
I = -m(v
1
v
2
)
DESPUS: v
1
v
2
I = m(v
1
v
2
)

con lo que la expresin del coeficiente de restitucin k para un caso general es:

k =
i
1
- i
2

1
-
2
(1)

El coeficiente de restitucin expresa la razn entre la velocidad relativa despus del
choque cambiada de signo y la velocidad relativa antes del choque.
v
1

v
1

v
1
= v
2
= 0 v
2
= 0
v
2
= 0
PRIMERA ETAPA SEGUNDA ETAPA
1 CONTACTO MXIMA DEFORMACIN
LTIMO
CONTACTO

k tiene valores comprendidos entre 0 y 1; estos valores extremos corresponden
respectivamente a los casos de choques perfectamente plstico y elstico. Los valores
del intervalo: 0 < k < 1 corresponden a los choques semielsticos.
Choque unidimensional, central y normal.
Veremos cmo se plantea un problema de choque. Lo ms comn es que los
datos tengan que ver con las condiciones iniciales del choque, y las incgnitas sean las
finales. Si ese fuera el caso, tendremos:
DATOS: v
1
v
2
k m
1
m
2

INCGNITAS: v
1
v
2

SOLUCIN: Habitualmente las incgnitas son 2. Para poder resolverlo ser necesario
planteas un sistema de 2 ecuaciones con 2 incgnitas. Una de ellas ser la que
corresponde al principio de la conservacin de la cantidad de movimiento entre antes y
despus del choque. sta ser una ecuacin vlida siempre, en cualquier choque:
m
1
v
1
+ m
2
v
2
= m
1
v
1
+ m
2
v
2
(2)
La otra ecuacin a plantear es la (1)
k =
i
1
- i
2

1
-
2
(1)
Ejercicio 1.
Se deja caer al piso una esfera desde 2 m de altura. Si k = 0,8, determinar la
altura alcanzada despus del choque.
Solucin: Como el piso no se mueve, estamos en el caso particular en que k = -
i


Deberemos calcular previamente las expresiones de v y de v. La primera es la
velocidad final de una cada libre desde 2m de altura y que comenz con velocidad
inicial nula. Por la cinemtica es: v = 2 g
La segunda (v) es la velocidad inicial de un tiro vertical que se elevar hasta
una altura final h: v = 2 g
Finalmente: k = -
i

= -
2ghi
2gh
= _
hi
h

Elevando al cuadrado: k
2
=
hi
h
h = hk
2
= 2 m(0,8)
2
= 1,28 m
Choque en dos dimensiones.
Si el choque tiene lugar en dos dimensiones, la solucin es muy fcil. Tan solo
hay que proyectarlo en las dos direcciones cartesianas y tratarlo como dos choques
unidimensionales. Veamos un par de ejemplos:
Ejercicio 2.
Dos patinadores sobre hielo se acercan uno al otro en ngulo recto. El patinador
A tiene una masa de 50 kg y viaja en direccin y sentido +x a 2 m/s. El B tiene una
masa de 70 kg y se mueve segn +y a 1,5 m/s. Chocan y quedan unidos. Calcular a) la
velocidad final de ambos; b) la prdida de energa cintica habida en el choque.
Solucin:
a) En x] m
A
v
A
= (m
A
+ m
B
)v
x

En y] m
B
v
B
= (m
A
+ m
B
)v
y

v
x
=
m
A

A
m
A
+ m
B
=
502
120
= 0,830 m/s
v
x
=
m
B

B
m
A
+ m
B
=
701,5
120
= 0,875 m/s
v
f
= :
x
2
+ :

2
= 0,7656 + 0,694 = 1,21 m/s
b) Ec
f
= mv
f
2
=
1
/
2
120(1,21)
2
= 87,60 J
Ec
0
= [m
A
v
A
2
+ m
B
v
B
2
] = [504 + 702,25] = 178,75 J
Ec = Ec
f
Ec
0
= (87,60 178,75) J = -91,15 J
Ejercicio 3.
Una bola de billar se mueve con v = 0,36 m/s y choca con otra igual que se
encuentra en reposo. Despus del choque, la primera rebota con velocidad de 0,15 m/s,
formando un ngulo de 37 con la direccin que traa antes del choque. Determinar la
velocidad con que sale la otra bola. Hubo prdida de energa cintica en el choque?

v
A
= 2 m/s
A
m
A
= 50 kg
v
B
=1,5 m/s
B
m
B
= 70 kg
ANTES DESPUS
1
v
1
= 0,36 m/s
2
v
2
= 0
1
2
v
1
= 0,15 m/s
v
2

37|
Solucin: a) v
1x
= v
1
cos 37 = 0,12 m/s
v
1y
= v
1
sen 37 = 0,09 m/s
En x] m
1
v
1
= m
1
v
1x
+ m
2
v
2x
v
2x
= 0,24 m/s
En y] 0 = m
1
v
1y
+ m
2
v
2y
v
2y
= - 0,09 m/s
Finalmente: v
2
= _:
2x
2
+ :
2
2
= 0,256
m
s

0
2f
= arc tg
-0,09
0,24
= 20 = 340
b) Ec
f
= m [v
1
2
+ v
2
2
] = 0,044m J
Ec
0
= mv
1
2
= 0,65m J
Ec = - 0,6m J
El pndulo balstico.
El tema del choque encuentra su principal aplicacin prctica en el pndulo
balstico, que se utiliza para medir la velocidad de una bala; consta de un gran bloque de
madera de masa M que cuelga sostenido por un par de cuerdas. Se elige el tipo de
madera de modo que la bala, que se
mueve horizontalmente, quede
incrustada en l.

Se procura que el perodo de
oscilacin del pndulo resulte muy
grande con respecto al tiempo que
tarda la bala en quedar incrustada. De
esta manera el pndulo empieza a moverse inmediatamente despus que la bala queda
ahogada en la madera y puede considerarse que se conserva la componente horizontal
de la cantidad de movimiento.
La cantidad de movimiento antes del choque es la cantidad de movimiento de la
bala: mv. La cantidad de movimiento del sistema inmediatamente despus del choque
es: (M + m)v.
Como se trata de vectores de igual direccin, podemos escribir la ecuacin en
forma escalar:
mv = (M + m)v
Despus del impacto, el pndulo entra en oscilacin y deben medirse, con buena
aproximacin, la altura mxima alcanzada en la primera oscilacin. La energa cintica
M
m v
h
del bloque, con la bala incrustada, se habr transformado en energa potencial, y por el
principio de la conservacin de la energa mecnica, podr ponerse:
1
2
( H+ m) :
2
= ( H+ m) g
de donde v = 2 g Finalmente es:
v = j
M+m
m
[ 2 g (3)
Ejemplo:
Una bala de 10 g choca, movindose horizontalmente, con un pndulo balstico
de M = 2 kg. En la primera oscilacin el pndulo se eleva verticalmente 16 cm. Calcular
la velocidad de la bala sabiendo que qued ahogada en el pndulo.
Solucin:
Reemplazando en la frmula (3): v = j
( 2+0,01)
0,01
[ 2 9,8 0,16 = 356
m
s




























11
Sistemas de partculas
Existen en la naturaleza slo dos formas posibles de movimiento: la traslacin y
la rotacin. La primera ocurre cuando la partcula sigue una trayectoria rectilnea,
mientras que en la segunda describe trayectorias circunferenciales alrededor de un
punto.
Nosotros, que ya hemos estudiado la dinmica de la traslacin, sabemos que la
causa de tales movimientos, son las fuerzas. Pero, cules son las causas del
movimiento de rotacin? Evidentemente no sern las fuerzas, ya que no podemos
atribuirle a una misma causa dos efectos diferentes. As que nos ocuparemos de
descubrir cul es la causa que provoca un movimiento de rotacin.
Supongamos tener una varilla homognea que
puede girar alrededor de un eje que pasa por el punto O.
Si le aplicamos una fuerza a la varilla, podrn suceder
tres cosas diferentes, segn dnde se aplique dicha
fuerza:
1- Si la recta de accin de la fuerza pasa por O,
la varilla permanecer inmvil.
2- Si aplicamos la fuerza a la derecha de O, la varilla girar en un cierto sentido.
3- Si la aplicamos a la izquierda, la varilla girar en sentido contrario.
F F
F
O
(3) (1) (2)
El efecto de rotacin producido se llama momento de una fuerza o torca, y su
valor depende de la combinacin de dos factores: el vector fuerza F

y el vector posicin
r. El vector r tiene su origen en el origen de nuestro sistema de referencia, origen que
ubicaremos en O, donde est el eje de rotacin, y su extremo est en contacto con la
fuerza. Como se ve, para producir un momento (o torca), la fuerza es necesaria, pero no
es suficiente.
Indicaremos al vector torca con la letra gamma mayscula del alfabeto griego
() y matemticamente est expresado por el producto vectorial:
I

= r F


Recordar que en un producto vectorial, el orden de los factores es importante;
aqu, r siempre va adelante y F

atrs. Debe ser as para que cuando se aplique la regla


prctica de los sentidos de los vectores, el sentido de I

concuerde con la realidad fsica.


El desarrollo escalar de este producto vectorial es:
| I| = | r| | F| scn 0

En el ejemplo de la figura, la direccin del
vector I

es perpendicular al plano que contiene a r y


a F

y su sentido es entrante.

Teorema de Varignon.
Sea un sistema de i fuerzas F
i

de las cuales R

es su resultante. Este teorema


expresa que:
La torca resultante de un sistema de fuerzas, es igual a la suma vectorial de las torcas
producidas por cada una de las fuerzas componentes, consideradas todas con respecto
a un mismo punto.

Es decir que si R

= F
i

, entonces, I
R
( 0)
= I
i
( 0)

Nosotros nicamente estudiaremos el caso de sistemas de fuerzas coplanares, es
decir que estn contenidas todas en un mismo plano. Para este caso particular, resultar
que todos los vectores I
i
( 0)
tendrn la misma direccin: la de la perpendicular al plano
que contiene a las fuerzas. Siendo as, la I
i
( 0)
podra efectuarse escalarmente. Eso s:
al desarrollar la sumatoria, habr que asignarle a cada trmino un signo (+ -) segn sea
el sentido de la rotacin correspondiente. Para la asignacin de signos se adoptar una
convencin arbitraria.
Sistemas de partculas.
0
F


r
partcula
origen
I


Recordar que llamamos partcula a un cuerpo ideal que si bien tiene las
dimensiones de un punto geomtrico (es decir que carece de dimensiones), no por ello
carece de masa.
Hemos comenzado este curso estudiando el caso de tener una partcula nica; en
realidad no hubo mucho para decir sobre ella, porque lo verdaderamente valioso aparece
cuando esa partcula interacta con otra. Conceptos tales como los de fuerzas, energa,
cinemtica, dinmica, choque, etc requieren como mnimo de un sistema de dos
partculas. As fue cmo incorporamos una segunda partcula. Dentro de pocas clases
comenzaremos a estudiar al slido, que no es otra cosa que la reunin compacta de
muchsimas partculas. Como una transicin en este camino, estudiaremos ahora los
llamados sistemas de partculas.
Un sistema de partculas es un conjunto finito de partculas, separadas entre s y
distribuidas en el espacio.

Se estudiarn algunas propiedades de estos sistemas y se introducirn algunos
conceptos nuevos que ms adelante, al estudiar al slido, resultarn de gran utilidad.
Centro de masa.
El primer concepto importante a presentar es el de centro de masa. Suponga que
tenemos una varilla homognea como la de la figura; la varilla
est suelta. Si le aplicamos una fuerza vertical hacia arriba en A,
el cuerpo subir realizando una traslacin combinada con una
rotacin en el sentido de las agujas del reloj. Si aplicamos la fuerza en C, subir
combinando una traslacin con una rotacin que, a diferencia del caso anterior, tendr
sentido antihorario. Esto nos lleva a pensar que entre A y C deber existir algn punto
donde al aplicar la fuerza, el cuerpo suba sin rotar, o sea con traslacin pura. Cuando
encontremos ese lugar, habremos encontrado el centro de masa. Y sta es una propiedad
importante del centro de masa:
Si se aplica una fuerza en el centro de masa de un cuerpo o un sistema de partculas,
ste adquirir un movimiento de traslacin pura.

Definicin:
Centro de masa es una posicin media, ponderada por la masa de las partculas.

Si el cuerpo es homogneo, el centro de masa se encuentra en su centro
geomtrico. Cuando el cuerpo tiene un eje de simetra, como una polea, el centro de
masa est sobre dicho eje. No necesariamente el centro de masa debe estar dentro del
cuerpo. El centro de masa de una arandela es un punto de su eje de simetra, pero no
est dentro de la masa de la arandela misma.
A B C
Determinacin de las coordenadas del centro de masa.
Sea un sistema de tres partculas, como se muestra en la figura de abajo a la
izquierda, y se desea determinar las coordenadas del centro de masa (c.m.). Se adopta
un sistema cartesiano de referencia. Los datos son los valores de las masas de las tres
partculas y sus respectivas coordenadas x;y. Cada partcula tiene un peso: P = mg. En
la figura de la derecha se representan las fuerzas peso de cada partcula.

Aplicando el teorema de Varignon, y tomando como centro de momentos al
origen del sistema cartesiano, podemos poner en general:
m
1
gx
1
+ m
2
gx
2
+ .. + m
i
gx
i
= Mgx
cm

donde la M representa a la masa total del sistema de partculas: M = m
i
.
g se simplifica y x
cm
se despeja:
M
x m
M
x m x m x m
x
i i i i
cm
...
2 2 1 1

=
+ + +
=
Para hallar y
cm
se puede imaginar que giramos 90 las figuras con lo que los
pesos toman la direccin del eje x, y razonando de manera similar se obtiene:
M
y m
M
y m y m y m
y
i i i i
cm
...
2 2 1 1

=
+ + +
=
Las expresiones m
i
x
i
, m
i
y
i
, etc se conocen con el nombre de momentos
estticos o de primer orden del sistema de masas.

Si las masas estn distribuidas en el espacio, habr una tercera expresin para
hallar z
cm
, similar a las anteriores.

y
x
1
2
3
0
y
x
1
2
3
0
y
2

y
3

y
1

x
1
x
2
x
3

m
1
g m
2
g m
3
g
Ejercicios.
1- Se tiene un sistema de partculas constituido por tres masas ubicadas en las siguientes
coordenadas: m
1
(2;3) m
2
(0;0) m
3
(4;1) en metros.
Sabiendo que m
2
= 3 m
1
y m
3
= 2 m
1
, hallar las coordenadas del centro de masa.
Solucin:
m
m
x m x m x m
x
cm
... 666 , 1
6
2 3
1
3 1 2 1 1 1
=
+ +
=
m
m
y y m y m
y
cm
... 8333 , 0
6
2 3
1
3 2 1 1 1
=
+ +
=
Luego: c.m. (1,66m; 0,83m)

2- La figura muestra una molcula de agua, donde la distancia d de
los enlaces O-H vale 10
-10
m. Sabiendo que la relacin entre las
masas de los tomos de oxgeno e hidrgeno es de 16 a 1, dnde
est el centro de masa de la molcula?
Solucin:
La enseanza que deja este problema es que cuando hay un
eje de simetra, no slo geomtrica, sino tambin de masas, el centro de masa pertenece
a dicho eje de simetra. Por ello, en estos casos conviene adoptar un sistema de
referencia tal que uno de sus ejes coincida con el eje de simetra, con lo cual slo har
falta calcular una sola coordenada porque la otra valdr cero. Por otra parte,
convendr ubicar el origen del sistema en el tomo de oxgeno, lo que ahorrar el
clculo de un trmino, en la frmula de x
cm
.

La abscisa de los tomos de hidrgeno es: x
H
= 10
-10
m cos 53 = 6x10
-11
m.
| | | |
m x
m x
m m
x m
x
O H
H H
cm
12
11
10 6 , 6
18
10 6 1 2
2
2

= =
+
=

Propiedades del centro de masa.
Las propiedades que daremos a continuacin, nos ensean que para estudiar el
movimiento de un sistema de partculas no es necesario considerar los movimientos
O
H
H
d
53
53
individuales de cada partcula, que son generalmente complicados, o se carece de la
informacin necesaria para poder hacerlo. Simplemente basta con estudiar el
movimiento que sigue el centro de masa del sistema, como si en l se reuniera en una
sola partcula puntual, la totalidad de la masa del sistema.
Por sencillez, en lo que sigue plantearemos el caso de un sistema constituido tan
solo por dos partculas, para que en las ecuaciones de clculo de las coordenadas del
centro de masa se tenga el menor nmero de trminos posible. Por otra parte, al tener
slo 2 partculas, se cuenta con otra ventaja adicional: el sistema es unidimensional y
bastar con plantear la ecuacin de una nica coordenada. Haremos:
2 2 1 1
x m x m x M
cm
+ = (1)
1- VELOCIDAD DEL CENTRO DE MASA
Si se deriva la (1): |

\
|
+ |

\
|
= |

\
|
dt
dx
m
dt
dx
m
dt
dx
M
cm 2
2
1
1

M v
cm
= m
1
v
1
+ m
2
v
2
(2)
i cm
p v M

=
La masa total multiplicada por la velocidad del centro de masa, es igual a la cantidad
de movimiento del sistema de partculas.

Esta primera propiedad nos ensea que para hallar la cantidad de movimiento
total del sistema, no es necesario hallar las cantidades de movimiento de cada una de sus
partculas.
2- ACELERACIN DEL CENTRO DE MASA.
Si se deriva la (2): |

\
|
+ |

\
|
= |

\
|
dt
dv
m
dt
dv
m
dt
dv
M
cm 2
2
1
1

M a
cm
= m
1
a
1
+ m
2
a
2

F
x
= m
1
a
1
+ m
2
a
2

Igualando los primeros miembros de las dos ltimas igualdades:
F
x
= M a
cm
(3)
En la (3) el primer miembro representa la fuerza resultante que acta sobre el
sistema, que no es otra cosa que la fuerza externa total. Por lo tanto, esta propiedad nos
dice que:

Bajo la accin de la fuerza externa resultante sobre el sistema, el centro de masa se
acelera como si se tratara de una nica partcula de masa M.

Esta segunda propiedad nos ensea que el movimiento del centro de masa de un
sistema slo puede ser modificado por las fuerzas externas; las fuerzas internas no
ejercen ningn efecto sobre el movimiento del centro de masa.
Ejemplo.
Un proyectil explota en dos trozos iguales, cada uno de ellos de masa m, cuando
est en la parte superior de la trayectoria. Uno de los trozos cae verticalmente hacia el
suelo impactando a 2000 m del lugar de lanzamiento, mientras que el otro se mueve
horizontalmente de modo que ambos tocan el suelo simultneamente. Dnde caer el
segundo trozo?
Solucin:
El 1 paso consiste en definir el sistema; l estar constituido por el proyectil ( al
principio ser una partcula nica; despus contendr dos partculas).
La explosin del proyectil es debida a fuerzas internas; segn la 2 propiedad,
estas fuerzas no pueden perturbar el movimiento del centro de masa, por lo que l
continuar describiendo la trayectoria de la parbola original. Como en este caso, el cm
deber equidistar de ambos trozos de proyectil, por tener stos masas iguales, resulta
fcil inferir dnde caer el 2 trozo: Caer a 6000m del lugar de lanzamiento.
Si se desea un planteo
matemtico, puede ponerse:
M x
cm
= M x
1
+ M x
2

x
cm
= (x
1
+ x
2
)
x
2
= 2 x
cm
x
1

x
2
= 2 4000 2000 = 6000 m






y
x (m)
0 2000 4000 6000











12
Momento angular
En esta clase continuaremos presentando magnitudes que ataen al movimiento
de rotacin.
El concepto de momento angular es tambin conocido con una diversidad de
nombres, tales como momento de la cantidad de movimiento, cantidad de movimiento
angular, impulso angular, momentum, etc. Es al movimiento de rotacin, lo que la
cantidad de movimiento lineal es al movimiento de traslacin.
La presentacin de este nuevo concepto, seguir pasos similares a los de la
introduccin del concepto de momento de una fuerza () a partir del de fuerza (F). As
como el vector torca o momento de una fuerza se obtena premultiplicando al vector
fuerza por el vector posicin:
I

= r F


de manera similar el vector momento angular se obtendr premultiplicando al vector
cantidad de movimiento lineal p con el vector posicin. Utilizaremos la letra L para
representar al momento angular:
I

= r p
La figura muestra una partcula de masa m, dotada de una cantidad de
movimiento lineal p que rota alrededor del origen de
una terna cartesiana, en el plano horizontal x;y. La
partcula es ubicada por el vector posicin r , con origen
en el origen de la terna y su extremo sobre la partcula.
Escalarmente, L = rpsen 0
O bi en: L = r mvsen 0.
Obsr vese que si 0 = 0, r esul t a L = 0.

Relacin entre la torca ( I

) y la variacin del momento


angular [
dL

dt

Sabemos que la fuerza neta que acta sobre una partcula, (de acuerdo con la
segunda ley de Newton) provoca su aceleracin, es decir que su velocidad cambie y con
ella, que su cantidad de movimiento lineal p, tambin cambie. Ahora vemos (a travs de
I

= r p) que tambin cambia a I

. Demostraremos a continuacin que justamente la


rapidez de cambio del momento angular es igual a la torca producida por la fuerza neta.
Para ello derivaremos la ecuacin:
I

= r p = r m :
respecto del tiempo, utilizando la regla de la derivada de un producto:
JI

Jt
= _
Jr
Jt
m:_ + _r m
J:
Jt
_
= ( : m:) + ( r m o)
El primer trmino del segundo miembro es nulo, por representar al producto
vectorial de un vector consigo mismo. Por lo tanto, finalmente nos queda:

dL

dt
= r F
nctu


dL

dt
= I
nctu


La rapidez de cambio del momento angular de una partcula es igual a la torca de la
fuerza neta que acta sobre ella.

Momento de inercia (J).
Cuando una partcula o sistema de partculas se encuentran en movimiento de
rotacin, resulta conveniente introducir una nueva cantidad fsica llamada momento de
inercia; ella depende de la masa y de la distancia al eje de rotacin; es una cantidad
escalar. Ser simbolizada con una J.
1- MOMENTO DE INERCIA DE UNA PARTCULA NICA.
Sea una partcula de masa m que rota describiendo una circunferencia de radio r
respecto del eje de rotacin. Su momento de inercia est dado por la expresin:
J = mr
2

r
m
:
p
Z eje de rotacin
Y
X
I


En el S.I. su unidad es kgm
2

2- MOMENTO DE INERCIA DE UN SISTEMA DE PARTCULAS.
Sea un sistema de n partculas en rotacin alrededor de un eje. Se conocen la
masa y la distancia al eje de cada una de ellas; entonces, su J estar expresado por:
J = m
i
r
i
2

Relacin entre L

, ] y m.
Siendo:
I

= r p = r m :
si se trata de una partcula que se mueve
describiendo una circunferencia alrededor del
eje de rotacin, como lo muestra la figura, el
mdulo de I

es:
L = rmvsen 90 = rmv.
La direccin y el sentido de I

se
muestran en la figura. Como v = r,
reemplazando:
L = mr
2

Finalmente: L = J
La conservacin del momento cintico.
La ecuacin
dL

dt
= I
nctu

, que fue escrita para una partcula, puede ser aplicada a


un sistema de partculas, interpretando que nos dice que la rapidez con que vara el
momento angular de un sistema con respecto a un punto dado, es igual a la sumatoria de
las torcas externas que actan sobre l:
JI

Jt
= I
cxt


Si el sistema se encuentra aislado, tal I
cxt

valdr cero y entonces tendremos:



dL

dt
= 0 o sea que I

= constante
Si la torca externa neta sobre el sistema es nula, el momento angular del mismo
permanece constante.

m
r
m
1

m
2

m
3

r
1

r
2

r
3

z
I



x
r
:
p y
m
90
O
ste es el enunciado del principio de la conservacin del momento angular. Es el
tercer principio de conservacin aplicable a los sistemas mecnicos aislados que
estudiamos. Se agrega a los ya vistos: principio de conservacin de la energa mecnica
y de la cantidad de movimiento lineal. Como aqullos, ste tambin constituye una ley
fundamental de la naturaleza.
Si se tiene un sistema de partculas aislado, los momentos angulares individuales
de cada partcula pueden variar, pero el valor del momento angular total del sistema
debe permanecer constante.
Cuando un cuerpo est girando alrededor de un eje fijo, si I
cxt

= 0,
entonces I

permanece constante. Pero como L = J podr suceder por ejemplo, que


aumente si al mismo tiempo J disminuye. Podemos expresar entonces al principio de
la conservacin del momento angular como:
J
1

1
= J
2

2
= constante
En la prctica, un cuerpo puede cambiar su J, con slo reacomodar su masa
respecto del eje de rotacin. Los acrbatas, los clavadistas, las bailarinas de ballet, los
patinadores sobre hielo, etc, hacen uso de este principio frecuentemente. Si se tiene en
cuenta que J es funcin del cuadrado de la distancia de las partes del cuerpo al eje de
rotacin, resulta fcil comprender que una persona puede lograr notables variaciones en
el valor de su J, con tan solo extender o recoger sus extremidades. Es as como el
patinador y la bailarina de ballet puede aumentar o disminuir la velocidad angular de
su movimiento de rotacin. Tambin los gatos se valen de este principio para caer
siempre parados, cuando se arrojan desde una cornisa. Para ellos, la cola es un apndice
adicional que rene una parte significativa de la masa total del cuerpo que, con solo
extenderla o replegarla, les permite modificar el valor del J de su cuerpo.
Ejercicio.
Se ata al extremo de una cuerda ligera un
objeto de masa m; la cuerda pasa por el interior de un
tubo hueco y sale por el otro extremo del tubo. Se
sostiene el tubo con una mano y la cuerda con la otra;
se hace girar al objeto en una circunferencia de radio
r
1
= 0,40 m con una velocidad tangencial v
1
= 1 m/s.
Despus se tira de la cuerda hacia abajo, acortando el
radio de la trayectoria hasta r
2
= 0,20 m. calcular la
nueva velocidad tangencial v
2
as como las
velocidades angulares inicial y final.
Solucin:
a) Cuando uno tira de la cuerda hacia abajo, le imprime una fuerza que le llega al
objeto como una fuerza radial o centrpeta; esta fuerza no ejerce ninguna torca sobre el
objeto; por lo tanto el momento angular se conserva y podemos poner:
L
1
= L
2
= constante
Desarrollando: mv
1
r
1
= mv
2
r
2
v
2
= v
1
[

2
(1)
v
2
= 1[
0,40
0,20
= 2
m
s

1
=

1
=
1
0,40
= 2,5
1
s

1

m
r
1

r
2

F
b) Reemplazando en la (1) las v por r, obtenemos
2
=
1
[

2
= 2,5 [
0,40
0,20

2
= 10
1
S

Mientras la v se duplic, la se cuadruplic.
































13
Cuerpo rgido

Se ha estudiado el movimiento de rotacin de una partcula; nos ocuparemos en
esta clase del movimiento de rotacin que realiza un cuerpo. Sean las aspas de un
ventilador, la hlice de un avin, un CD en su reproductor, en todos los casos se trata de
cuerpos que giran alrededor de un eje.
Para que los conceptos a estudiar resulten ms claros, evitaremos aquellas
complicaciones accesorias que la rotacin de un cuerpo real nos pueda traer. Para eso
formularemos la hiptesis del cuerpo rgido.
El cuerpo rgido no existe en la naturaleza; es un cuerpo ideal. Mientras todos
los cuerpos reales, bajo la accin de las fuerzas que soportan, son susceptibles en mayor
o menor grado, de sufrir deformaciones tales como torcerse, doblarse o vibrar, el cuerpo
rgido conservar rigurosamente su forma siempre, sin importar cunto de grandes sean
las fuerzas que soporte. Entonces definiremos:

CUERPO RGIDO es todo cuerpo extenso, es decir constituido por muchas partculas,
donde todas ellas guardan entre s relaciones de distancia invariables.

Adems, en la rotacin de un cuerpo (rgido de ahora en ms) podemos
distinguir dos casos, segn que el eje alrededor del cual gira permanezca fijo o no. En el
primer caso hablaremos de rotacin pura, mientras que en el segundo diremos que hay
traslacin y rotacin combinadas (o movimiento rototraslatorio).
El tema de esta clase ser la rotacin pura, y dejaremos para la prxima clase el
caso del movimiento rototraslatorio.
Cinemtica del cuerpo rgido.
Suponga una formacin de soldados que viene marchando durante un desfile, y
que en un cierto momento comienzan a girar en una esquina. Para poder conservar la
formacin, los soldados que se encuentran ms cerca del centro de giro reducen su
rapidez, mientras que los que estn en el otro extremo de la fila apuran el paso. Una vez
completado el giro, podremos decir que todos ellos han girado un mismo ngulo en el
mismo tiempo, por lo que la velocidad angular ser la misma para todos los soldados.
Pero no podemos decir lo mismo de la velocidad tangencial v: cada soldado de la fila ha
girado con una v diferente. En la frmula que relaciona ambas formas de expresar la
velocidad: v = r, mientras ha permanecido constante, v ha crecido con r, siendo r la
distancia desde cada soldado hasta el eje de giro.
Lo mismo le sucede a los habitantes de la Tierra. Cuando ella da una vuelta
completa alrededor de su eje, todos habremos girado con la misma , porque habremos
dado una vuelta completa en 24 horas; pero la v habr sido mayor para quienes estn en
el Ecuador.
La rotacin puede ser uniforme, uniformemente variada o variada; rigen las
respectivas ecuaciones horarias estudiadas para el movimiento circular:

ROTACIN UNIFORME ROTAC. UNIFORM. VARIADA
= 0 = constante
0 = 0
0
+ t + t
2

0 = 0
0
+ t =
0
+ t
a
t
= r
a
n
=

=
2
r a
n
=

=
2
r


Una rotacin queda perfectamente determinada cuando se
conocen 3 cosas: el eje de rotacin, la velocidad angular y el sentido
del movimiento. Estas tres caractersticas quedan representadas mediante
el vector (ver figura) cuya direccin es la del eje de rotacin, su
mdulo es la rapidez angular y su sentido obedece a la regla de la mano
derecha.

Consideremos a continuacin el caso de un cuerpo que se mueve
en un plano, pasando de la posicin inicial (1) a la posicin final (2); ver
la figura en la prxima pgina. Siempre es posible pasar de (1) a (2) componiendo una
rotacin con una traslacin; pero tambin es posible lograrlo mediante una rotacin
nica. En este caso interesar determinar la ubicacin del punto Q al que
denominaremos centro instantneo de rotacin. Entonces ese nico movimiento que
nos lleve de (1) a (2) ser una rotacin alrededor de este centro.
En el movimiento, el segmento AB

pasa a ocupar la posicin AB

. Para ubicar a
Q se hace la construccin geomtrica que se muestra en la figura. Se trazan los
segmentos AA

y BB

y se determinan los puntos medios de ambos. Las perpendiculares


trazadas por dichos puntos medios se cortan en Q.
La posicin de Q se va modificando de acuerdo con las caractersticas del
movimiento; por eso decimos que Q es un centro instantneo. Si deseamos agregar en la
figura los vectores :
A
y :
B
en el estado inicial (1), ellos debern ser perpendiculares
respectivamente a las direcciones A

y B

. Como vimos, la rotacin tiene una nica,


por lo que deber ser:
eje
O

v
A
= A

(1) y v
B
= B

(2)

Mediante las (1) y (2) es posible, conocida la velocidad de un punto, calcular la
de otro cualquiera. Despejando en ambas e igualando las expresiones, se obtiene:

:
A
A
=
:
B
B



Ejercicio 1.
Una chapa cuadrada de 4 m de lado se mueve sobre un plano manteniendo los
vrtices A y D sobre dos rectas ortogonales entre
s. Cuando el punto A se halla a 2 m de O, su
rapidez es v
A
= 1 m/s. Determinar a) la ubicacin
del centro instantneo de rotacin; b) la
instantnea. c) la velocidad del punto D.
Solucin:
Medida de 0

= 4
2
2
2
= 12 = 3,46 m

a) El punto Q es el centro instantneo de rotacin:
Q(3,46;2) en metros.

b) v
A
= r
AQ
=

AQ
=
1
3,46
= 0,29
1
s


c) v
D
= r
DQ
= 0,292 = 0,58
m
s

Dinmica del cuerpo rgido.
La dinmica estudia relaciones de causa-efecto. En la dinmica de la rotacin la
causa es el momento de una fuerza o torca () y el efecto es la aceleracin angular del
cuerpo (). Veremos a continuacin la energa cintica de la rotacin.



Q
A
B
A
B


:
A

:
B



1
2
2 m
A
B
C
D
v
D

v
A
= 1 m/s
Q
O
La energa cintica de la rotacin.

Sea un cuerpo rgido girando con velocidad angular
alrededor de un eje fijo. Cada partcula del cuerpo posee
una cierta cantidad de energa cintica. La figura muestra
una partcula al azar, de masa m
i
que gira con radio r
i
, con
velocidad angular y velocidad lineal v
i
. Por lo tanto su
energa cintica es:
Ec = m
i
v
i
2
= m
i
r
i
2

2

La energa cintica total del cuerpo ser la
sumatoria de las Ec
i
de cada una de las partculas:
Ec = Ec
i
=
1
2

2
( mi ri
2
) = [
2

Si comparamos la expresin de la Ec obtenida para la rotacin, con la Ec de la
traslacin, vemos que J es en la rotacin lo que m es en la traslacin. Pero la analoga
entre J y m no va ms all de esto, porque mientras que m no depende de su localizacin
en el cuerpo, J s depende, e incluso depende de la posicin del eje de rotacin. Para
ilustrar lo expresado, consideremos el siguiente ejemplo:

Ejemplo 1.
La figura muestra una haltera; consiste en dos esferas situadas en los extremos
de una barra rgida ligera de 1 metro de largo.
Cada esfera tiene una masa de 5 kg.
Suponiendo que las esferas se comportan
como partculas, calcular el momento de
inercia de la haltera para los siguientes casos:
a) el eje de rotacin es perpendicular a la
pieza y pasa por su punto medio C. b) el eje es perpendicular a la pieza y pasa por una
de las esferas.
Solucin:
Comentarios sobre el enunciado: 1) Cuando dice barra ligera, significa que el
momento de inercia J correspondiente a la barra es nulo o que puede despreciarse. 2)
Cuando dice que se suponga que las esferas se comportan como partculas, lo hace
porque hasta este momento slo hemos presentado la expresin de J correspondiente a
la partcula.

a) J
(C)
= m
i
r
i
2
= m
A
r
A
2
+ m
B
r
B
2
= 5(0,50)
2
+ 5(0,50)
2
= 2,5 kgm
2


b) J
A
= J
B
= m
A
r
A
2
+ m
B
r
B
2
Uno de los trminos se anula y queda el otro:
J = 5(1)
2
= 5 kgm
2


Los resultados muestras que con slo desplazar el eje de rotacin a otro lugar, el
valor del momento de inercia de una pieza, cambia. Al respecto, J alcanza su valor
mnimo cuando el eje pasa por el centro de masa de la pieza.

Clculo de momentos de inercia.
Si el cuerpo tiene una distribucin continua de masa, la expresin de J deber
obtenerse mediante los procedimientos del clculo integral. Dado que es J = m
i
r
i
2
,
deber suponerse al cuerpo dividido en elementos infinitesimales de masa m y r ser la

v
i
= r
i

r
i

m
i

A B
1 m
C
distancia desde cada uno de estos elementos hasta el eje de rotacin. El producto m
i
r
i
2

nos dar el J
i
. Finalmente ser
J = J
i
.
Matemticamente se plantea as:
J = l i m
m0
Am

2
= r
2
Jm
Si el cuerpo presenta una geometra sencilla, la integral
indicada puede resolverse sin mayores dificultades. El cuadro
siguiente resume las expresiones de J para los cuerpos
geomtricos ms comunes.

FRMULAS DE MOMENTO DE INERCIA J



Cilindro hueco o
macizo con eje de
rotacin
coincidente con el
eje de simetra.


2
2
2
1

2
1
R R M J

Si es macizo:
J = M R
2



Si es de paredes muy delgadas:
J = M R
2
.

1


2



3



Barra delgada
uniforme y eje de
rotacin
perpendicular a
ella

) 3 3 (
3
1
2 2
h h l l M J
Si el eje pasa por el centro:

2

12
1
l M J
Si el eje pasa por un extremo:

2

3
1
l M J

4


5


6




Esfera uniforme
con eje de rotacin
coincidente con un
dimetro


Si es maciza:


2

5
2
R M J

Si es hueca y de pared delgada:


2

3
2
R M J


7





8


Radio de giro (k).
RADIO DE GIRO es la distancia respecto del eje de rotacin, donde debera ser ubicada en
forma puntual la totalidad de la masa de un cuerpo dado, para que su momento de inercia posea
el mismo valor que el que tiene el cuerpo respecto de dicho eje.
m
i

r
i

eje
R
h l - h
l
R
1
R
2

Si la masa M del cuerpo estuviera
realmente a esa distancia k, el J tendra la
expresin que le corresponde a una masa puntual
M situada a la distancia k del eje, o sea:
J = Mk
2
(3) Luego,

k = _
]
M
(4)
J y k son dos conceptos equivalentes, y las
frmulas (3) y (4) muestran cmo puede calcularse uno a partir del otro.

Ejemplo 2.
Cunto vale el radio de giro de una barra delgada de masa m y longitud l
respecto de un eje perpendicular trazado por su centro?
Solucin:
Buscamos en la tabla de momentos de inercia, la frmula de J que corresponde
al cuerpo y eje que describe el enunciado; dicha frmula es la n 5: J =
1
12
m l
2

Ingresando esta expresin en la (4) y operando, resulta:
k =
I
12


Teorema de los ejes paralelos.
Hemos visto que un cuerpo no tiene un J nico, sino infinitos J dado que puede
girar alrededor de infinitos ejes diferentes. Sin embargo hay una relacin simple entre el
Jcm (para un eje baricntrico cualquiera) y el J
(P)
(para cualquier otro eje paralelo al
baricntrico original, pero desplazado una distancia d.)
Esta relacin es conocida con el nombre de teorema de los ejes paralelos o
teorema de Steiner. La demostracin de este teorema es la siguiente:

J
(II)
= r
1
2
Jm
Aplicando el Teorema del coseno (ver figura):
r
1
2
= r
2
+ J
2
2 r J cos0
Reemplazando:
J
(II)
=
( r
2
+ J
2
2 r J cos0) Jm
Desarrollando:
J
(II)
= r
2
Jm+ J
2
Jm 2J r cos0 Jm
En el 2 miembro, el primer trmino representa a
Jcm; el segundo a Md
2
, y el tercero es nulo por
representar el momento esttico del cuerpo con
respecto a su eje baricntrico. En consecuencia
queda:
J = Jcm + Md
2
(5)
Esta expresin nos dice que el J de un cuerpo es mnimo cuando se lo calcula
para un eje baricntrico, frente al J que tendra en cualquier otro eje paralelo a l.

Ejemplo 3.
Deducir la expresin de J
(P)
para una esfera maciza de masa M y radio r, siendo
P un punto de su periferia.
M
k
M
J iguales
I II
d
r
1

0
c.m.
r
dm
M
Solucin:
Buscamos la expresin de Jcm en la tabla de momentos de
inercia. Corresponde la N 7: Jcm =
2
5
H r
2

Aplicamos el teorema de Steiner: J = Jcm + Md
2
donde
d es en nuestro caso, igual a r.
Reemplazando: J
(P)
=
2
5
H r
2
+ Mr
2

Operando: J
(P)
=
7
5
H r
2


El trabajo en la rotacin. (Trabajo de una torca).
Cuando pedaleamos en una bicicleta, aplicamos
fuerzas a un cuerpo en rotacin y efectuamos trabajo
sobre l. Para ese caso, (y el de muchos otros ejemplos
similares de la vida real) podemos expresar el trabajo en
trminos de la torca y el desplazamiento angular.
Supongamos una fuerza tangencial ejercida por
un nio que corre empujando un carrusel sencillo. En un
tiempo dt la rueda gira un ngulo d0. El trabajo dW
efectuado por F
tung.

es:
dW = F
tang
ds
ds es la distancia recorrida por un punto del borde, y representa el recorrido realizado
por el punto de aplicacin de F
tung.


dW = F
tang
Rd0 = Izd0
El t r abajo t ot al efect uado por l a t or ca dur ante el despl azami ent o angul ar
desde 01 hast a 02 es:
W = I
z
J0
0
2
0
1
Uni dad S.I.: Nm = joul e
Est a expr esi n guar da anal oga con l a de l a t r asl aci n: W = F
x
Jx
x
2
x
1
.
Si l a fuer za apl i cada t uvi er a una component e axi al ( par al el a al ej e de
r ot aci n) o r adi al (haci a el ej e o al ejndose de l ) , di cha component e no har a
t r abajo, por que el despl azami ent o del punt o de apl i caci n sl o t i ene component e
t angenci al . De maner a que l a ecuaci n de ar r iba es vl i da par a cual qui er fuerza,
i ndependi ent ement e de sus component es.
Par a el caso par t i cul ar en que l a t or ca per manezca const ant e, ser :

W = Iz J0
0
2
0
1
= I
z
( 0
2
0
1
) = 0 (6) Tr abajo de una t or ca
const ant e.

La potencia en la rotacin.
Par a obt ener l a expr esi n de l a pot enci a en l a r ot aci n, bast a con di vi di r l a
ecuaci n ( 6) el t i empo:

w
At
= I
A0
At
P = I ( 7)
Expr esi n anl oga a l a P = Fv de l a t r asl aci n.

Frmula fundamental de la dinmica de la rotacin.
c.m. P
F
tung.


Si el cuerpo es rgido, no hay movimiento interno de las partculas y por lo tanto
no puede haber disipacin interna de energa; por lo tanto podemos relacionar la
potencia P con la rapidez con que aumenta la energa cintica, a la cual podemos
expresar como
d
dt
[
1
2
[
2

Como J = constante, por ser el cuerpo rgido y tener el eje fijo, puede ponerse:

d
dt
[
1
2
[
2
=
1
2
[
d
dt

2
= [
do
dt
= [ o (8)
Igualando los segundos miembros de las (7) y (8):
= J = J (9)
La (9) es la frmula fundamental de la dinmica de la rotacin; relaciona la
causa, representada por la torca con el efecto, representado por la aceleracin angular
. La (9) tiene su anlogo en F = ma de la dinmica lineal.

Ejercicio 2.

Una polea de M = 2,5 kg y R = 0,20 m, est montada
sobre un eje sin friccin y tiene enrollada sobre su borde una
cuerda ligera. Si se tira del extremo de la cuerda, con una
fuerza constante hacia debajo de T = 5N, calcular a) la
aceleracin angular de la rueda; b) la aceleracin tangencial
a de un punto de su borde. C) Si en vez de tirar con una fuerza
T, se atara al extremo de la cuerda una pesa de 5 N, cules
seran ahora los valores de y de a?
Solucin:
Lo primero que debemos hacer es identificar el tipo de movimiento que
tenemos; la polea hace una rotacin. Por lo tanto, nuestro punto de partida ser la (9).

a) = J donde J = MR
2
y = TR.
Reemplazando en la frmula fundamental y despejando :
=
21
MR
=
25
2,50,20
= 20
1
s
2


b) a = R = 200,20 = 4,0
m
s
2


c) El agregar una masa al extremo de la cuerda, implica que ahora habr dos
cuerpos movindose: la polea (con rotacin) y la masa (con traslacin). Para resolver el
problema deberemos ahora plantear las ecuaciones de dinmica de los dos cuerpos.

-Ecuacin para la masa colgada: Ella realiza traslacin, luego:
mg T = ma Luego: T = mg ma (10)

-Ecuacin para la polea. Ella realiza rotacin. La ecuacin deducida
en a) nos sirve, slo que la fuerza sobre la polea ya no es T, sino
T:
=
21i
M
con =
u
R
Luego: T = Ma (11)
Igualando los segundos miembros de las (10) y (11):

mg ma =
1
2
Ho donde m =
P
g
=
5
9,8
= 0,51 kg
T
M
R
a
T
mg
m
a =
2mg
M+2m
=
20,519,8
2,5+20,51
= 2,85
m
s
2


=
u
R
=
2,85
0,20
= 14,3
1
s
2


Ntese que las aceleraciones son menores ahora, a pesar que el cuerpo
suspendido pesaba lo mismo que el esfuerzo T. Esto se debe a que T < T y con ello,
tambin es menor la torca .






























































14
Rodadura

Hemos estudiado el movimiento de rotacin pura, o sea de eje fijo. En esta clase
trataremos un movimiento ms general, donde el cuerpo rgido realiza simultneamente
movimientos de rotacin y de traslacin. Sin embargo nos limitaremos al caso especial
en el cual ambos movimientos, rotacin y traslacin no ocurren en forma independiente,
sino que se encuentran ligados fsicamente, de manera que hay un vnculo matemtico
que los relaciona.
Supongamos un cilindro que rueda por un plano; hay una rotacin al girar el
cilindro alrededor de su eje geomtrico y hay una traslacin de dicho eje, al desplazarse
paralelamente a s mismo. El contacto entre el cilindro y el plano tiene lugar a lo largo
de una recta generatriz. Si el cilindro no resbala ambos movimientos estn relacionados.

Se llama RODADURA al movimiento combinado de rotacin y traslacin que realiza
un cuerpo (cilindro, aro, esfera) cuando rueda sin resbalar por un plano.

Cul es la relacin que vincula en la rodadura, a la rotacin con la traslacin?

r
L = 2r
La figura muestra un cilindro de radio r que rueda sin resbalar por un plano
horizontal. Hacemos una marca de referencia sobre la tapa del cilindro y hacemos que
ella se ponga en contacto con el plano. Si a partir de aqu le hacemos dar una vuelta
completa al cilindro (lo que ocurrir cuando la marca vuelva por primera vez al piso), la
longitud L que habr avanzado el cilindro por el plano estar relacionada con su radio
por:
L = 2r (1) CONDICIN DE RODADURA

Si el cilindro hubiera resbalado, podra haber rotado ms de lo que se traslada o
bien al revs; en ninguno de estos casos la igualdad de arriba se hubiera cumplido. La
igualdad entre L y 2r nos garantiza que el cilindro no ha resbalado, por lo que habr
realizado un movimiento de rodadura.
La condicin de rodadura tambin puede expresarse como un vnculo entre
velocidades, dividiendo por el tiempo t a la ecuacin (1):

L
t
=
2n
t
r v = r (2) CONDICIN DE RODADURA
Y hay una tercera manera de expresar la condicin de rodadura, como un vnculo
entre aceleraciones. Si se repite el procedimiento con la (2), resultar:
a = r (3) CONDICIN DE RODADURA

Hemos dicho que el contacto entre el cilindro y el plano tiene lugar a lo largo de
una recta generatriz a la que distinguiremos de las dems generatrices, llamndola la
generatriz de contacto. Pero mientras el cilindro rueda, hay una continua sustitucin de
generatrices de contacto por las que le siguen. Una generatriz de contacto lo es por un
instante fugaz. Pero ntese que si bien el cilindro est en movimiento, la generatriz de
contacto est en reposo: si ella se moviera, el cilindro resbalara. Para mostrar esto,
dibujaremos los perfiles de velocidad por separado, para la rotacin y para la traslacin,
para luego sumarlos y obtener el perfil de velocidades definitivo. All vemos que en el
punto Q (generatriz de contacto) la velocidad es efectivamente nula.
La observacin de los perfiles de velocidad nos permite tener otra mirada sobre
el movimiento de rodadura. Cuando nicamente hay rotacin, la velocidad tangencial en
el eje es nula. Si observamos el perfil de velocidades definitivo (de la derecha), vemos
que velocidad nula hay en el punto Q. Luego es vlido considerar que el movimiento de
rodadura es un movimiento de rotacin pura alrededor del punto Q. Por tal motivo, a la
generatriz de contacto Q se la llama tambin eje instantneo de rotacin. Esta nueva
Si solo consideramos
rotacin
Si solo consideramos
traslacin
P v
P

v
Q
Q
v
P

v
Q

v
cm

v
P
= 2v
cm

P
Q v
Q
=0
v
cm

manera de interpretar la rodadura es vlida para un instante solamente, ya que la
generatriz de contacto se recambia constantemente.

El movimiento de rodadura puede interpretarse de dos maneras: a) como una
combinacin de traslacin del centro de masa mas una rotacin alrededor del eje
baricntrico; b) como una rotacin pura alrededor de un eje que pasa por el punto de
contacto entre el cuerpo y la superficie sobre la cual rueda.

Como una forma de verificacin de lo que acabamos de exponer, calcularemos
la energa cintica de un cilindro que hace rodadura; lo haremos de las dos maneras,
pero deberemos llegar a la misma conclusin.

1 FORMA El cilindro combina una traslacin con una rotacin alrededor del centro
de masa.
Ec
TOTAL
= Ec
tras
+ Ec
rotac
= mv
2
+ J
cm

2

= mv
2
+ [
1
/
2
mr
2
]

2
= mv
2
+ mv
2
= mv
2


2 FORMA El cilindro hace rotacin pura alrededor del punto Q
Previamente, aplicando el teorema de los ejes paralelos, deduciremos la
expresin de J
(Q)
. J
(Q)
= J
cm
+ md
2
donde d = r
J
(Q)
=
1
/
2
mr
2
+ mr
2
=
3
/
2
mr
2

Ec
TOTAL
= Ec
rotac
= J
(Q)

2
= [
3
/
2
mr
2
]

2
=
3
4
m :
2


Para aprender ms sobre el movimiento de rodadura nos plantearemos el
siguiente ejercicio:

Ejercicio 1.
Un cilindro macizo, de
masa M y radio R, desciende sin
resbalar por un plano inclinado.
Deducir la expresin de la
velocidad de su centro de masa,
para cuando el cilindro llega a la
base del plano.
Solucin:
Este problema puede
resolverse de 2 maneras: utilizando mtodos energticos o por dinmica. Lo
resolveremos por las dos formas. Por otro lado, como todo problema de rodadura puede
considerarse como una combinacin de rotacin alrededor del c.m. ms una traslacin,
o bien como una rotacin pura. Elegiremos la combinacin de rotacin ms traslacin.
1 Solucin por energa:
Se utiliza el principio de la conservacin de la energa mecnica y supondremos
que en el estado inicial el cilindro parte del reposo. El estado final ser el momento en
que el cilindro llegue a la base del plano inclinado.
Em
1
= Em
2

Mgh = J
cm

2
+ Mv
2

Se reemplaza J
cm
por su expresin; para el cilindro macizo es MR
2
. Por la
condicin de rodadura, se reemplaza =

R
. Nos queda:
R
M
h
0
Mgh =
1
2
[
1
2
H R
2
[

2
+
1
2
H :
2
= [
1
4
+
1
2
H :
2


Despejando, :
2
=
4
3
g : = _
4
3
g
Resulta interesante comparar este resultado con el que se hubiera obtenido si el cilindro
hubiese resbalado sin rozamiento por el plano inclinado; en tal caso la velocidad del
centro de masa habra sido
v = 2 g
Por lo tanto, la velocidad del cilindro que rueda, es menor que la del cilindro que
resbala, debido a que en el primero, parte de la energa potencial ha sido transformada
en energa cintica de rotacin, dejando una cantidad menor para la parte traslacional de
la energa cintica.. Aunque el primer cilindro tarda ms que el segundo en llegar abajo,
ambos llegan con la misma energa cintica.

2 Solucin por dinmica:
Volvemos a resolver el problema, ahora usando mtodos dinmicos.
Comenzamos haciendo un diagrama de cuerpo libre.

Hay que plantear una ecuacin por
cada movimiento y una condicin de
rodadura:

Ec. de traslacin:
Mgsen 0 F
R
= Ma (4)
Ec. de rotacin:
= J
cm
La nica fuerza
cuya recta de accin NO pasa por O y
que puede producir torca es F
R
. Luego: F
R
R = J
cm

F
R
R =
1
2
H R
2
o (5)
Ec. de condicin de rodadura: =
u
R
(6)

Las (4), (5) y (6) forman un sistema de 3 ecuaciones con 3 incgnitas. Reemplazando en
la (5) por la (6), y despejando a continuacin F
R
, se obtiene: F
R
= Ma (7)

Llevando la (7) a la (4): Mgsen 0 Ma = Ma

Despejando, a =
2
3
g scn 0 (8)

La (8) nos muestra que un cilindro que rueda adquiere una aceleracin menor
que otro que resbala sin rotar para el cual hubiera sido a = gsen 0. Por otra parte, la (8)
nos muestra que a = constante, por lo que el cilindro desciende por el plano inclinado
realizando un MRUV.
Aqu nos deja la dinmica. Si deseamos llegar a la frmula de la velocidad,
deberemos agregar un paso de cinemtica. De la frmula del MRUV: v
2
v
0
2
= 2al
donde v
0
= 0 y l (longitud recorrida sobre el plano inclinado) =
h
scn 0
queda:
v
2
= 2[
2
3
g scn 0] [
h
scn 0
] =
4
3
g
h
0
N
O F
Roz

Mgcos 0
Mgsen 0
y
x
Finalmente: : = _
4
3
g
que es el mismo resultado obtenido cuando se resolvi el problema por energa.

El primer mtodo es ms simple y directo, pero cuando se desean conocer
fuerzas tales como N y F
R
, debe emplearse el segundo mtodo. Con este ltimo puede
hallarse la expresin de la fuerza de rozamiento mnima necesaria para que el cilindro
no resbale:
F
R mnima
= Ma = M[
2
3
g scn 0] =
1
3
H g scn 0
A veces a la expresin Mgsen 0 se la suele llamar fuerza activa, por ser la que
motoriza el movimiento del cilindro. Utilizando esta denominacin, llegamos a esta
interesante conclusin:
F
R mnima
=
1
3
F
activa

Tener en cuenta que todas las deducciones realizadas, han sido hechas para un
cilindro macizo. Si el cuerpo que realiza la rodadura fuera otro, las expresiones de
velocidad, aceleracin, fuerza de rozamiento mnima, etc seran distintas, simplemente
porque J sera diferente.

Observacin: Por qu no se tom en cuenta la eventual energa disipada por la fuerza
de rozamiento, cuando se resolvi el problema por energa? Simplemente porque no
existe tal energa disipada. Ocurre que cuando el cilindro rueda por el plano inclinado,
no se realiza ningn trabajo, por lo que tampoco se desarrolla calor, como puede
comprobarse experimentalmente. La fuerza de rozamiento existe, y es gracias a ella que
el cilindro hace rodadura y no resbala. Pero precisamente porque no resbala, as como la
generatriz de contacto no se mueve, la fuerza de rozamiento no se traslada, requisito
necesario para que pueda hacer trabajo.
Como la generatriz de contacto cambia continuamente por otra, cada una plantea
instantneamente en su momento, a la F
R
, pero esta F
R
es cambiada por otra F
R
con la
llegada de la siguiente generatriz de contacto; ninguna de estas F
R
traslada su punto de
aplicacin y por lo tanto no realizan trabajo.
Por otra parte, si la fuerza de rozamiento no se traslada, podemos decir que en el
movimiento de rodadura siempre tenemos una fuerza de rozamiento esttico, y si
debiera calcularse con la expresin F
R
= N, habr que usar siempre es
s
.

Problema.



F
F
CASO a) CASO b)
Las figuras muestran dos rodillos iguales (cilindros macizos) de masa M a los
que se les aplica la fuerza F de modo que ellos ruedan sin resbalar, experimentando sus
centros de masa, un MRUA. En el caso a) F

es aplicada en el eje, mientras que en el


caso b) se aplica en la periferia del rodillo, mediante una cuerda enrollada alrededor de
l. En ambos casos F

es horizontal y tiene sentido hacia la derecha. Determinar cul es


el sentido de la fuerza de friccin en cada caso.

Solucin:
Para poder plantear las ecuaciones de movimiento, tentativamente le asignamos
un sentido a la fuerza de friccin. Si el sentido atribuido no fuera el correcto, al resolver
las ecuaciones, F
oz

tendr signo negativo. El sentido que le supondremos es F


oz

hacia
la izquierda.

CASO a) Ecuaciones; De traslacin: Fx ] F F
roz
= Ma (1)
De rotacin: ] F
roz
r = J
cm
(2)
Condicin de rodadura a = r (3)

De la (2): F
roz
r = [
1
/
2
M r
2
]
u

F
roz
= Ma (4)
Reemp. 4) en la (1) y despejando F: F =
3
2
H o (5)
Despejando a de la (5): a =
2
3

P
M
(6)
Finalmente, reemp. La (6) en la (4):
F
roz
=
1
3
F (7)
La (7) nos muestra que F
roz
est bien dibujada y que tiene sentido opuesto al de
F

.

CASO b) Ecuaciones;
De traslacin: : Fx ] F F
roz
= Ma (1)
De rotacin: ] Fr + F
roz
r = J
cm
(2)
Condicin de rodadura a = r (3)

De la (2): Fr + F
roz
r = [
1
/
2
M r
2
]
u

F + F
roz
= Ma (4)
Resolviendo el sistema de dos ecuaciones con dos incgnitas
formado por las (1) y (4):
F F
roz
= Ma
F + F
roz
= Ma
__________ ___________
Sumando: 2 F =
3
2
H o Luego: a =
4
3

P
M
(5)

Reemplazando la (5) en la (4): F + F
roz
= M [
4
3

P
M
] F + F
roz
=
2
3
F
F
roz
= -
1
3
F (6)
La (6) nos muestra que F
roz
est mal dibujada y que su verdadero sentido
coincide con el sentido de F.




F


r
F
oz


O
r
F
oz


O
F























15
Esttica

ESTTICA es la parte de la Fsica que se ocupa del equilibrio de los sistemas de
fuerzas.

Cuando sobre un cuerpo acta su sistema de fuerzas, el efecto neto sobre el
cuerpo est determinado por la sumatoria de todas ellas; el resultado de la sumatoria se
llama fuerza resultante o fuerza neta. La simbolizamos con R

. O sea:

F
i

= R


Se trata de una sumatoria vectorial. Si R

no es nula, el cuerpo no est en


equilibrio. Para lograr el equilibrio, habr que agregar otra fuerza, capaz de contrarrestar
a R

. Dicha fuerza se llama equilibrante y la simbolizaremos


con E

.
R

y E

son fuerzas de igual mdulo y direccin, pero


con sentidos opuestos; o sea que difieren angularmente en
180.
Los sistemas de fuerzas pueden ser unidimensionales,
bidimensionales o tridimensionales. Estos ltimos no sern estudiados, por lo que nos
ocuparemos directamente de los bidimensionales, tambin llamados sistemas de fuerzas
coplanares, por el hecho de compartir el mismo plano.
R


Los sistemas coplanares de fuerzas pueden clasificarse en dos: concurrentes y no
concurrentes. En los primeros, todas las fuerzas estn aplicadas en un mismo punto; en
los segundos, no.

Sistemas de fuerzas concurrentes.
Para que un sistema de fuerzas sea estrictamente concurrente, deberemos
introducir el concepto de punto material, definindolo como un cuerpo sin dimensiones
(como el punto geomtrico), pero no obstante con masa. De esta manera, todas las
fuerzas que se apliquen, debern ser forzosamente concurrentes.
El primer paso para resolver un sistema de fuerzas concurrentes consiste en
adoptar un sistema de referencia con dos ejes cartesianos con origen en el punto
material.
En un segundo pasa se proyecta cada una de las fuerzas del sistema, sobre cada
uno de los ejes cartesianos, planteando para cada fuerza:

F
ix
= F
i
cos 0
i
F
iy
= F
i
sen 0
i


En un tercer paso se calculan las componentes de la fuerza resultante, haciendo:

F
ix
= R
x
F
iy
= R
y

Si el sistema estuviera equilibrado, tales sumatorias resultaran nulas; caso
contrario, pasamos al prximo paso.
En un cuarto paso se calcula el mdulo y la direccin (ngulo) de la fuerza
resultante, haciendo:
R = R
x
2
+ R

2
0
R
= orc tg
R
j
R
x


Condiciones generales de equilibrio para un sistema de
fuerzas concurrentes.
Para que el sistema est en equilibrio, debe ser R = 0. Por lo tanto las
condiciones matemticas necesarias y suficientes son:
F
ix
= 0 F
iy
= 0
Ejemplo 1.
Hallar el mdulo de F
2
y su ngulo 0
2
, necesarios para
equilibrar al sistema de fuerzas constituido por F
1
y F
3
de la
figura.
DATOS: F
1
= 500 N 0
1
= 120 F
3
= 1435 N 0
3
= 270
Solucin:
F
ix
= 0 F
1
cos 0
1
+ F
2
cos 0
2
+ F
3
cos 0
3
= 0 (1)
F
iy
= 0 F
1
sen 0
1
+ F
2
sen 0
2
+ F
3
sen 0
3
= 0 (2)
Teniendo en cuenta que en la (1) F
3
cos 0
3
= 0 por ser 0
3

= 270, ponemos:
F
2
cos 0
2
= - F
1
cos 0
1
(3)
F
2
sen 0
2
= - F
1
sen 0
1
- F
3
sen 0
3
(4)
Calculando los segundos miembros y dividiendo miembro a miembro la (4) con
la (3) queda:
tg 0
2
=
1000 N
250 N
= 4 0
2
= 76 Finalmente, F
2
= 1042 N

F
2

F
1

F
3

y
x
Descomponer una fuerza dada en dos direcciones
dadas.
Dentro de las fuerzas concurrentes, hay un problema tpico, que es el de
descomponer una fuerza dada en dos direcciones dadas. Puede resolverse con el mtodo
de las proyecciones empleado en el ejemplo 1, pero tambin puede utilizarse un mtodo
alternativo: el del tringulo vectorial. Explicaremos este mtodo a travs de la
resolucin de un ejemplo:

Ejemplo 2.
Un objeto que pesa 3000 N cuelga del techo y la pared, mediante las cuerdas OA
y OB, como se muestra en la figura. Determinar el valor de los esfuerzos en cada
cuerda.
Solucin:

Para aplicar el mtodo del tringulo vectorial, seguir los siguientes pasos:
1- Respetando las direcciones de las tres fuerzas (la P y los esfuerzos en las dos
cuerdas) construir un tringulo. Como el sistema est en equilibrio, la resultante es nula
y la figura del tringulo debe cerrarse por s misma.

2- A partir de los ngulos conocidos, identificar los valores de los ngulos interiores del
tringulo.

3- Finalmente, resolver el tringulo por trigonometra. En nuestro ejemplo, se conoce un
lado: P = 3000 N y los tres ngulos. Con esta informacin, puede emplearse el teorema
del seno:

3000 N
scn 105
=
1
AO
scn 30
=
1
BO
scn 45


I
A0
= 3000 N
scn 30
scn 105
= 1553 N
I
B0
= 3000 N
scn 45
scn 105
= 2196 N

Sistemas de fuerzas no concurrentes.
Cuando las fuerzas del sistema tienen puntos de aplicacin diferentes sobre el
cuerpo, se dice que son no concurrentes. Debemos abandonar la hiptesis del punto
material usada en el caso anterior. Ahora el cuerpo es extenso, y formularemos para l
otra hiptesis: la del cuerpo rgido. Esto significa que se mantendr indeformable, que
conservar su geometra intacta, no importa cunto de grandes sean las fuerzas
aplicadas.

O
A
B
60
45
T
AO

T
BO

P
P
45
105
30
Cupla o par de fuerzas.
Un sistema de dos fuerzas del mismo mdulo, que actan sobre rectas paralelas
y tienen sentidos opuestos, forman una cupla. La cupla es un ejemplo de un sistema de
fuerzas no concurrentes; obsrvese que la sumatoria de las proyecciones de sus fuerzas
sobre las direcciones cartesianas nos da: Rx = 0; Ry = 0. Se cumplen las condiciones de
equilibrio establecidas; no obstante el cuerpo que est sometido a la accin de una cupla
no se encuentra en equilibrio, sino que rota. Ocurre que toda cupla genera una torca
pura, cuyo valor podemos obtener aplicando el
teorema de Varignon. Para un punto cualquiera,
como el O:
I
R
( 0)
= I

( 0)

= F (d + a) - Fa
Como F

= |F

| : I
R
( 0)
= Fd + Fa - Fa
Finalmente: I
R
( 0)
= Fd

Si hay varias cuplas actuando en el plano, la torca resultante se obtiene mediante
una simple suma algebraica, donde se tomarn como positivas a las que provoquen un
giro antihorario y como negativas a las que den un giro horario; esto es por convencin.


Condiciones generales de equilibrio para un sistema de
fuerzas no concurrentes.
El ejemplo de la cupla y su efecto, pone en evidencia que las condiciones de
equilibrio estudiadas para el caso de fuerzas concurrentes ( F
ix
= 0 F
iy
= 0)
ahora no son suficientes, si bien siguen siendo necesarias. Deber agregarse una
condicin que garantice la no existencia de torcas. De modo que las condiciones sern:
F
ix
= 0
F
iy
= 0

i
(O)
= 0
Ejemplo 3.
Una escalera uniforme de 6 m de largo y 400 N de peso, est apoyada contra una
pared vertical sin rozamiento, con su extremo inferior a 3,6 m del pie de la pared.
Calcular los mdulos y direcciones de las fuerzas ejercidas contra la escalera en sus
puntos de apoyo.
Solucin:
Algunas consideraciones previas sobre el enunciado del problema. Qu
significa uniforme? Que la escalera es igual de punta a punta, del mismo material y con
la misma geometra; todos los peldaos son de la misma longitud; quedan descartadas
las escaleras que se angostan con la altura. Sabiendo eso, sabemos dnde est su centro
de masa y con ello dnde ubicar la fuerza peso de la escalera.
Ms adelante, el enunciado dice que entre la escalera y la pared no hay
rozamiento. Qu informacin til sacamos de esto? Si hubiera rozamiento, la Froz
tendra direccin vertical en el punto A del dibujo. Al no haberlo, estamos sabiendo que
la reaccin en A tendr la direccin horizontal.
Clculo de la distancia AO. Aplicando el teorema de Pitgoras:
AO = 6
2
3,6
2
= 4,8 m

F


d
a
O
I
R



El primer paso es construir el diagrama de cuerpo libre, representando todas las
fuerzas actuantes sobre la escalera. La reaccin en A ya se sabe que es horizontal: la
llamamos H
A
. Como de la reaccin en B no se conoce su direccin, se dibujan sus
componentes horizontal y vertical: H
B
y V
B
. Los sentidos de las reacciones se ponen al
azar; si una vez calculadas, alguna tuviera un valor negativo, significar que el
verdadero sentido de esa reaccin es contrario al asignado al comienzo.
Segundo paso: sabiendo que la escalera se encuentra en equilibrio, aplicamos las
ecuaciones que surgen de las condiciones de equilibrio y formaremos un sistema de 3
ecuaciones con 3 incgnitas, cuya resolucin resuelve el problema.

F
ix
= 0 H
A
H
B
= 0
F
iy
= 0 V
B
400 N = 0

i
(B)
= 0 4,8mH
A
1,8m400N = 0

Resolviendo, H
A
= H
B
= 150 N; V
B
= 400 N. Los sentidos asignados son los
correctos.
Tercer paso: Para calcular la reaccin en B, aplicamos Pitgoras y trigonometra:
R
B
= E
B
2
+ I
B
2
= 150
2
+ 400
2
= 427 N
0
B
= orc tg
I
B
E
B
= src tg
400
150
0
B
= 6930

Vnculos.

VNCULO es toda condicin geomtrica que limita la movilidad de un cuerpo o de un
conjunto de puntos materiales.

El ms sencillo de los vnculos es el que muestra la figura,
donde el punto M slo podr moverse sobre una superficie esfrica
de radio MN, ya que la longitud MN es invariable. Se dice que los
puntos M y N estn unidos por el vnculo de la rigidez. Esta barra
constituye un vnculo para el punto M; se denomina biela.

Grados de libertad.
Supongamos tener un punto material A en el plano y un sistema de ejes
cartesiano x;y. Las coordenadas del punto A son (x
A
;y
A
). Si el punto A cambiara de
A
O
B
4,8 m
3,6 m
3 m
3 m
P
A
H
A

P
B H
B

V
B

M N
posicin, para poder ubicarlo nuevamente deberamos dar dos nuevas coordenadas
(x
A
;y
A
). De manera que un punto material en un plano posee dos coordenadas libres.

El nmero de coordenadas libres que tenga un punto material (o un sistema) se
denomina grado de libertad.


El concepto de grado de libertad puede darse tambin de otra manera,
relacionndolo con cada uno de los movimientos elementales que puede hacer el punto
material, la chapa o el cuerpo. Y los movimientos elementales son las rotaciones y las
traslaciones. As, un cuerpo en el espacio, tiene 6 grados de libertad, porque puede hacer
3 traslaciones (una en la direccin de cada eje cartesiano) y 3 rotaciones (tambin una
en torno de cada eje cartesiano).
Llamaremos chapa a un cuerpo que slo posee dos dimensiones, de modo que
siempre estar contenida en un plano. Por lo tanto, una chapa tiene tres grados de
libertad, que se corresponden con 2 traslaciones (una en la direccin de cada eje) y una
rotacin (alrededor del origen, o si se prefiere, alrededor del eje faltante, z).

Tipos de vnculos.
Ahora podemos dar una mejor definicin de vnculo:

VNCULO es todo dispositivo que restringe uno o ms grados de libertad.

Aunque hay otros, nos limitaremos a presentar slo dos tipos de vnculos: el
apoyo fijo y el apoyo mvil.

APOYO FIJO: Suele llamrselo tambin articulacin fija a tierra, simplemente
articulacin o tambin vnculo de segunda especie.
Supongamos que tenemos una chapa y la fijamos por uno de sus puntos: o sea
que si ese punto es A, sus coordenadas (x
A
;y
A
) quedan fijadas. Cualquier otro punto de
la chapa slo podr moverse sobre arcos de circunferencia con centro en A; o sea que la
chapa slo puede rotar; ha perdido la posibilidad de hacer sus dos traslaciones; ha
perdido 2 grados de libertad. Esto es lo que sucede cuando se aplica un vnculo de este
tipo.

Un APOYO FIJO restringe dos grados de libertad.

APOYO MVIL: Suele llamrselo tambin biela o vnculo de primera especie. Este
tipo de vnculo restringe una traslacin, pudiendo una chapa realizar una rotacin y una
traslacin.

Un APOYO MVIL restringe un grado de libertad.

Para los dos tipos de vnculos presentados, se resumen en un cuadro en la
prxima pgina, sus formas constructivas, los smbolos grficos que los representan, y
las caractersticas que posee la reaccin que cada uno de estos vnculos trasmite a la
chapa.


VNCULO Forma constructiva Smbolo Reaccin

APOYO
FIJO





Tiene cualquier direccin. Se
representan sus componentes
horizontal y vertical.


APOYO
MVIL






Tiene la direccin de la
normal al vnculo.

Ejemplo 4.
Calcular los valores de las componentes de las reacciones en los vnculos, para
la viga homognea, uniforme de 2 t de peso, simplemente apoyada, y cargada como
muestra la figura:

Solucin:
1 Paso: Hacer el D.C.L. No olvidar agregar el peso propio de la barra, que no fue
indicado en el dibujo, pero que se da en el enunciado.
2 Paso: Reemplazar los vnculos por las componentes de reaccin que ellos
producen.
3 Paso Expresar los ngulos de acuerdo con la convencin angular habitual.
Queda:
4 Paso: Clculo de las componentes de las fuerzas:
F
1x
= F
1
cos 240 = -1,5 t F
2x
= F
2
cos 330 = +3,46 t
F
1y
= F
1
sen 240 = -2,6 t F
2y
= F
2
sen 330 = -2,0 t

Finalmente el esquema queda as:

60
F
1
= 3t
A
B
F
2
= 4t
150
2 m 4 m 2 m
240
F
1
= 3t
F
2
= 4t
330
2 m 2 m 2 m
x
2 m
P = 2t
V
A

V
B

H
A

5 Paso: Planteo de las condiciones de equilibrio:

F
ix
= 0 H
A
+ 3,46 t 1,5 t = 0
F
iy
= 0 V
A
+ V
B
2,6 t 2 t 2 t = 0

i
(A)
= 0 F
1y
2 m - P2 m + V
B
4 m F
2y
6 m = 0

Resolviendo el sistema se obtiene: H
A
= - 1,96 t
V
A
= + 3,9 t
V
B
= + 2,7 t
El signo negativo en el valor de H
A
indica que el sentido de esta componente de
reaccin es contrario al que se le supuso al comenzar la solucin del problema.


F
1y

F
2y

2 m 2 m 2 m
2 m
P = 2t
V
A

V
B

H
A

F
1x
F
2x

A
B













16
Movimiento Armnico Simple.
Sus ecuaciones.

En lo que va del ao, hemos estudiado diferentes tipos movimientos; lo que hace
diferentes a los tipos de movimiento, son las caractersticas de la fuerza que lo propulsa.
Pero an nos falta presentar un movimiento ms: el MAS. Repasando lo aprendido hasta
ahora y lo que estudiaremos:

MOVIMIENTO
F U E R Z A
MDULO DIRECCIN
MRU Fuerza neta nula
MRUV constante constante
MCU constante variable
MAS variable constante

Una fuerza con las caractersticas indicadas para producir un MAS es por
ejemplo, la que provee un resorte que obedece a la ley de Hooke: F = -ky. El dispositivo
ideal donde se genera el MAS recibe el nombre de oscilador armnico.

El oscilador armnico.
Este dispositivo consta de un resorte perfectamente
elstico, carente de masa, fijo por uno de sus extremos y
unido a un cuerpo masivo por el otro; el eje del resorte es
horizontal y el movimiento de la masa ocurre sin rozamiento.

Si apartamos la masa M de su posicin de equilibrio (estirando o comprimiendo
al resorte), al soltarla, inicia un movimiento alternativo hacia la derecha y la izquierda,
en torno de la posicin de equilibrio. ste es el MAS.

Puntualizaremos a continuacin las principales caractersticas que posee este
movimiento:
k M
Es peridico, o sea que se repite en su forma, a intervalos iguales de tiempo.
Es armnico. Este calificativo obedece a razones histricas; antiguamente se le daba
el nombre de armnicas a las funciones trigonomtricas seno y coseno. Como en
las ecuaciones horarias de este movimiento, como veremos enseguida,
invariablemente alguna de estas funciones est presente, por extensin se le dio el
nombre de armnico al movimiento.
La trayectoria es rectilnea.
La fuerza es elstica. Esto significa que obedece estrictamente a la ley de Hooke.

Vale la pena advertir que no todos los cuerpos elsticos obedecen a la ley de
Hooke; pero slo cuando es as, el movimiento recibe el nombre de MAS. Caso
contrario, al movimiento se le da un nombre ms general: movimiento oscilatorio
armnico.

La ley de Hooke se plantea tanto para la traslacin como para la rotacin:
la traslacin: F = -ky En la rotacin: = -k0

La i mpor t anci a del movi mi ento que comenzamos a est udi ar est en que l
ocur r e espont neament e en l a nat ur al eza y de una maner a muy di fundi da. No es
si mpl ement e por que ocur r e cuando t enemos una masa vi ncul ada en el ext r emo de
un r esor t e. As por ejempl o l o encont r amos en el movi mi ent o de l os punt os de una
cuer da de vi ol n en vi br aci n, en un pndul o en osci l aci n y en l os t omos dent r o
de una mol cul a.

Glosario.
OSCILACIN COMPLETA: Por ci n de movi mi ent o compr endida ent r e dos pasos
sucesi vos y con el mi smo sent i do, por un punto.
PERODO ( T) : Ti empo r equer i do par a r eal i zar una osci l aci n compl et a.
FRECUENCIA ( f) : Cant i dad de osci l aci ones compl et as r eal i zadas en l a uni dad de
t i empo.
ELONGACIN ( y) : Di st anci a desde l a posi ci n de equi l i br i o hast a l a par t cul a,
par a un i nst ant e dado.
AMPLITUD ( R) : Es l a mxi ma el ongaci n.

Ecuaciones horarias del MAS.
Par a deduci r l as ecuaci ones hor ar i as
del MAS es necesar i o saber que est e
movi mi ento puede consi der ar se como l a
pr oyecci n sobr e un eje cual qui er a, de un
MCU ( ver fi gur a) . Mi ent r as el punt o P
r ecor r e l a ci r cunfer enci a con MRU, su
pr oyecci n sobr e el eje y ( punto Q) r eal i za
un MAS. R

es el vect or posi ci n del punt o P;


y es el vect or el ongaci n del punt o Q. La
r el aci n ent r e ambos es:
y = Rsen 0
En est a expr esi n, 0 es funci n del t i empo; l a funci n que l os r el aci ona es l a
ecuaci n hor ar i a de l a posi ci n angul ar del MRU: 0 = 00 + t
0
y
x
A
B
O
R
y
P Q
Reempl azando, obt enemos l a ecuaci n hor ar i a de l a el ongaci n del MAS:
y = Rsen ( 00 + t ) ( 1)
Las r est ant es ecuaci ones hor ar i as se obt i enen por der i vaci n:
v =
d
dt
= R cos ( 00 + t ) ( 2)
a =
d
dt
= -
2
Rsen ( 00 + t ) = -
2
y ( 3)
Relacin entre , k y M
En el MAS t enemos t r es i mpor t ant es const ant es: l a del r esor t e ( k) , l a masa
col ocada en su ext r emo ( M) y que l l ega a t r avs de l a r el aci n que el MAS t i ene
con el MCU. Es l a vel oci dad angul ar del MCU y es const ant e; per o cuando
apar ece en el MAS, l e damos ot r o nombr e: es l a fr ecuenci a angul ar del MAS.

Est as t r es const ant es no son i ndependi ent es si no que est n vi ncul adas
mat emt i cament e; par a hal l ar l a expr esi n que l as r el aci ona, par t i r emos de l a 2
l ey de Newt on: F = Ma sta es una expresin general, que vale para cualquier
movimiento; para particularizarla al caso especfico del MAS, reemplazaremos F por la
expresin de la ley de Hooke y a la aceleracin, por la (3):
-ky = M( -
2
y)
Si mpl i fi cando: k = M
2
= _
k
M
( 4)

Perodo y frecuencia
Sabemos que T =
2n
o
sta es tambin una expresin genrica del perodo; sirve
en cualquier movimiento peridico. Para particularizarla para el MAS, reemplazaremos
por la (4):
T = 2 _
M
k
y f =
1
1
=
1
2n
_
k
M

Problemas.
1- Una partcula realiza un MAS de 1,83 m de amplitud, alcanzando una velocidad
mxima de 3,05 m/s. Determinar el valor del perodo del movimiento.
Sol uci n:
vmax = R =

mx
R
=
3,05
m
s

1,83 m
= 1,66
1
s

Como T =
2n
o
=
6,28
1,66
= 3,77 s

2- La jaula de un pjaro est suspendida de un resorte. Cuando el pjaro, de 0,2 kg est
en su percha, la jaula baja 0,25 cm con respecto a su nivel cuando est vaca. La masa
de la jaula es de 0,8 kg. Calcular los valores a) de la constante del resorte; b) del perodo
de oscilacin de la jaula vaca.
Solucin:
-Peso del pjaro: mg = 0,210 = 2 N
a) k =
P

=
2 N
0,025 m
= 800 N/ m
b) T = 2 _
M
k
= 6,28 _
0,8 kg
800 N/ m
= 0,20 s
3- Un automvil va por un camino cuya superficie est ondulada. Un pasajero vibra
verticalmente con MAS cuya amplitud es R = 2,45 cm. Cul es la mxima frecuencia a
la cual permanecer en contacto con el asiento?
Solucin:
En el l mi t e, l os mdulos de l a fuer za el st i ca y del peso del
pasaj er o, ser n i gual es: ky = mg
Luego:
k
m
=
g

Reempl azando, dent r o de l a r az de l a


fr mul a de l a fr ecuenci a:
f =
1
2n
_
g

=
1
6,28
_
10
0,0245
= 3,18
buconcs
s


La ecuacin diferencial del MAS
A par t i r de l a 2 l ey de Newt on: F = Ma y par a r efer i r l a al caso par t i cul ar
del MAS, r eempl azamos F por l a l ey de Hooke y a l a acel er aci n, por
d
2

dt
2
:
-ky = M
d
2

dt
2
M
d
2

dt
2
+ ky = 0
d
2

dt
2
+
k
M
y = 0
sta es la ecuacin diferencial del MAS; se trata de una ecuacin diferencial de
segundo orden, incompleta y homognea. Ntese que el coeficiente del trmino de la
variable sin derivar (
k
M
) segn la (4) es
2
.

Solamente el MAS posee una ecuacin diferencial con las caractersticas
apuntadas arriba. Ningn otro fenmeno de la naturaleza posee una ecuacin diferencial
del mismo tipo. Aprovecharemos esta circunstancia para estudiar la prxima clase los
diferentes tipos de pndulos, con el propsito de encontrar la expresin del perodo de
cada uno de ellos. En cada caso buscaremos la ecuacin diferencial del pndulo y
mostraremos que es del mismo tipo que la ecuacin diferencial del MAS. Comprobado
eso, la frmula del perodo del MAS, servir para dicho pndulo y nuestro trabajo se
reducir a adaptar esta frmula a los elementos particulares del pndulo en
consideracin.






















ky
mg















17
Aplicaciones del MAS

Estamos estudiando el MAS, un movimiento que se caracteriza por ser la
posicin una funcin peridica senoidal del tiempo. Todo MAS es peridico, pero no
todo movimiento peridico es un MAS.

El caminar es un ejemplo de movimiento peridico, pero que no es un MAS. A
propsito, todos los animales que caminan, (incluido el ser humano) tienen un ritmo
natural de marcha (una frecuencia de pasos por minuto) que les es propia. De qu
depende la rapidez con que lo hacen? Por un lado, de la longitud de las patas (o
piernas): piernas ms largas permiten una longitud de paso (zancada) mayor. Sin
embargo, un perro, con piernas de 20 cm nos gana en una carrera, a pesar que nuestras
piernas son 4 veces ms largas. Hay otro factor a tomar en cuenta. Los animales de
piernas largas, como las jirafas, tienen en general una frecuencia de paso baja y
viceversa. As, pruebas fsiles demuestran que el Tyrannosaurus rex, tena piernas de 3
m de longitud y daba pasos de 4 m de longitud; sin embargo su rapidez de marcha era
comparable a la del hombre.

En esta clase estudiaremos diferentes tipos de pndulos con el propsito de
obtener una frmula que nos permita calcular el valor del perodo de oscilacin.

El pndulo fsico.
Recibe el nombre de pndulo fsico, cualquier cuerpo rgido soportado de tal
forma que pueda oscilar en un plano vertical en torno de algn eje que pase por uno de
sus puntos.

De hecho todos los pndulos reales son pndulos fsicos; es como el padre de
todos los pndulos. La figura muestra un cuerpo de forma cualquiera, plano, que puede
girar sin rozamiento alrededor de un eje y que en este momento se encuentra apartado
de su posicin de equilibrio en un ngulo 0. Ntese que 0 es un ngulo que se mide a
partir de la posicin de equilibrio; as que es un ngulo orientado.

Lo primero que debemos ubicar son 2 puntos
importantes: S (punto de suspensin) y G (centro de gravedad)
La distancia entre ambos puntos es d. La posicin de
equilibrio es aquella en la que G se encuentra en la vertical que
pasa por S. La torca restauradora es cuyo sentido siempre es
opuesto al del ngulo 0. Su expresin es:
= - Mgdsen 0
Los factores M, g y d son constantes, y vamos a
agruparlos bajo una C: C = Mgd
Luego: = -Csen 0
Hemos llegado a una expresin que no es igual a la ley
de Hooke para la rotacin: ( = -C0). Esto nos indica que el pndulo fsico, cuando
oscila, NO realiza un MAS.

Vamos a imponer entonces una restriccin: que el ngulo 0 sea pequeo (por
ejemplo, menor de 6). Siendo as, se cumple que 0 (rad) y sen 0 son aproximadamente
iguales. Con esto logramos que la ley de Hooke se nos cumpla y que el movimiento del
pndulo pueda ser considerado un MAS. Entonces podemos seguir adelante con nuestra
deduccin, pero no debemos olvidar que a partir de este momento, todo lo que se
deduzca slo ser vlido para 0 pequeos.

Nuestro prximo paso ser obtener la ecuacin diferencial del pndulo fsico.
Partimos de la ecuacin fundamental de la dinmica de la rotacin:
= J
Y reemplazando por su expresin en la ley de Hooke y a por
d
2
0
dt
2
:
-C0 = J
(S)

d
2
0
dt
2

Ordenando:
d
2
0
dt
2
+
C
]
S
0 = 0 EC. DIF. DEL PNDULO FSICO
Esta ecuacin es del mismo tipo de la del MAS. En su momento vimos que el
coeficiente del trmino de la variable sin derivar, de la ecuacin diferencial representa a

2
. Luego en el pndulo fsico,
= _
C
]
S

La expresin del perodo es: T =
2n
o
= 2 _
]
S
C

Reemplazando C por su expresin, queda finalmente: T = 2_
]
S
Mgd


El pndulo simple.
Un pndulo simple es un cuerpo idealizado, consistente en una masa puntual,
unida al extremo de una cuerda sin masa e inextensible.

Para obtener la frmula del perodo, haremos una analoga entre sus elementos y
los correspondientes en el pndulo fsico. Llamaremos l a la longitud de la cuerda y m a
la masa suspendida. Entonces:
0
d
S
G
Mg

J = ml
2
M = m d = l
Haciendo estos reemplazos en la frmula del perodo del pndulo fsico,
obtenemos la conocida expresin del perodo del pndulo simple:
T = 2_
I
g

Como se ve, para un lugar determinado (g = cte) T depende nicamente de la
longitud de la cuerda. No depende de la amplitud de las oscilaciones ni del valor de la
masa colocada en su extremo. Significa entonces que si uno variara la masa del
pndulo de un reloj, ste no adelantara ni atrasara?

En el pndulo fsico, T tampoco depende de M. Probarlo.

El pndulo de torsin.
La figura muestra un disco suspendido mediante un alambre que lo toma por su
centro de masa. Estando el disco en equilibrio, marcamos sobre l el
radio OP. Si ahora giramos el disco en un ngulo 0, el alambre se
torsionar y ejercer una sobre el disco que tender a volverlo a la
posicin inicial; sta es la torca restauradora. Para 0 pequeos, se
cumple que es proporcional a 0, o sea que se cumple la ley de
Hooke:
= -k0
donde k es la constante de torsin, cuyo valor depende de las
propiedades del alambre.
El signo menos indica que

y 0

son vectores de sentidos opuestos.



A continuacin deduciremos la ecuacin diferencial de este pndulo. A partir de
la ley fundamental de la dinmica de la rotacin:
= J
cm

y reemplazando: -k0 = J
cm

d
2
0
dt
2

Ordenando, se obtiene la ecuacin diferencial del pndulo de torsin:

d
2
0
dt
2
+
k
]
cm
0 = 0
que por sus caractersticas corresponde a un MAS. Por lo tanto, el coeficiente del
trmino que tiene a la variable sin derivar, equivale a
2
:

k
]
cm
=
2

Para obtener la frmula del perodo se reemplaza esta expresin de en la
frmula general del perodo:
T =
2n
o
= 2n _
]
cm
k


Pndulo simple sincrnico.
Se llama as, al pndulo simple hipottico, que se equivale con otro pndulo dado
(fsico, de torsin, etc) en el sentido de igualar con el valor de su perodo, al perodo del
pndulo dado. Cuando en un problema se pide calcular el pndulo simple sincrnico, lo
que se debe hallar es la longitud que ese pndulo deber tener, para que se cumpla tal
equivalencia. Por ejemplo, si se pide hallar el pndulo simple sincrnico equivalente a
un pndulo fsico dado, se igualan las frmulas de perodo de ambos pndulos:

O
0

P
k
T = 2_
I
g
= 2_
]
S
Mgd

I
g
=
]
Mgd
l =
]
Md


Problemas.
1- Una barra homognea de 1 m de longitud, se cuelga por uno de sus extremos y
cuando se la aparta de su posicin de equilibrio en un ngulo 0 pequeo, comienza a
oscilar con un perodo T = 1,64 s. Determinar el valor de la aceleracin de la gravedad.
DATO: J
CM
(barra) =
1
12
Ml
2

Solucin:
Calculamos primero el J
(S)
= JC
M
+ Md
2
=
1
12
Ml
2
+ M[
I
2

2
=
1
3
Hl
2

Reemplazando en la frmula del perodo del pndulo fsico:
T = 2n _
2MI
2
3MgI
= 2n
_
2I
3g

Despejando g: g =
8
3
l [
n
1

2
= 9,79 m/s
2
.

2- Un adorno navideo con forma de esfera hueca de masa M = 0,015 kg y radio R =
0,050 m se cuelga de una rama con un lazo de alambre unido a la superficie de la esfera.
Si el adorno se desplaza una distancia corta y se suelta, oscila como pndulo fsico con
friccin despreciable. Calcular su perodo. DATO: J
CM
(esfera hueca) =
2
3
H R
2

Solucin:
Por Steiner: J
(S)
= J
CM
+ MR
2
=
2
3
H R
2
+ H R
2
=
5
3
H R
2

Reemplazando en la frmula del perodo del pndulo fsico:
T =2 _
5R
3g
= 2n _
50,05
310
= 0,57 s

































18
M.A.S. Otras consideraciones

Resortes agrupados.
Estudiaremos a continuacin el caso del movimiento de una masa M que se
encuentra vinculada a dos (o ms) resortes, de constantes k
1
y k
2
respectivamente. En
este caso se procurar determinar cunto debera vales la constante elstica de un nico
resorte (k
uiveq
que fuera capaz de producir sobre la masa, el mismo MAS que el que
producen juntos los resortes originales.

Existen dos formas de combinar los resortes: en serie y en paralelo, que pasamos
a considerar:

1 CASO: Resortes en serie.
En este caso, el alargamiento del conjunto es
igual a la suma de los alargamientos de cada resorte:
X = x
1
+ x
2
(1)
Las fuerzas elsticas son todas iguales:
Fe = Fe
1
+ Fe
2

Por Hooke: Fe
1
= k
1
x
1
x
1
= Fe
1
/k
1
Fe
2
= k
2
x
2
x
2
= Fe
2
/k
2

Reemplazando en (1):

P
k
cqu
=
Pc
1
k
1
+
Pc
2
k
2

1
k
cqu
=
1
k
1
+
1
k
2


2 CASO: Resortes en paralelo.
Aqu todos los resortes se estiran igual, y el alargamiento
del conjunto es igual al de un resorte cualquiera:
X = x
1
= x
2

En cambio la fuerza elstica total es igual a la suma de las
fuerzas elsticas en cada resorte:
Fe = Fe
1
+ Fe
2
(1)
Por Hooke: Fe
1
= k
1
x
1
Fe
2
= k
2
x
2

Reemplazando en la (1): k
eq
X = k
1
x
1
+ k
2
x
2
K
equ
= k
1
+ k
2

k
1
k
2
M
M
k
1

k
2


Problemas:
1- Analizar si los resortes que se muestran en el dibujo, estn conectados en serie o en
paralelo.
Solucin: Se M se corre hacia un costado,
un resorte se acorta y el otro se alarga, pero
ambos en la misma medida; luego x
1
= x
2
;
eso ocurre cuando la conexin es en
paralelo.

2- Un resorte de masa despreciable, de k = 7 N/m se corta en dos partes iguales. a)
Cunto vale el k de cada una de las mitades? b) Ambas mitades, unidas en paralelo,
con sus ejes verticales, soportan un cuerpo de masa M; si el sistema vibra con f = 3 Hz,
cunto vale M?
Solucin:
a) Se puede considerar que cuando el resorte estaba entero, era lo mismo que si tuviera
sus dos mitades puestas en serie; luego:

1
k
cqu
=
1
ki
+
1
ki
k
i
= 2 k
cqu
= 2 7 = 14
N
m

b) Ahora, con las mitades en paralelo, el nuevo k es:
k
equ
= k + k= 14 + 14 = 28 N/m

f =
1
2n
_
k
M
M =
k
( 2n])
2
= 0,0788 kg 79 g

Valores de v y de a en el MAS
Hemos visto que en el MAS, tanto v como a tienen valores variables, que
dependen de la posicin en que se encuentre la partcula. Nos proponemos encontrar
dnde estas magnitudes valen cero y dnde son mximas.

Sean A y B los extremos de la trayectoria seguida por una masa que realiza un
MAS; O (punto medio) representa la posicin de equilibrio. La fuerza que el resorte
aplica, tiene en todo momento su sentido dirigido hacia O y su mdulo es nulo en O y
crece hacia los extremos, siendo mxima justamente en ellos. A travs de F = Ma
vemos que a vara de la misma manera que F, por lo que es mxima en los extremos A y
B y nula en la posicin de equilibrio O.

Por el contrario, v es nula en los
extremos (aqu M se detiene para cambiar el
sentido del movimiento) y se hace mxima
en la posicin de equilibrio, a la que llega
acompaada por una a que si bien tiene
mdulo variable, tiene el sentido del movimiento; pero cuando traspasa O, a pasa a tener
sentido opuesto al movimiento, y comienza a frenar a la masa.

Los trapecistas de circo hacen sus demostraciones de acrobacia, cuando el
trapecio est llegando a un extremo, porque all la velocidad es muy baja, y tienen ms
tiempo para organizar sus movimientos y coordinarlos con los de sus compaeros.

M
k
1
k
2

A B
O
amx a = 0 amx
v = 0 vmx v = 0
La energa en el MAS
Teniendo en cuenta que en el MAS slo intervienen fuerzas conservativas (no
hay friccin), la energa mecnica debe conservarse, teniendo que cumplirse que:
Em = Ep
e
+ Ec

Como el movimiento ocurre en direccin horizontal, la Ep
g
es constante a lo
largo de toda la trayectoria, y por eso no la tomamos en cuenta. Nos proponemos trazar
un diagrama cartesiano E = f(y) que nos muestre cmo varan la Em, la Epe y la Ec a
medida que la masa va recorriendo su trayectoria.

Si la Em es constante, estar representado por una lnea horizontal; (ver figura).
La Epe es la energa que se encuentra almacenada en el resorte y depende de y segn
la relacin:
Ep
e
= ky
2

Ella es nula en la posicin de equilibrio, donde y = 0 y es mxima donde la deformacin
del resorte es mxima, o sea en los extremos A y B, donde el | y| es mximo. Pero es los
extremos, por ser v = 0, es Ec = 0, lo
que hace que Ep
e
= Em. Todo esto me
permite ubicar tres puntos de la curva
de la Ep
e
; pero, qu clase de curva
es?: la ecuacin escrita arriba,
corresponde a una funcin de 2 grado
en y, por lo que es una parbola.
(Trazamos entonces la parbola en el
diagrama).

Las barras verticales compren-
didas entre la parbola y el eje de abs-
cisas miden Epe, mientras que las barras verticales punteadas complementarias (de la
parbola hacia arriba hasta la horizontal de Em), miden la Ec.

La frmula para calcular la Ec es la conocida: Ec = mv
2
. De modo que
conocemos frmulas para hallar la Ep
e
y para la Ec; sin embargo no conocemos ninguna
para hallar en forma directa la Em. Pero ahora podemos obtenerla. En los extremos de la
trayectoria, al ser Em = Ep
e
y teniendo en cuenta que en los extremos la y es mxima,
o sea es la amplitud A:
Em = kA
2


La energa mecnica de una partcula en MAS es proporcional al cuadrado de la
amplitud.

Problema.
Una porrista ondea su pompn en MAScon amplitud de 18 cm y frecuencia de
0,85 Hz. Calcular: a) la magnitud mxima de de la aceleracin y de la velocidad; b) la
aceleracin y rapidez cuando la coordenada del pompn es y = + 9 cm; c) el tiempo que
tarda en moverse directamente dede la posicin de equilibrio hasta un punto situado a
12 cm de distancia.
Solucin:
= 2f = 6,280,85 = 5,34
1
/
s

E
Ep
e

y
0
A B
Em
Ec
a) v
mx
= A = 5,340,18 = 0,96 m/s
a
mx
=
2
A = 5,34
2
0,18 = 5,13 m/s
2
.

b) a = -
2
y = -(5,34)
2
0,09 = -2,56 m/s
2
.
y
b
= A sen 0
b
0
b
= arc sen

b
A
= orc scn
0,09
0,18
= 30
v = A cos 0 = 5,340,18cos 30 = 0,83 m/s

c) y
c
= A sen 0
c
0
c
= arc sen

c
A
= orc scn
0,12
0,18
= 0,7297 roJioncs
0
c
= t t =
0
c
o
=
0,7297
5,34
= 0,14 s

MAS vertical.
Supongamos que colgamos un resorte de constante k y suspendemos de l un
cuerpo de masa m. Las oscilaciones ahora sern verticales; seguirn siendo MAS? Para
considerar este caso, nos remitimos a las figuras de abajo. En (a) se muestra al resorte
con su longitud natural , sin ninguna masa colocada en su extremo. En (b) se muestra al
mismo resorte, pero con la masa m en su extremo; su extremo libre ha descendido en
una longitud y
0
hasta encontrar la posicin de equilibrio esttico (p.e.e.) En esta
posicin, la fuerza elstica del resorte se equilibra con el peso del cuerpo:
mg = ky
0
y
0
=
mg
k


En (c) la masa m ha sido apartada de su pee, y cuando se la suelte iniciar un
MAS. Es importante saber que la masa m oscila simtricamente alrededor de su ppe, por
lo que nuestro sistema de referencia (un eje vertical y) convendr adoptarlo con su
origen en la p.e.e.

Problema.
En el dispositivo de la figura, la polea tiene un
J
cm
= 0,5 kgm
2
y un R = 30 cm; el resorte tiene un k
= 2 N/m. Se ata al extremo de la cuerda, una masa de
100 g y se la suelta desde esta posicin. Determinar:
a) la p.e.e. b) la rapidez con que la masa pasa por la
p.e.e.
Solucin:
0
y
p.e.e
.
y
0

ky
0

mg
(a) (b) (c)
(1)
(2)
0
y
p.e.e
.
a) Si bajamos lentamente con la mano a la masa m, evitando que oscile hasta dejarla en
su posicin de reposo (pee), se cumple en ella:
mg = ky
0
y
0
=
mg
k
= 0,5 m
b) La velocidad con que la masa pasa por la pee es la mxima; para calcularla aplicamos
el principio de la conservacin de la Em, dado que se trata de un sistema conservativo.
Podemos poner: Em
1
= Em
2

Epg
1
= Ec
tras
+ Ec
rotac
+ Epe
2
.
mgy
0
= mv
2
+ J
cm

2
+ ky
0
2


Reemplazando y calculando, resulta v = 0,297 m/s

Oscilaciones compuestas.
Estudiaremos a continuacin el caso de una partcula que se encuentra sometida
a la accin simultnea de 2 MAS independientes. El movimiento resultante de la
partcula ser otro MAS cuyas caractersticas podrn conocerse aplicando el principio
de superposicin de efectos. Nosotros slo consideraremos el caso en que ambos MAS
originales tienen la misma direccin. Dentro de este caso, veremos dos posibilidades:
1- Ambos MAS son de la misma frecuencia angular
2- Ambos MAS son de diferente frecuencia angular

1 caso: Ambos MAS son de igual direccin e igual
Sean las ecuaciones horarias de la elongacin de ambos MAS:
y
1
= A
1
sen (0
1
+ t)
y
2
= A
2
sen (0
2
+ t)
La ecuacin horaria del MAS resultante ser:
y = Asen (0 + t)
Las incgnitas son A y 0; nuestra tarea es encontrar cmo estas incgnitas estn
relacionadas con los datos originales. Recordaremos que todo MAS puede ser
considerado como la proyeccin sobre un eje (elegimos el eje y) de un MCU. Mientras
una partcula realiza un MCU, su vector posicin est girando a = cte, en torno del
origen; un vector giratorio, se llama fasor. La proyeccin de este fasor sobre el eje y
nos da la elongacin del MAS instante a instante.

Los vectores A
1
y A
2
se componen para dar el vector A, cuyo mdulo, por el
teorema del coseno es:
A = A
1
2
+ A
2
2
+ 2 A
1
A
2
cos ( 0
1
0
2
)
La fase inicial se calcula con:
0
2

01 0
A
1

A
2

A
0
1
0
2

180 - (01 - 02)

y
1

y
2

y
x

0 = orc tg
A
1
scn 0
1
+ A
2
scn 0
2
A
1
cos 0
1
+ A
2
cos 0
2

Una vez obtenidas A y 0, se escr i be l a ecuaci n de l a el ongaci n del
movi mi ento r esul t ant e como:
Y = Asen ( 0 + t )
donde es el mi smo de l os MAS or i gi nal es. Consi der ar emos a cont i nuaci n un par
de casos ext r emos:

Interferencia constructiva:
Si 01 = 02, si gni fi ca que ambos MAS or i gi nal es est n en fase; en ese caso A
adqui er e su mxi mo val or , si endo st e di r ect ament e:
A = A1 + A2.
Interferencia destructiva:
Si en cambi o 01 y 02 di fi er en en , si gni fi ca que ambos MAS est n en
oposi ci n de fase; en ese caso A adqui er e su mni mo val or , si endo st e
di r ect ament e:
A = A1 A2.

Las expresiones de la amplitud, tanto en la interferencia constructiva como en la
destructiva, pueden deducirse a partir de la expresin de A de la pgina anterior.

Haciendo un resumen de este 1 caso,
lo importante es que la amplitud resultante
NO ES funcin del tiempo
Las lneas horizontales en el grfico
de la figura, corresponden a los casos de
interferencia. Para cualquier otro caso, la
amplitud estar representada por una lnea
horizontal que estar comprendida dentro de
la franja determinada por las lneas de la
figura.

2 caso: Ambos MAS son de igual direccin pero distinta
Si los fasores A
1
y A
2
giran con distintas frecuencias angulares (como ocurre con
las agujas del reloj) pasarn peridicamente por un momento de interferencia
constructiva (cuando sus direcciones y sentidos coincidan) y por otro momento de
interferencia destructiva (cuando sus sentidos se opongan). Por lo tanto, la amplitud
resultante ES funcin del tiempo y vara peridicamente con l. La representacin
grfica de la amplitud resultante A en funcin del tiempo es ahora:

Si se trata de 2 ondas sonoras
emitidas por sendas fuentes (por ej: 2
diapasones) con ligeramente diferentes, la
interferencia de ellas en el odo alterna entre
la interferencia constructiva y la destructiva.
La intensidad del sonido alterna entre alto y
bajo. Cada pulsacin se denomina batido. El
batido tiene una frecuencia calculable como
Interf. Constructiva. A
1
+ A
2

Interf. Destructiva. A
1
A
2

A
t
A es constante en el tiempo
1 batido
A
1
+ A
2

A
1
A
2
t
A
A es funcin del tiempo
la diferencia entre las frecuencias de los dos focos sonoros.

El fenmeno de las pulsaciones o batidos se aplica para comparar una frecuencia
desconocida con otra conocida, como sucede cuando se afina un instrumento musical
con un diapasn. El odo puede detectar pulsaciones hasta aproximadamente 10 por
segundo; por encima de este valor las fluctuaciones resultan demasiado rpidas para ser
percibidas.

El ciudad de Viena posee la mayor densidad de msicos del mundo; en el
servicio de telefona local, cuando uno llama al nmero *1640 no es atendido por
ningn contestador automtico ni escucha la voz de ninguna telefonista; slo escucha un
silbido continuo que corresponde a la frecuencia de la nota la (440 Hz). Con ella,
afinan sus instrumentos los msicos de Viena.

Tambin se utilizan los batidos para detectar pequeas variaciones de frecuencia
como las producidas en un haz de radar reflejado desde un auto en movimiento. La
velocidad del auto se halla midiendo la variacin de frecuencia, determinada midiendo
los batidos producidos por el haz reflejado y la fuente de radar original. Para
comprender mejor esto, supongamos que un muchacho tiene un canasto lleno de
pequeas pelotitas, y se encuentra ubicado frente a una pared mvil. En un momento
dado comienza a arrojar las pelotitas sucesivamente contra la pared, con una frecuencia
de 1 por segundo. Si la pared no se mueve, las pelotitas que regresan despus de haber
rebotado, tendrn tambin la misma frecuencia de 1 por segundo. Pero si la pared se
mueve, acercndose o alejndose, la frecuencia de las pelotitas que regresan variar,
aumentando o disminuyendo respectivamente en cada caso.

Oscilaciones amortiguadas
Hasta ahora hemos estudiado el caso del oscilador
armnico simple, en el cual no intervienen las fuerzas de
rozamiento; pero ste no es un caso real, ya que sin un pndulo
o un cuerpo suspendido de un resorte oscilaran eternamente;
pero sabemos que la amplitud de la oscilacin disminuye poco a
poco hasta extinguirse, como resultado de la friccin. Este movi-
miento real se llama movimiento armnico amortiguado. La
friccin proviene a menudo de la resistencia del aire o de las
fuerzas internas (friccin entre molculas dentro del resorte). Se
ha encontrado que el mdulo de la fuerza de rozamiento es
proporcional a la velocidad del cuerpo, y su sentido es opuesto al
de sta. Si nombramos con F
R
a la fuerza de amortiguamiento,
ser:
F
R
= -bv
siendo b la constante de amortiguamiento, siempre positiva.
La figura muestra muestra el dispositivo generador de oscilaciones amortiguadas.

ECUACIN DIFERENCIAL DEL OSCILADOR AMORTIGUADO.
Partiendo de la 2 ley de Newton: F
neta
= ma
Desarrollando para este caso: -ky bv = ma
-ky b
d
dt
= m
d
2

dt
2

Ordenando:
d
2

dt
2
+
b
m

d
dt
+
k
m
y = 0 (1)
k
m
b
Si b es pequea, la solucin de la ecuacin diferencial (1) es:

y = Ac
-
bt
2m
cos(
i
t + 0
0
) (2)
donde A c
-
bt
2m
recibe el nombre de factor de amplitud. La frecuencia angular est
dada por:
=
_
k
m
[
b
2m

2
(3)
Si no hubiera friccin, sera b = 0; en este caso vemos en la (3) que se hace
igual a . Pero habiendo friccin resulta:
<
Esto nos indica que el perodo de la oscilacin amortiguada es mayor que el de
la oscilacin sin amortiguar.

La figura muestra la represen-
tacin grfica de la ecuacin (2) para el
caso en que b es pequea, o sea que haya
baja amortiguacin, y para 0
0
= 0.

En la (2) vemos cmo la amplitud
tiende a cero al crecer t. Se define al
tiempo de vida medio de la oscilacin (o
constante de tiempo) () como el tiempo
que debe transcurrir para que la amplitud
de la oscilacin se reduzca a
1
c
= 0,37 de su valor inicial.

Las lneas asintticas en guiones de la figura responden a la ecuacin que define
al factor de amplitud. Para obtener una expresin que permita calcular el valor de ,
tomamos la expresin del factor de amplitud: A c
-
bt
2m
donde representa el valor de t
que hace que el exponente valga -1:
-
bt
2m
= -1 =
2m
b

Si no hubiera friccin (b = 0) la (2) muestra que sera A = constante, y = .
La constante b puede adoptar valores dentro de un rango amplio; para decidir si
el valor de b es pequeo o grande, se lo compara con el valor del producto 2m. De
dicha comparacin surgen 3 posibilidades:

a) Si b < 2m se dice que b es pequeo y que el sistema es subamortiguado. Este es el
nico caso en que el movimiento es peridico, y por lo tanto la ecuacin (2) nicamente
es vlida para este caso. Como que tambin, la grfica de arriba ilustra nicamente este
caso.

b) Si b = 2m se dice que hay amortiguamiento crtico; el movimiento no es peridico
(no hay oscilaciones) y el sistema se dirige hacia la posicin de equilibrio en un tiempo
mnimo.

c) Si b > 2m se dice que b es grande y que el sistema est sobreamortiguado. Como
en el caso anterior, el movimiento no es peridico y el sistema se dirige hacia la
posicin de equilibrio sin oscilar. Slo que el tiempo que emplea para llegar al
equilibrio es mayor al del caso de amortiguamiento crtico.
y
t
0
A

37% A

Los amortiguadores de un automvil procuran que el vehculo vuelva al
equilibrio rpidamente, evitando las oscilaciones. Sin embargo no se los disea para que
respondan con amortiguamiento crtico, sino que se procura que sean ligeramente
subamortiguados, por una cuestin de confort. Esto puede comprobarse empujando
verticalmente encima de las ruedas del vehculo y observando que se producen una o
dos oscilaciones antes de quedar en reposo.

Oscilaciones forzadas.
Hasta ahora hemos estudiado las oscilaciones naturales de un cuerpo, sin o con
friccin. Pero una situacin diferente se presenta cuando el cuerpo se halla sometido a la
accin de una fuerza oscilatoria externa, a la que llamaremos fuerza impulsora. Las
oscilaciones resultantes reciben el nombre de oscilaciones forzadas. Estas oscilaciones
tienen la frecuencia de la fuerza impulsora y no la frecuencia natural del cuerpo.

En este fenmeno que vamos a estudiar, se da el hecho de que una sucesin de
pequeos impulsos peridicos (fuerza impulsora) aplicados con la frecuencia apropiada,
es capaz de producir oscilaciones de gran amplitud. Es el caso del nio que, empujando
un columpio a intervalos apropiados de tiempo, logra que el columpio se mueva con una
amplitud grande. Cuando un cuerpo o sistema es sometido a una amplitud muy grande,
ya no es vlida la ley de Hooke y se sobrepasa el lmite elstico; en tales casos el
sistema puede sufrir una ruptura. Un conocido ejemplo de esto lo tenemos en lo que
sucedi con el puente de Tacoma sobre el ro Potomac. El 1 de julio de 1940 se termin
su construccin en Washington, y se lo abri al trnsito. Tan solo 4 meses despus, un
ventarrn moderado puso al puente en oscilacin, hasta romper el tramo principal, que
se desprendi de los cables y cay al agua. El viento haba producido una fuerza
resultante peridica que entr en resonancia con la frecuencia natural de la estructura.
Esto provoc un aumento continuo en la amplitud, hasta destruir al puente.

Una formacin de soldados que marchan sobre un puente puede hacer que ste
vibre en forma creciente hasta alcanzar una amplitud destructiva, si se diera que la
frecuencia de sus pasos coincidiera con alguna frecuencia natural del puente; esta es l
razn por la que los soldados rompen la marcha al pasar por un puente.

La fuerza impulsora obedece a una expresin del tipo: F
impuls
. = F
mx
cos t

ECUACIN DIFERENCIAL DE LAS OSCILACIONES FORZADAS.

Aplicando la 2 ley de Newton: F
neta
= ma
-ky - b
d
dt
+ F
mx
cost = m
d
2

dt
2
(1)
donde los trminos del 1 miembro, representan respectivamente a la fuerza
restauradora, a la fuerza de amortiguamiento y a la fuerza impulsora. En lo que sigue
emplearemos la siguiente nomenclatura:
frecuencia de la fuerza impulsora.

0
frecuencia natural del cuerpo
b constante de amortiguamiento
0 ngulo de fase inicial.

La solucin de la ecuacin
diferencial (1) que damos a
continuacin, contiene elementos y
relaciones que, se recordarn ms
fcilmente usando como ayuda el
tringulo rectngulo siguiente:

La solucin de la (1) es:
y = Asen (t 0) (2)
donde A =
P
mx
u
(3)
siendo G = m
2
(
2

0
2
)
2
+ b
2

2
(4)
y la fase 0 puede calcularse como:
0 = arc cos
bo
u
o bien como 0 = orc tg
m(o
2
- o
0
2
)
bo


La ecuacin (2) nos permite ver que el sistema vibra con y no con
0
; que el
movimiento es armnico y que la amplitud no disminuye (A = cte).

Si un sistema carece de amortiguamiento (en realidad siempre lo hay), sera b =
0; en esas condiciones, si adems
0
, la ecuacin (4) dar G = 0 y en la ecuacin
(3) la amplitud tender a infinito. En los sistemas amortiguados reales, con b 0, existe
un cierto valor de para el cual la amplitud A es mxima; ese valor de se llama
frecuencia de resonancia.

RESONANCIA es el fenmeno que tiene lugar cuando la frecuencia de la fuerza
impulsora () posee un valor tal que hace que las oscilaciones forzadas alcancen su
mxima amplitud.

m(
2

0
2
)
b
G
0





















19
Movimiento ondulatorio

Todos alguna vez hemos dejado caer una piedra en aguas tranquilas; habremos
observado entonces que a partir del sitio donde la piedra haya tocado el agua se
producen en la superficie unos movimientos con la forma de circunferencias
concntricas que se van agrandando. La superficie del agua se curva alternadamente
hacia abajo y hacia arriba. Las formas obtenidas se denominan ondas y a su conjunto,
tren de ondas. El punto ms alto de la onda se denomina cresta y el ms bajo, valle. Un
corcho flotando, ante el paso de las ondas, alcanzar sucesivamente en un mismo punto,
valles y crestas; vale decir que ser movido hacia arriba y hacia abajo y recibir
cantidad de movimiento y energa cintica. Y es en esto que reside la particularidad
caracterstica del movimiento ondulatorio.

El movimiento ondulatorio es un recurso de la naturaleza que hace posible el
transporte de cantidad de movimiento y de energa desde un punto del espacio hasta
otro, sin transporte de materia.

Clasificacin de las ondas.
Las ondas pueden clasificarse de diversas maneras, segn el criterio de
referencia que se elija. As, por su naturaleza, las ondas pueden ser mecnicas o
electromagnticas. Como ejemplo de las primeras estn las ondas en el agua ya citadas,
las ondas en una cuerda y las ondas sonoras; en todas ellas las ondas son perturbaciones
que se propagan gracias a las propiedades elsticas del medio material.
En las ondas electromagnticas (radio, luz, TV, radar, etc) en vez, la
propagacin no necesita de ningn medio material para que pueda ocurrir.
En este curso solamente se estudiarn las ondas mecnicas. Como un ejemplo
ms de onda mecnica tenemos el caso de una bandera flameando en lo alto de un
mstil; las ondas se propagan a lo largo del tejido, pero as como el agua no se desplaza
ante el paso de las ondas superficiales, tampoco en este caso los hilos que forman la
trama del pao de la bandera abandonan sus lugares ante el paso de la onda.
Otro criterio de clasificacin las separa en ondas peridicas y en pulsos. Aqu,
depende de la duracin de la perturbacin; cuando el movimiento se repite una y otra
vez tenemos las primeras. Si la perturbacin es muy breve y es nica, tenemos el pulso.
Tanto las ondas peridicas como los pulsos gozan de propiedades similares, pero
seguramente resulta ms sencillo estudiarlas a partir de los pulsos que de las ondas
peridicas; por eso comenzaremos por estudiar los pulsos.

Pulsos.
PULSO es una onda de corta duracin. Tiene un principio y un final. En un instante
cualquiera slo se ve perturbada una regin limitada del espacio

Ejemplos: 1) Si una corriente de aire cerrara bruscamente la puerta del aula. El aire del
interior se comprime, se origina una perturbacin que cruzar el espacio y podremos
observar cmo una cortina en el fondo del aula sufre un movimiento sbito.
2) Si se ponen 3 o ms monedas en fila y en contacto sobre la mesa y se lanza otra
moneda para que golpee con fuerza a la del extremo de la fila, se ver cmo la moneda
del otro extremo sale despedida. Cada moneda ha sido perturbada; la perturbacin pas
a travs de la fila completa de monedas, pero ninguna de ellas se ha movido, excepto la
ltima.
3) El disparo de un arma de fuego (pulso de onda sonora).
4) El disparo de un flash fotogrfico (pulso de luz)
5) Una ola gigante (tsunami) en un maremoto. (pulso de onda en el agua).

Cmo podemos producir un pulso de una manera sencilla? Atemos el extremo
de una cuerda a un soporte y sometmosla a tensin, tirando con una mano desde el otro
extremo; si en esas condiciones le damos un golpe a la cuerda con el borde de la otra
mano, la cuerda sufrir un cambio de forma que comenzar a propagarse por la cuerda.
Hemos producido un pulso.

Un pulso es una perturbacin de la cuerda, es decir una deformacin o distorsin
respecto de la forma que posee la cuerda en su situacin de equilibrio.

La funcin de ondas.
La funcin de onda es una funcin de 2 variables, porque y depende tanto del
tiempo t como de la posicin x. Matemticamente lo expresamos poniendo que:
y = f(x;t)

ONDA PERIDICA PULSO
Ejemplificando: sea y la perturbacin o pulso que viaja por la cuerda en el
sentido +x. Si se toma una fotografa instantnea, observando la foto (en ella el tiempo
est detenido), diremos que y = f(x) ya que al pulso se lo ve en un cierto valor de x y
no en los dems. Pero si comparamos una secuencia de fotografas instantneas,
veremos que el pulso aparece en distintas posiciones x a medida que transcurre t,
por lo que diremos que y = f(t).

Para deducir la expresin de esta funcin de 2 variables, y dado que no estamos
acostumbrados a trabajar con ecuaciones con 2 variables, procederemos en dos etapas:
1 ETAPA: Por un momento dejaremos de lado una variable y deduciremos la relacin
de y con la otra variable.

2 ETAPA: Incorporamos ahora la otra variable y llegamos a la expresin final.

En la primera etapa eliminaremos el efecto del tiempo. La figura muestra un
pulso en una cuerda en un cierto instante; un rato ms tarde el pulso se habr alejado por
la cuerda, de modo que el pulso estar expresado por otra funcin de x.
Introduciremos un segundo sistema de referencia con origen en O, que se
mueve con la rapidez v del pulso. En este sistema de referencia mvil, el pulso no
depende del tiempo (es estacionario). La forma de la cuerda es y = f(x) en todo
momento. Hemos cumplido la primera etapa y ha resultado:
y = f(x). (1)
Para la segunda etapa, relacionamos las coordenadas de los dos sistemas de
referencia: y = y
x = x + vt
Si es esta ltima, despejo x: x = x - vt
y reemplazando en la (1):
y = f(x - vt)
Esta es la expresin de la funcin de onda, donde v es la velocidad de
propagacin de la onda e y representa el desplazamiento de la cuerda, para un lugar x
dado y un tiempo t dado. El signo (-) en la funcin de onda indica que el pulso se
propaga hacia la derecha. Si la propagacin ocurriera hacia la izquierda, se tendra un
signo (+).

Interferencia.
Sean dos pulsos movindose en sentidos opuestos. La forma de la cuerda donde
ambos pulsos se cruzan se obtiene sumando los desplazamientos producidos por cada
pulso individualmente (principio de superposicin).
Si se tratara de dos pulsos geomtricamente iguales, pero uno invertido respecto
del otro, la suma da por resultado cero. En este instante la cuerda est horizontal pero no
y y
vt
x
x
0
O
Sistema de
ejes fijos.
Sistema de
ejes mviles
v
en reposo. Despus de un breve tiempo los pulsos emergen y cada uno contina su
camino en su direccin y sentido original.

La combinacin de ondas separadas para producir una onda resultante se denomina
interferencia. La interferencia es una propiedad caracterstica del movimiento
ondulatorio. Se presenta siempre que dos ondas se encuentran en una misma regin
del espacio. No existe ninguna situacin anloga en el movimiento de partculas. Dos
partculas nunca se solapan o se suman de este modo. La interferencia es una
caracterstica nica correspondiente al movimiento ondulatorio.

La suma matemtica de dos funciones de onda para obtener la funcin de onda
resultante, se denomina principio de superposicin. En la mayor parte de los fenmenos
ondulatorios este principio se cumple, pero hay
excepciones; las ondas que no obedecen este principio
se llaman ondas no lineales y no las estudiamos.
Si el desplazamiento de los dos pulsos es en el
mismo sentido, el pulso resultante cuando se solapan es
mayor que cualquiera de los pulsos aislados. En este
caso se denomina interferencia constructiva. Si los
pulsos estn invertidos, cuando se solapan, dan un pulso
resultante menor que el mayor y quizs menor que
cualquiera de los dos. Los pulsos tienden a anularse
entre s cuando se solapan. Esta interferencia se
denomina interferencia destructiva.

Reflexin de pulsos.
Cuando un pulso se est propagando a lo largo de una cuerda tensa y llega al
extremo de la misma, se produce el fenmeno de la reflexin. Igual a lo que sucede con
un rayo de luz que se propaga por el aire, cuando llega a la superficie de un espejo, se
refleja, es decir que retorna al mismo medio por el cual vena. En el caso de un pulso en
una cuerda, cuando llega a un extremo, se iniciar su regreso. Sin embargo, la forma en
que se produzca dicho regreso, podr ser diferente, segn cmo sea la fijacin de la
cuerda. Por eso hablamos de extremo fijo y extremo libre. El primero se logra con una
atadura firme que impida todo movimiento a la cuerda en dicho extremo. El segundo,
permite mantener la tensin en la cuerda, sin impedir su desplazamiento, ante la llegada
del pulso. Estudiaremos a continuacin cmo es la reflexin en cada uno de estos
extremos.

REFLEXIN EN UN EXTREMO FIJO. El pulso reflejado se invierte respecto del
pulso incidente y comienza a recorrer la cuerda en sentido inverso, con la misma
velocidad con que lo haca el pulso incidente. Ocurre que cuando el pulso incidente
alcanza el extremo de la cuerda e intenta desplazarla, al no poder moverse por estar
atada, la cuerda trasmite la fuerza al soporte. Por el principio de accin y reaccin, el
soporte responde aplicando a la cuerda otra fuerza igual y contraria; esta fuerza genera
al pulso reflejado, que aparece invertido en relacin al pulso incidente.

REFLEXIN EN UN EXTREMO LIBRE. Cuando el pulso llega al extremo de la
cuerda, intenta desplazar a la cuerda y como el vnculo no impide tal desplazamiento, la
cuerda no transmite ninguna fuerza al vnculo. Si el vnculo no recibe ninguna fuerza,
tampoco responde con ninguna fuerza, por lo que el pulso reflejado se conserva
derecho. No hay inversin.


Velocidad de las ondas en cuerdas.
Una propiedad general de las ondas es que su velocidad depende de las
propiedades del medio, y que es independiente del movimiento de la fuente relativo al
medio, con excepcin de las ondas electromagnticas, que se propagan en el vaco con
v
0
= 3x10
8
m/s. Por ejemplo, la velocidad de una onda sonora producida por el silbato
de un tren depende solo de las propiedades del aire y no del movimiento del tren. En el
caso de pulsos de ondas en una cuerda que no vara de forma, puede deducirse una
frmula para la velocidad en funcin de las propiedades de la cuerda, que es:

v = _
1


siendo T la tensin en la cuerda (en N) y la inercia de la cuerda, que es el cociente
entre la masa de la cuerda y su longitud; (en
kg
m
).

Otras clasificaciones de ondas.
Estamos ahora en condiciones de agregar otros criterios de clasificacin de
ondas, a los dados al comienzo de esta clase.

ONDAS TRANSVERSALES, LONGITUDINALES Y MIXTAS.
Este criterio surge de comparar la direccin en que se mueven las partculas del
medio con la de la propagacin de la onda. As por ejemplo, en el caso de una cuerda,
mientras la onda se propaga en la direccin de la cuerda misma, masas elementales de la
cuerda se mueven en direccin perpendicular a la cuerda misma. Cuando las direcciones
de la perturbacin y de la onda son perpendiculares, se dice que se tiene una onda
transversal. Son tambin transversales todas las ondas electromagnticas.
En cambio, cuando la direccin de la perturbacin es paralela a la direccin de la
propagacin de las ondas, stas reciben el nombre de longitudinales. El ejemplo ms
importante de este tipo lo tenemos en las ondas sonoras. Cuando vibra un diapasn, o la
cuerda de un violn, ella empuja a las molculas del aire, alternativamente hacia
adelante y atrs, y con ello, las molculas se acumulan generando un aumento de
presin, o se desacumulan, generando una depresin; aparecen as crestas y valles de
presin, que avanzan. As, el movimiento alternativo de las molculas ocurre en la
misma direccin en que avanza la onda sonora.
Queda una tercera posibilidad, que es la onda mixta; ella no es completamente
transversal ni completamente longitudinal, sino que es una combinacin de ambas. Un
ejemplo lo tenemos en la propagacin de las olas en el agua; la siguiente secuencia de
figuras, muestra cmo una partcula de agua (o un pequeo objeto flotando) recorre una
EXTREMO FIJO EXTREMO LIBRE
trayectoria casi circular ante el paso de la ola, sin abandonar su posicin media; al
recorrer el crculo, se mueve tanto hacia adelante y atrs, como hacia arriba y abajo.


ONDAS UNIDIMENSIONALES, BIDIMENSIONALES, TRIDIMENSIONALES.
Aqu se clasifican las ondas por la cantidad de dimensiones en el medio en el cual se
propagan. Son unidimensionales las ondas en cuerdas, bidimensionales las ondas
superficiales en el agua y tridimensionales las ondas sonoras en el aire.

ONDAS VIAJERAS Y ONDAS ESTACIONARIAS. Cuando nuestro ojo es capaz de
percibir el movimiento de una onda, las llamamos viajeras. Pero no siempre ocurre esto;
a veces la interferencia entre una onda directa y su reflejada produce un esquema
resultante particular de onda donde no se puede percibir el movimiento; en este caso las
llamamos ondas estacionarias.

Ondas armnicas, viajeras y unidimensionales.
Ahora dejaremos atrs los pulsos, esas perturbaciones nicas y aisladas y
comenzaremos a estudiar las ondas armnicas, que se caracterizan por ser
perturbaciones continuas y repetitivas en el tiempo. Se llaman armnicas porque las
ecuaciones que las describen contienen invariablemente alguna de las funciones
trigonomtricas seno o coseno. Son generadas por un MAS; as, para producir en la
prctica una onda armnica, puede sujetarse uno de los
extremos de una cuerda tensa a una de las ramas de un
diapasn, de modo que el diapasn vibre en direccin
perpendicular a la cuerda. Al trasladarse esta perturbacin por la cuerda, sta toma la
forma de la figura.
De puntos tales como A y B sucesivos con la misma elongacin (y
A
= y
B
) se dice
que estn en fase. La distancia x que los separa (x = x
B
x
A
) se llama longitud de
onda ().



Longitud de onda es la distancia entre dos crestas sucesivas. Es la distancia que
recorre la onda en el tiempo de un perodo.
A
B
y
A

y
B


y
x
A
0
x
A

x
B

v

Si la onda se propaga con velocidad v, entonces , v y T estn vinculados por la
relacin:
= vT

Deduciremos la expresin matemtica que le corresponde a una onda peridica
armnica como la representada en la figura, que se propaga en la direccin x con
sentido hacia la derecha. Nuevamente estamos frente a una funcin de dos variables: y
= f(x;t). Por las caractersticas de la onda, la expresin deber contener senos o cosenos.

Consideremos el punto P marcado en la
onda en las figuras de la izquierda.
Arriba, para t = 0 y x = 0, P est en el
origen. Este punto describe en el origen
un MAS cuya expresin es
y = Asen t
Un poco despus en el tiempo,
el punto P se habr desplazado hacia la
derecha y lo vemos como en la figura
de abajo. El punto P llega a x
P
con
demora, respecto de x = 0. Esa demora
o retardo, expresado como un desplazamiento angular lo vamos a llamar 0. Luego:
y = Asen (t 0)
Para que aparezca explcitamente la segunda variable (la x), hacemos la
siguiente proporcionalidad:

2n
x
=
0
x
k =
0
x
0 = k x
Luego:
y = Asen (t - kx) (1)

Esta ecuacin me da la forma general de la onda; despus hay que ajustarla para
que cumpla con las condiciones iniciales de cada caso, para lo cual se agrega dentro del
argumento un 0
0
(que generalmente no se escribe expresamente).
La ecuacin (1) con su signo menos corresponde a una onda que se desplaza
hacia la derecha; si fuera hacia la izquierda, sera (+).
La ecuacin (1) tambin puede escribirse como:


y = Asen 2 j
t
1

x
x
[ (2)
Y tanto la (1) como la (2) tambin pueden escribirse permutando las diferencias
contenidas en los parntesis.

Relacin entre , k y v
Para una onda y un medio dados, , k y v son constantes, pero no son
independientes entre s. Encontraremos la relacin que las vincula.
De = vT T =
x


De =
2n
1
I =
2n
o

Igualando los segundos miembros de las de arriba:
x

=
2n
o

x
x
y
y
v
v
P
P
Para t = 0
Para t > 0
0
0
x
P


Intercambiando los extremos:
o

=
2n
x


Pero el segundo miembro es k. Luego:
o

= k
Finalmente:
= kv

Problema 1
Una cuerda tiene un extremo atado a un punto fijo; una persona toma el otro
extremo y lo mueve hacia arriba y abajo senoidalmente con f = 2 Hz y A = 7,5 cm. La
onda se propaga con una rapidez v = 12 m/s. Para t = 0 el extremo libre tiene
desplazamiento positivo mximo y est instantneamente en reposo. Suponiendo que no
hay onda reflejada, a) calcular , T, y k. b) Escribir la ecuacin de la onda. C) Escribir
las ecuaciones de y = f(t) para el extremo de la cuerda que es movido por la persona y
para un punto de la cuerda situado a 3 m de dicho extremo.
Solucin:
a) = 2f = 6,282 = 12,56 rad/s
I =
1
]
=
1
2
= 0,5 s
z =

]
=
12
2
= 6 m
k =
2n
x
=
6,28
6
= 1,05
ud
s


b) y(x;t) = Asen 2 j
t
1

x
x
[ = 0,075 m scn 2n j
t
0,5 s

x
6 m
+
n
2
[
y(x;t) = 0,075 m cos j12,56
ud
s
t 1,05
ud
m
x[

c) En el extremo de la cuerda es x = 0. Sustituyendo este valor de x en la ecuacin
obtenida en b):
y(t) = 0,075 m cos [12,56
ud
s
t
Y para x = 3 m es:
y(t) = 0,075 m cosj12,56
ud
s
t n[
y(t) = - 0,075 m cos [12,56
ud
s
t

Problema 2
Una barra que se mueve hacia arriba y hacia abajo una distancia total de 0,50
cm, genera en un extremo de una cuerda larga horizontal, una onda sinusoidal
transversal. El movimiento es continuo y se repite regularmente 120 veces por segundo.
a) Si la cuerda tiene una densidad lineal de 0,25 kg/m y se mantiene bajo una tensin de
90 N, determinar la amplitud A, la frecuencia f, la velocidad v y la longitud de onda
del movimiento ondulatorio.
Solucin:
La amplitud es la mitad del desplazamiento vertical total; A = 0,25 cm. La
frecuencia es 120 Hz. La velocidad de la onda es:
: = _
F
p
=
_
90 N
0,25
kg
m
= 19
m
s

La longitud de onda es: z =

]
=
19
m
s
120 Hz
= 16 cm

Problema 3
Para el problema anterior, escribir la ecuacin de la onda, suponiendo que se
mueve en el sentido +x y que en el tiempo t = 0 el extremo de la cuerda correspondiente
a x = 0 est en posicin de equilibrio y = 0.
Solucin:
Tomaremos como ecuacin de la onda armnica que va hacia la derecha:
y = Asen (kx t 0
0
)
Como necesitamos que y valga cero para cuando x = t = 0, reemplazando:
0 = Asen (-0
0
) 0
0
= 0. Luego:
y = Asen (kx t)
Calculamos k y
k =
2n
x
=
6,28
16 cm
= 0,39
1
cm
= 2f = 6,28120 Hz = 740
1
s

La ecuacin de la onda es:
y = 0,25sen (0,39x - 740t)
donde x e y estn expresadas en cm y t en segundos.
















































20
Ondas estacionarias.

En la clase 19 estudiamos la propagacin de pulsos y de ondas armnicas a lo
largo de una cuerda, cmo stas se reflejan al llagar al extremo vinculado de la cuerda y
cmo aplicando el principio de superposicin, componiendo la onda directa con la
reflejada, puede obtenerse la onda resultante, en un proceso conocido con el nombre de
interferencia.

En la clase de hoy estudiaremos la propagacin de ondas a lo largo de una
cuerda que se encuentra fija en sus dos extremos, de modo que tales ondas estn
confinadas dentro de un espacio unidimensional limitado. Este tema es importante
porque es lo que ocurre en todo instrumento musical de cuerdas, donde cada una de
ellas se encuentra fija por sus dos extremos. Y tambin vale para el caso de los
instrumentos de viento (como el rgano y el clarinete) donde en sus tubos se producen
reflexiones en ambos extremos.

Tanto en el caso de una cuerda fija en ambos extremos, como en un tubo de
rgano, existen reflexiones en ambos extremos y hay ondas movindose en ambos
sentidos. En estos casos, hay determinadas frecuencias para las cuales la interferencia da
por resultado un esquema vibratorio muy particular al que se denomina onda
estacionaria.

En qu consiste este esquema vibratorio muy particular, es algo que iremos
descubriendo en el transcurso de esta clase. Para comenzar, buscaremos obtener la
ecuacin que le corresponde a una onda estacionaria y para ello aplicaremos el principio
de superposicin de efectos que establece que
y = y
1
+ y
2


La ecuacin de las ondas estacionarias.
El efecto producido en un lugar por 2 o ms trenes de ondas se llama
interferencia. Estudiaremos a continuacin la interferencia producida por una onda
viajera en una cuerda, con su onda reflejada. Supongamos que las ecuaciones de ambas
ondas fueran:
y
1
= Asen (kx + t)
y
2
= Asen (kx - t)
o sea que ambas tienen la misma amplitud A, la misma frecuencia angular y la misma
rapidez. Efectuando la suma algebraica:
y = Asen (kx + t) + Asen (kx - t)
En trigonometra existe una frmula que permite convertir en producto la suma
de los senos de dos ngulos; ella es:
sen + sen = 2sen j
+ [
2
[ cos j
u- [
2
[
siendo: = kx + t
= kx - t
Sumando m. a m. ambas igualdades, se obtiene que:
+ [
2
= kx
Restando m. a m. ambas igualdades, se obtiene que:
u- [
2
= t
Aplicando esta identidad se obtiene:
y = 2Asen kxcos t (1)
La (1) es la ecuacin de una onda estacionaria; en ella A representa la amplitud de cada
una de las ondas viajeras que interfieren. Pero como se ve, la amplitud de la onda
estacionaria es el doble de la amplitud de las ondas viajeras. La (1) podra escribirse as:

y = A
st
sen kxcos t (1)

Una onda estacionaria se nos presenta con caractersticas diferentes a las de una
onda viajera; as, mientras una onda viajera se propaga conservando la amplitud A
constante acompaando su avance, en el caso de la onda estacionaria, la onda
permanece fija en posicin (no se la ve viajar), mientras que la amplitud vara. Esto se
puede verificar matemticamente a partir de la ecuacin (1). Si consideramos un punto
dado de la cuerda (o sea que x permanece fijo el factor sen kx tendr tambin un valor
fijo y la (1) se podra escribir as:
y = Ccos t
Ello nos expresa que cada partcula x realiza en el tiempo un MAS. Y que todas
las partculas hacen lo mismo, con la misma frecuencia . En sntesis:

En una onda estacionaria, la amplitud A tiene un valor diferente para cada partcula x
de la cuerda. O sea: A = f(x).

Esta conclusin marca una primera diferencia entre ondas estacionarias y ondas
viajeras. Una consecuencia de esto es que cuando uno observa una onda estacionaria en
una cuerda, no reconoce que hay dos ondas recorrindola simultneamente con sentidos
opuestos. El ojo no puede seguir tales movimientos y lo que aprecia es algo esttico,
como si estuviera contemplando una fotografa.
Nodos y vientres.
La figura representa uno de los modos posibles
de visualizacin de una cuerda con una onda
estacionaria. Los puntos indicados con N estn
siempre en reposo, tienen amplitud nula siempre.
Tales puntos reciben el nombre de nodos.
A mitad de distancia entre dos nodos
consecutivos hay una V; corresponden a puntos
donde la amplitud es mxima siempre. Tales
puntos reciben el nombre de vientres (o de antinodos).

Para el modo representado en el dibujo, en toda la extensin de la cuerda se
aprecia una sola sinusoide. Podemos decir que la distancia entre extremos es
precisamente . Si acomodamos un eje de referencia graduado en valores de , podemos
sacar las siguientes conclusiones:

POSICIONES DE LOS NODOS.
Los nodos consecutivos estn separados por media longitud de onda.

Sus posiciones son x = 0, , , .. y en general: x =
1
2
n z siendo n un
nmero natural.
POSICIONES DE LOS VIENTRES.
Los vientres consecutivos estn separados por media longitud de onda.

Sus posiciones son: x =
1
4
z,
3
4
z,
5
4
z, .. y en general x = j
2n+1
4
[ z siendo n un
nmero natural.

Qu sucede con la energa en una onda estacionaria? En el caso de las ondas
viajeras, hemos estudiado que ellas transportan la energa de un lugar a otro; esto es
posible porque la amplitud A viaja con ellas. Pero en las ondas estacionarias no sucede
lo mismo. Los nodos, puntos inmviles, se comportan como fronteras que no pueden ser
cruzadas por la energa, la que queda atrapada entre nodo y nodo, sin posibilidad de
transportarse. Esta es otra diferencia ms entre las ondas estacionarias y las viajeras.
Resumiendo las diferencias entre ambas ondas:

ONDAS VIAJERAS ONDAS ESTACIONARIAS
La onda viaja se la ve en reposo
La amplitud A no es funcin de x A = f(x)
La energa viaja y se transporta no viaja.

Problema 1
Una cuerda vibra segn la ecuacin y = 0,5sen j
n
3
x[ cos40 n t donde x e
y estn en cm y t en s. a) Cul es la amplitud y velocidad de las ondas componentes
cuya superposicin da origen a esta vibracin? b) Cul es la distancia entre nodos? c)
Cul es la velocidad de una partcula de la cuerda que est en la posicin x = 1,5 cm
cuando t =
9
8
s?
Solucin:
N V N V N
0
x
4

x
2

3x
4
z
a) Por comparacin entre la ecuacin del enunciado y la ecuacin (1) de las ondas
estacionarias, surge que A = 0,25 cm. Por otro lado, v =
o
k
. y k se toman de la
ecuacin: = 40
1
s
k =
n
3

1
cm
. Reemplazando resulta v = 120 cm/s

b) La separacin entre dos nodos consecutivos es
x
2
donde =
2n
k
= 6 cm
Luego, la distancia entre nodos es de 3 cm.

c) Reemplazando los valores de x y de t en la ecuacin, resulta:
y = 0,5sen 90cos 180.
Luego y = - 0,5 cm. La partcula se encuentra en un extremo de su recorrido; all
su velocidad es nula.

Reflexin de ondas.
Ya hemos estudiado que la forma en que se produce la reflexin de un pulso al
llegar al extremo de la cuerda depende decisivamente de la forma con que la cuerda est
sujetada al vnculo: si es fijo o si es libre.

1 CASO: EL EXTREMO ES FIJO.
Sabemos que en este caso el pulso se invierte. Si en vez de un pulso, fuera un
tren de ondas, ocurrira lo mismo:

La reflexin en un extremo fijo hace que la onda reflejada sufra un desfasaje de 180
respecto de la onda incidente.

Esta oposicin entre las ondas incidente y reflejada en el extremo fijo, hace que
haya entre ellas, interferencia destructiva y que se genere un nodo. Luego:

En todo extremo fijo hay siempre un nodo.

2 CASO: EL EXTREMO ES LIBRE.
Sabemos que en este caso, un pulso no se invierte. Tampoco lo hace una onda
armnica:

En la reflexin en un extremo libre, la onda no sufre cambio de fase.

A diferencia con lo que ocurre en un extremo fijo, ahora habr interferencia
constructiva y se generar un vientre.

En todo extremo libre hay siempre un vientre.

Vibracin de una cuerda que tiene ambos extremos
fijos.
Conforme a lo estudiado, deber haber un nodo en cada extremo. Como la
distancia entre dos nodos consecutivos es
x
2
, la longitud de la cuerda deber ser un
mltiplo de
x
2
. O dicho de otra manera, si la longitud de la cuerda fuera l, podr haber
ondas estacionarias para:
La menor frecuencia con que
puede producirse una onda estacio-
naria se llama frecuencia fundamen-
tal (f
0
). Tambin para frecuencias
mltiplo de sta, ser posible obtener
ondas estacionarias. A todas estas
frecuencias se las denomina armni-
cas, siendo la frecuencia fundamen-
tal, la primera armnica (f
0
= f
1
).
Ntese que el nmero de armnico es
igual a la cantidad de nodos (N)
menos 1. Para calcular los valores de
estas frecuencias, tomamos como
punto de partida la relacin f =

x

donde la velocidad v es la misma
para todas las frecuencias, y
completamos el siguiente cuadro de
clculo:

Frecuencia Longitud (l) Despeje de
Reemplazo en f =

x

Fundamental o
1 armnico
l =
\
2

= 2l
f
0
= f
1
=

2I
= 1 f
0

2 armnico l = = l
f
2
=

I
= 2 f
0

3 armnico
l =
3x
2
=
2I
3
f
3
=
3
2I
= 3 f
0

4 armnico l = 2
=
I
2
f
4
=
2
I
= 4 f
0

5 armnico
l =
5x
2
=
2I
5
f
5
=
5
2I
= 5 f
0

Ensimo armnico f
n
= n f
0


Ntese que estn presentes todos los armnicos y en este caso n representa la
sucesin de los nmeros naturales a partir del 1 y determina el nmero de armnico.
Podemos apreciar una diferencia fundamental entre un sistema constituido por
una masa y un resorte y una cuerda vibrante. El primero posee una sola frecuencia
propia [ f =
1
2n
_
k
m
] mientras el segundo posee infinitas frecuencias propias: las de sus
infinitos armnicos. As, cuando se golpea una cuerda de un instrumento musical, no
solo estn presentes en el sonido emitido el correspondiente a f
0
, sino tambin todos
aquellos armnicos que el odo nos permita percibir.

Problema 2.
Una cuerda de acero de 1 metro de longitud y densidad = 8000
kg
m
3
est tensa
entre dos soportes rgidos. Cuando oscila con su frecuencia fundamental: f
0
= 200 Hz,
a) cul es la velocidad transversal de las ondas en dicha cuerda? b) cunto vale el
esfuerzo de traccin T de la cuerda, en
N
m
2
?
Solucin:
Comenzar por hacer notar que T no es una fuerza sino un esfuerzo; por eso se
una y no .
Fundamental 1 armnico
2 armnico - 1 sobretono
3 armnico 2 sobretono
4 armnico 3 sobretono
5 armnico 4 sobretono
n = 1
n = 2
n = 3
n = 4
n = 5
Para la frecuencia fundamental es = 2l = 2 m.
a) v = f
0
= 2 m200 Hz = 400 m/s.
b) Despejando T de v = _
1
p
T = v
2
= (400 m/s)
2
(8000 kg/m
3
) = 1,28x10
9
N
m
2


Problema 3.
Una cuerda de piano de l = 50 cm y m = 5 g est sometida a una fuerza de 400
N. Hallar a) la frecuencia f
0
de la vibracin fundamental. b) El nmero del armnico
ms alto que puede ser odo por una persona capaz de percibir frecuencias de hasta
10.000 Hz.
Solucin:
Inercia de la cuerda: p =
m
I
=
0,005 kg
0,5 m
= 0,01
kg
m

v = _
1

=
_
400 N
0,01
kg
m
,
= 200
m
s

a) Para la frecuencia fundamental es = 2l = 1 m
f
0
=

x
=
200
m
s

1 m
= 200 Hz
b) Tomando la frmula generalizada: fn = n f
0
y despejando:
n =
]
n
]
0
=
10.000
200
= 50
La persona puede percibir hasta el armnico N 50.

Vibracin de una cuerda que tiene un extremo fijo y
otro libre.

En la prctica una cuerda con sus extremos, uno fijo y el otro libre como indica
el ttulo, puede lograrse armando el dispositivo de la figura:

Cuando las ondas se reflejan, se invierte en el extremo fijo (diapasn), pero no
en el extremo libre.

En la prxima pgina se muestra los modos correspondientes a la frecuencia
fundamental y a sus nueve primeros armnicos.



cuerda hilo muy largo y ligero.
diapasn extremo fijo extremo libre
Polea


Pesa

Para calcular las frecuencias de las distintas armnicas, desarrollamos un cuadro
de clculo, en la misma forma que en el caso anterior:

Frecuencia Longitud (l) Despeje de
Reemplazo en f =

x

Fundamental o
1 armnico
l =
1x
4

z = 4 l
f
0
= f
1
=
1
4I
= 1 f
0

3 armnico
l =
3x
4

z =
4 l
3

f
3
=
3
4I
= 3 f
0

5 armnico
l =
5x
4

z =
4 l
5

f
5
=
5
4I
= 5 f
0

7 armnico
l =
7x
4

z =
4 l
7

f
7
=
7
4I
= 7 f
0

9 armnico
l =
9x
4

z =
4 l
9

f
9
=
9
4I
= 9 f
0

Ensimo armnico f
n
= n f
0


Como se ve, los armnicos pares estn ausentes y en este caso n representa a la
sucesin de los nmeros impares.

Ondas estacionarias en una columna de aire.
Recordemos que mientras las ondas que se propagan en cuerdas son ondas
transversales, las que lo hacen en el aire, son ondas longitudinales. No obstante esta
diferencia, tanto unas como otras, cuando se reflejan en los extremos (de la cuerda o del
tubo), lo hacen en forma anloga. Tambin es anloga la interferencia entre las ondas
que se propagan en sentidos opuestos, y tambin es anloga la circunstancia de que las
ondas estacionarias que se generan tanto en cuerdas como en tubos de aire, poseen un
nmero infinito de frecuencias propias. Tambin hay analoga en la cantidad de casos a
considerar; as como en cuerdas tenamos dos casos (cuerda con ambos extremos fijos y
cuerda con un extremo fijo y otro libre), en las columnas de aire hay dos casos (tubo
abierto en ambos extremos y tubos con un extremo abierto y el otro cerrado). Ambos
casos de ondas estacionarias en tubos, pueden estudiarse siguiendo pasos similares a los
utilizados en cuerdas.
n = 1



n = 3



n = 5


n = 7


n = 9
Simplificando el lenguaje, llamaremos simplemente tubo abierto al que tiene
ambos extremos abiertos; y llamaremos simplemente tubo cerrado al que tiene un
extremo cerrado y el otro abierto.

Diferencia entre cuerdas y tubos de aire: En las primeras hacamos un nico tipo
de representacin grfica (los diagramas de elongacin y = f(x) ). En los tubos de aire
podemos hacer dos tipos de grficos: de elongacin (mostrando cunto se desplazan las
molculas del aire ante el paso de la onda en cada lugar) y de presin (mostrando cmo
vara la presin en el aire ante el paso de la onda, en cada lugar; se llama onda de
presin p = f(x) ).

Se nos presenta una dificultad prctica al intentar trazar los grficos de
elongacin; mientras en las ondas transversales, las elongaciones y son
perpendiculares a la direccin de propagacin de la onda, lo que facilita su
representacin en un diagrama cartesiano x;y, en las ondas longitudinales las
elongaciones y tienen la misma direccin en que se propaga la onda. Por eso, para
dibujar las ondas de elongacin en columnas de aire, se procede a girar en sentido
antihorario 90 a la elongacin y poder as mostrarla en un diagrama cartesiano,
perpendicular a x.

ONDAS DE PRESIN Y DE ELONGACIN EN TUBOS ABIERTOS Y
CERRADOS.

En el extremo abierto del tubo, por encontrarse en contacto con la atmsfera, la
presin en el tubo tiene que tener el valor de la presin atmosfrica. Nunca podr ser
otro valor. Por lo tanto, en la onda de presin tenemos un nodo.
Por otra parte, entre dos nodos consecutivos, y equidistante de ellos, siempre
tiene que haber un vientre. Si el tubo es cerrado, la mayor presin (vientre) se tendr
siempre en el extremo cerrado. Estas consideraciones justifican la forma de la onda de
presin, tanto en el tubo abierto como en el cerrado.

Para razonar la forma de la curva de elongacin, considerar que las fuerzas van
siempre del lugar de mayor presin al de menor presin, es decir del vientre de presin
hacia el nodo de presin. Estas fuerzas desplazan las molculas del medio, de su
posicin de equilibrio promedio, en los sentidos que se muestran en el dibujo; esos
TUBO ABIERTO TUBO CERRADO
ONDA DE
PRESIN
ONDA DE
ELONGAC
N V N
V N V
N V
V N
F F
F
-A
A= 0 +A
A= 0
-A
desplazamientos, girados 90 permiten trazar la curva de elongacin, tanto en el caso
del tubo abierto como en el cerrado.
Resumiendo las observaciones anteriores podemos decir que en tubos de aire,

En el extremo: La onda de presin tiene: La onda de elong. tiene:
abierto nodo vientre
cerrado vientre nodo

Haciendo una analoga entre cuerdas y tubos de aire, podemos decir que un tubo
abierto se comporta de manera anloga a una cuerda con sus dos extremos fijos,
mientras que un tubo cerrado guarda analoga con la cuerda que tiene un extremo fijo y
otro libre.
En las figuras de la pgina anterior solo se han representado los modos
fundamentales para ambos tipos de tubos; pero a ellos pueden agregarse los
correspondientes a sus armnicos, como se hizo con las cuerdas. Los cuadros de clculo
para obtener la expresin de f
n
, hechos en cuerdas, son vlidos tambin aqu, por lo que
podemos poner:
En tubos de aire, f
n
= nf
0
. Si el tubo es abierto, n = nmero natural y f
0
=

2I

Si el tubo es cerrado, n = nmero impar y f
0
=

4I


Problema 4.
Un tubo de rgano cerrado de 60 cm de longitud es recorrido interiormente por
ondas estacionarias de = 0,80 m. a) Graficar la onda de presin e indicar las
posiciones correspondientes a nodos y vientres. b) Si la amplitud es A = 10
-6
cm, cul
ser la elongacin y de una partcula del aire ubicada en el centro del tubo?
Solucin:
a)
b)
2n
x
=
0
x

360
0,80
=
0
0,30
0 = 135

y = Asen 0 = (10
-6
cm)sen 135 = 7,1x10
-7
cm

Problema 5.
Hallar los 4 primeros armnicos de un tubo de 0,20 m, a) si es abierto; b) si es
cerrado; c) Cuntos armnicos puede percibir una persona de odo normal en cada
caso? Considerar v = 340 m/s y mxima frecuencia audible 20.000 Hz.
Solucin:
a) f
0
= f
1
=

2I
=
340
20,20
= 850 Ez
f
2
= 2f
0
= 1700 Hz f
3
= 3f
0
= 2550 Hz f
4
= 4f
0
= 3400 Hz
n =
]
n
]
0
=
20.000
850
= 23 Sc pcrcibcn los 23 primcros ormnicos.
N V N V
0 20 30 40 60
x (cm)
b) f
0
= f
1
=

4I
=
340
40,20
= 425 Ez
f
3
= 3f
0
= 1275 Hz f
2
y f
4
, ausentes.

n =
]
n
]
0
=
20.000
425
= 47 Sc pcrcibcn los 47 primcros ormnicos.


































































21
Acstica

ACSTICA es la parte de la Fsica que trata sobre la produccin y propagacin del
sonido, as como de su relacin con nuestro sentido del odo.

Esta definicin puntualiza que deberemos ocuparnos de 3 aspectos: produccin,
propagacin (ambas, partes objetivas) y de su relacin con el odo (una parte subjetiva).
En realidad mucho de esto se ha venido estudiando en clases anteriores y ahora solo nos
limitaremos a recordar las conclusiones.

Las ondas sonoras son ondas mecnicas, es decir que necesitan de un medio
material para poder propagarse; ellas no se transmiten en el vaco.

Las ondas sonoras son longitudinales, es decir que las partculas del medio se
desplazan en la misma direccin en que se propaga la onda.

El medio por el que se propagan las ondas, puede ser cualquiera: gaseoso,
lquido o slido, pero el valor de la velocidad de propagacin depender de cada medio.
En el aire, esta velocidad es de 340 m/s, valor que puede variar ligeramente segn las
condiciones atmosfricas y la composicin del aire. En general, en los medios lquidos
esta velocidad es mayor y en los slidos, mucho mayor an.

Todos los elementos vibrantes comprimen el aire que los rodea en un
movimiento hacia adelante, y lo rarifican en un movimiento hacia atrs. El aire
transmite estas perturbaciones como una onda a partir de la fuente. Al llegar al odo, las
ondas producen la sensacin del sonido. Las formas de ondas que sean
aproximadamente peridicas, o que estn compuestas por un nmero pequeo de
componentes casi peridicas, producirn a menudo una sensacin placentera (si la
intensidad no es demasiado grande) como por ejemplo los sonidos musicales. El sonido
cuya forma de onda no es peridica se oye como ruido. Ruido es todo sonido
indeseable.
La ecuacin de una onda armnica es: y(x;t) = Asen (kx t)
Generalmente en acstica es ms conveniente tratar con ondas de presin que
con ondas de elongacin; por lo tanto adaptaremos la ecuacin de la onda armnica para
escribirla en trminos de presin:

p (x;t) = Psen (kx - t) (1)

donde P (amplitud de presin) = kv
2
A = vA (2)
siendo: k (nmero de onda) k =
2n
x

(densidad del aire) = 1,20
kg
m
3

v (velocidad del sonido) v = 340 m/s
A (amplitud de la onda sonora)

Problema 1.
La mxima variacin de presin P que puede tolerar el odo en los sonidos
fuertes, es de unos 28 Pa. Calcular el desplazamiento mximo (A) correspondiente para
una onda sonora en el aire que tenga una frecuencia de 1.000 Hz.
Solucin:
Clculos previos: z =

]
=
340
m
s

1.000 Hz
= 0,34 m
k =
2n
x
=
6,28
0,34 m
= 18,47
1
m
,


De la (2): A =
P
kp
2
=
28 Pu
18,471,20340
2
= 1,1x10
-5
m
Este resultado nos ensea que las amplitudes de los sonidos ms ruidosos estn
en el orden de la centsima de milmetro.

Potencia e intensidad de la onda.
La figura muestra una fuente sonora F; desde ella
se propagan en todas las direcciones ondas, y ellas
transportan energa. Esta energa es provista por la fuente
en el tiempo, por lo que podemos decir que la fuente
sonora posee una cierta potencia (P) siendo:

P =
cncgu
tcmpo
=
w
t
j
]ouIc
s
[ = w ( wott)
Las ondas que emanan de la fuente son esferas en
continua expansin, cuyo radio crece con la velocidad del
sonido. Por lo tanto, toda la energa invertida en producir en la fuente un sonido
determinado, se distribuye sobre una esfera y comienza a debilitarse a medida que la
superficie de dicha esfera va creciendo. Recordar que la superficie de una esfera se
calcula con la frmula: A = 4R
2
.
F



2
1
3
:
Se define intensidad de onda (I) como la potencia emitida por la fuente, por unidad de
superficie.

I =
P
A
j
w
m
2
[ I =
w
At

por lo que tambin podemos definir la intensidad de onda I como la energa
transportada por la onda, por unidad de tiempo y de superficie.
Ntese que en la figura, la potencia es la misma en 1, 2, 3, etc pero la intensidad
de onda disminuye: I
1
> I
2
> I
3
> etc.

Supongamos que la fuente sonora fuera una mosca produciendo un fuerte
zumbido (potencia). Segn acabamos de ver, la intensidad de la onda decrece con la
distancia a la mosca. No debemos confundir la intensidad de la onda con la intensidad
del sonido que nuestro odo percibe; en forma aproximada puede considerarse que la
energa que llega a nuestra oreja y que es capaz de producirnos el sonido del zumbido
de la mosca, es la energa que est contenida en un rea de tan solo 1 cm
2
; un rea de 1
cm
2
es insignificante cuando se la compara con el rea total de la esfera que tiene por
radio la distancia mosca-oreja. As por ejemplo, a una distancia de 1,80 m el rea de la
esfera vale ms de 4x10
5
cm
2
; o sea que nuestro odo percibe el zumbido de la mosca
con solo recibir la fraccin
1
400.000
de la potencia de la mosca; (en realidad la fraccin es
menor an, si se toman en cuenta las prdidas de energa que siempre existen). Esto nos
da idea de la extraordinaria sensibilidad del odo humano.
La intensidad de la onda puede expresarse en funcin de las variaciones de presin, ya
que la circulacin de energa en la unidad de tiempo es proporcional al cuadrado de la
variacin de presin. La relacin matemtica es:
I =
P
mx
2
2p

donde P
mx
representa la amplitud de presin. y tiene una expresin dada por la (2).
Tambin puede deducirse una expresin para la intensidad de onda en funcin
de A:
I =
1
2
p :
2
A
2
I = 2 p : ( n A {)
2
(3)
La (3) pone de manifiesto algo muy importante; en ella 2, , v y son constantes; la
intensidad de la onda solo depende de dos cosas: la amplitud y la frecuencia. O sea que I
es proporcional al cuadrado de la amplitud y al cuadrado de la frecuencia. Esto es una
propiedad que poseen todas las ondas armnicas.

PREGUNTA: Dos diapasones producen ondas sonoras en el aire, de la misma amplitud.
Uno de ellos tiene una frecuencia f
1
= 256 Hz y el otro una f
2
= 512 Hz. Cul de ellos
origina el sonido ms fuerte, o sea de mayor intensidad, y cunto vale la relacin de
intensidades entre ambos sonidos?
RESPUESTA: El segundo diapasn origina el sonido ms fuerte. La relacin de
intensidades es:
I
2
I
1
= 4.

Caractersticas del sonido.
Todo sonido posee tres caractersticas: Frecuencia, tambin llamada altura o
tono; timbre e intensidad.

Frecuencia:
El intervalo de frecuencias en las que pueden generarse las ondas mecnicas es
muy amplio; dentro de este amplio rango se encuentran las ondas sonoras; ellas estn
restringidas al intervalo de frecuencias que pueden estimular al odo y al cerebro
humanos para darle la sensacin de sonido. Este intervalo va desde 20 Hz hasta 20.000
Hz y se llama intervalo audible. Se dice que una onda mecnica longitudinal cuya
frecuencia sea menor que la del lmite inferior del intervalo audible es una onda
infrasnica y si su frecuencia es mayor que la del lmite superior del intervalo audible,
se conoce como una onda ultrasnica.

Podemos definir la frecuencia como aquella caracterstica por la cual el odo le
asigna un lugar en la escala musical. Al pulsar una cuerda tensa se emite un sonido de
una frecuencia dada; si se aumenta la tensin de la cuerda, la frecuencia aumenta, el
sonido se hace ms agudo. Si se disminuye la tensin de la cuerda, la frecuencia
disminuye y el sonido se hace ms grave.
Los valores de las frecuencias extremas que determinan el intervalo audible
varan considerablemente de un individuo a otro y en particular el lmite superior
disminuye con la edad del individuo.
La palabra comprende un rango que va de 100 a 8.000 Hz; la msica de una
orquesta comprende un rango de 40 a 14.000 Hz.
Los lmites del intervalo audible son muy variables en los distintos animales; as
el perro escucha sonidos de hasta 50.000 Hz (silbato ultrasnico para perros); los
murcilagos perciben sonidos hasta la frecuencia de 100.000 Hz, frecuencias que
emplean en su sistema de orientacin que funciona de manera similar al radar, ya que
son ciegos.

Timbre:
Esta es otra caracterstica del sonido; tiene que ver con la complejidad de la onda
sonora que llega al odo. Ocurre que salvo el diapasn, que es el nico instrumento que
vibra con una frecuencia nica, cualquier instrumento musical emite un sonido
constituido por una mezcla de varias frecuencias, de forma ms o menos compleja. La
complejidad est regida por la cantidad e intensidad relativa de los armnicos presentes
(un sonido de violn puede contener ms de 10 armnicos). El timbre hace que podamos
distinguir fcilmente una misma nota, si es ejecutada en un piano o en un clarinete, por
ejemplo, o que podamos diferenciar la voz de Juana de la voz de Laura.

Intensidad:
Esta caracterstica del sonido ya ha sido tratada; est ligada a la cantidad de
energa comprometida en la emisin de un sonido, y ello determina la amplitud de la
onda; un sonido de poca intensidad tiene una amplitud pequea.
f (Hz)
0 20 20.000
Ondas infrasnicas Intervalo audible Ondas ultrasnicas
Grandes fuentes
(terremotos)
Cuerdas vibrantes
Columnas de aire
Membranas vibrantes
Vibraciones elsticas
de cristales de
cuarzo.
El sonido y su relacin con el odo.
Nos referiremos a continuacin al tercer aspecto que trata la acstica, conforme
con lo expresado al comienzo de la clase. La sensibilidad del odo no es plana, respecto
de la frecuencia; es decir que no es igual para todas las frecuencias del intervalo audible.
Un odo normal tiene su mxima sensibilidad para frecuencias comprendidas entre
2.000 y 4.000 Hz. La grfica muestra cmo vara la sensibilidad del odo frente a las
distintas frecuencias de sonido. Cabe destacar que la curva que se muestra es suave
porque representa la media para una poblacin de muchos individuos, pero que las
curvas individuales presentan numerosos picos y valles.

El arco superior, desde A hasta B representa el lmite superior, momento en que
el sonido, por su gran intensidad convierte la audicin en una sensacin dolorosa y no se
oye; este lmite est en el orden de 1
w
m
2
. Solamente dentro de la zona grisada, se tiene
sensacin de audibilidad. El arco inferior, desde A hasta B representa el lmite inferior,
donde la intensidad de la onda en el odo es increblemente pequea: 10
-12

w
m
2
. La
relacin entre estos valores extremos de intensidad vale un billn, valor que ilustra
acerca del extraordinario rango de respuesta que posee nuestro odo.

Nivel de intensidad ().
Hasta ahora hemos hablado de intensidad sonido (I); hemos visto que ella es
proporcional al cuadrado de la amplitud y al cuadrado de la frecuencia de la onda. La
intensidad de onda expresa un valor objetivo, es decir que est ms all de cmo lo
perciba cada individuo. Pero a continuacin presentaremos otra magnitud, el nivel de
intensidad (), que a diferencia de la anterior, es una cantidad subjetiva de la que
podramos decir que expresa no lo que es, sino lo que parece ser.

Existe una ley aproximada en medicina, conocida como la ley de Fechner
(Fechner es reconocido como el fundador de la Psicofsica y public esta ley en 1860)
que establece que la magnitud de una sensacin es proporcional al logaritmo de la
intensidad:
S = k l ogI
donde S = Magnitud de la sensacin sonora.
I = Intensidad de la onda
1 10 100 1000 10.000
f (Hz)
(dB) I [
w
m
2
]
120 1
100 10
-2

80 10
-4

60 10
-6

40 10
-8

20 10
-10

0 10
-12

A B
Esta ley es aplicable a cualquiera de los sentidos humanos, sea vista, tacto, odo,
etc. Sin embargo la frmula tal como acabamos de presentarla es incorrecta, ya que no
es posible calcular el logaritmo de un nmero que posee unidades; por eso, para
adimensionalizarlo, se reemplaz I por
I
I
0
quedando finalmente:
S = k l og
I
I
0
(4)
siendo I
0
la intensidad de la onda en el nivel de referencia (umbral inferior). El valor de
I
0
es 10
-12

w
m
2


Si en la (4) se lleva k al primer miembro, al cociente formado [
S
k
se le da el
nombre de nivel de intensidad, y se representa con . Antiguamente, la unidad en que se
expresaba recibi el nombre de Bel en homenaje a Graham Bell cuyos estudios acerca
del sonido son bien conocidos.
[
S
k
= (Bel) = log
I
I
0
(5)
Pero ocurri que para el uso cotidiano, esta unidad result ser muy grande y por
lo tanto, poco prctica. Por esa razn se la reemplaz por un submltiplo, el decibel
(dB) que es la unidad que se utiliza actualmente. De modo que:

[ ( dB)
[ ( BcI)
= 10 [ ( JB) = 10 [ ( Bcl)
E ingresando la (5) en ella, finalmente nos queda:
[ ( JB) = 10 l og
I
I
0
(6)
Si en la (6) se desea despejar I, aplicando la definicin de logaritmo, se obtiene
que:
I = I
0
10

10

(7)
Se puede demostrar que 1 decibel equivale a una variacin de intensidad del
26% y en la prctica representa la mnima variacin de intensidad que puede reconocer
el odo. La demostracin es la siguiente:
Sea I
1
la intensidad de un sonido que en dB tiene un nivel: I
1
= I
0
10

10
(8)
y sea I
2
la de otro sonido que en dB vale ( + 1) I
2
= I
0
10
+1
10
(9)
Dividiendo:
( 9)
( 8)
=
I
2
I
1
=
I
0
10
[+1
10
I
0
10
[
10
= 10
_
[+1
10
-
[
10
_
= 10
0,1


Luego: I
2
= I
1
10
0,1
= 1,26 I
1

I debe incrementarse en un 26 % para que se incremente en una unidad. En el grfico
de la pgina anterior, se tiene una columna con los valores de (dB), calculados
utilizando la (6).






Niveles de intensidad de algunos sonidos comunes (en dB)
Umbral de audicin
Respiracin normal
Biblioteca / Hogar tranquilo
Oficina tranquila
Conversacin normal a 1 m
Trnsito pesado / Esquina
Oficina ruidosa / Fbrica comn
Tren subterrneo
Concierto de rock con amplif. a 1 m
Despegue de avin a reaccin a 60 m
Martillo neumtico/Ametralladora
Despegue de un avin a reaccin
prximo

0 dB
10
40
50
60
70
80
100
120
130
130
150

Escasamente audible
Muy suave
Suave


La exposicin constante hace
peligrar el odo.

Umbral de dolor.

Problema 2.
Una pequea fuente sonora irradia energa acstica uniformemente en todas
direcciones con una potencia de 1,5 W. Hallar la intensidad y el nivel de intensidad en
un punto a 25 m de la fuente si: a) no hay absorcin; b) a lo largo del camino de 25 m
hay una absorcin del 10 %.
Solucin:
a) I =
P
A
=
1,5 w
4n( 25 m)
2
= 1,9x10
-4
w
m
2

[ = 10 l og[
I
I
0
= 10 log [
1,9x10
-4
1x10
-12
= 82,78 JB

b) I = 0,9 1,9x10
-4
w
m
2
= 1,7x10
-4

w
m
2

[ = 10 log [
1,7x10
-4
1x10
-12
= 82,33 JB





















22
Gravitacin

En el mdulo 5 presentamos la ley bsica que rige las interacciones
gravitacionales, esa ley que fuera enunciada por Isaac Newton en 1686 y que
conocemos como la ley de la gravitacin universal. Tales interacciones se originan
entre dos masas materiales cualesquiera; se manifest ante los ojos de Newton cuando
vio cmo caa una manzana del rbol, atrada por la Tierra, y es la misma que se
manifiesta entre la Tierra y la Luna, los planetas y el Sol, la responsable de la evolucin
de las estrellas, la que rige las interacciones entre galaxias, y es tambin la determinante
de la existencia de los intrigantes agujeros negros.
La expresin de la ley de la gravitacin universal es:
F = G
m
1
m
2

1-2
2
(1)
donde m
1
y m
2
representan las masas de los dos cuerpos que interactan, r
1-2
la distancia
que los separa y G es la constante de gravitacin universal, cuyo valor es, en el S.I.
6,67x10
-11

Nm
2
kg
2
. F es la fuerza de interaccin, a la que llamamos fuerza de gravedad.
La figura ilustra la ecuacin (1). La recta
de accin de las fuerzas de gravedad pasa por
los centros de masa de ambos cuerpos; por eso
decimos que la fuerza de gravedad es una
fuerza central; adems es una fuerza
conservativa ya que el valor de su mdulo slo
depende de r.
F


r
1-2
m
1

m
2

La ley de la gravitacin universal marca un hito en la historia del conocimiento
humano de la naturaleza, y abri la puerta a una nueva rama de la ciencia: la Mecnica
Celeste, que se ocupa de la dinmica de los cuerpos celestes. Por ella pudo el hombre
poner en rbita sus satlites artificiales o calcular la trayectoria que se le deba imprimir
a una nave espacial para enviarla a otro planeta.

La masa de la Tierra.
Con la ley de la gravitacin universal y una vez conocido el valor de su
constante G, result sencillo calcular el valor de la masa de la Tierra.

Supongamos tener un objeto cualquiera
cuya masa es m, apoyado sobre la superficie de
la Tierra; el peso del objeto es la fuerza con que
la Tierra lo atrae, y est dado por:
F = mg (2)
siendo g la aceleracin de la gravedad. Si fuera
posible compactar la Tierra hasta convertirla en
una partcula, ella sera un punto situado en el centro de la Tierra donde se reunira toda
la masa M de la Tierra. La situacin real podra cambiarse por el esquema equivalente
de la figura derecha. La distancia que separa ambas masas es el radio R de la Tierra. El
valor del radio de la Tierra es conocido por el hombre desde 250 aos antes de Cristo,
cuando Eratstenes lo midi por primera vez; su valor es R = 6,378x10
6
m. Si
aplicamos la ley de la gravitacin universal:
F = 0
Mm
R
2
(3)
Tanto en la (2) como en la (3) se trata de la misma F. Si igualamos sus segundos
miembros:
mg = 0
Mm
R
2
(4)

M =
gR
2
u

Reemplazando y calculando:
M =
9,8(6,378x10
6
)
2
6,67x10
-11
= 5,98x10
24
kg

El volumen de la Tierra.
Suponiendo que la Tierra fuera una esfera (lo que es una buena suposicin), su
volumen podra ser calculado con la frmula de la geometra:
Vol =
4
3
n R
3
=
4
3
3,14 ( 6,378x10
6
)
3
= 1,087x10
21
m
3


La densidad media de la Tierra.
Una vez conocidos los valores de la masa y del volumen de la Tierra, fue una
tentacin calcular el valor de la densidad media (
m
):

m
=
M
v
=
5,98x10
24
1,087x10
21
= 5500
kg
m
3

Este valor de la densidad media de la Tierra sorprendi. Es 5,5 veces mayor a la
densidad del agua, elemento que ocupa el 80% de la superficie terrestre. Si se agrega
que el 20% restante est conformado principalmente por rocas y arenas cuyas
densidades medias son inferiores a los 3000
kg
m
3
debe sacarse como conclusin que el
Tierra

M
m m
M
R
F
interior de la Tierra contiene materiales cuya densidad debe ser notablemente superior al
valor medio de 5500
kg
m
3

El valor de la constante de gravitacin universal G fue determinado
experimentalmente por Cavendish en el ao 1798. Resulta interesante destacar cmo,
gracias al valor de G, fue posible realizar los clculos anteriores, los que nos
permitieron saber que la Tierra no tiene una densidad uniforme, sino que sta aumenta a
medida que nos acercamos al centro. Trabajos recientes estiman que la mxima
densidad (que se alcanza en el centro de la Tierra) es cercana a los 13.000
kg
m
3


Variacin de | g| con la altura.
El valor de la aceleracin de la gravedad terrestre que habitualmente usamos, de
9,8 m/s
2
es un valor medio vlido para la superficie terrestre. Depende de la distancia al
centro de la Tierra y de la densidad local del manto y ncleo de ella. En realidad la
Tierra no es una esfera perfecta, sino que est levemente ensanchada en el Ecuador y
aplastada en los polos, existiendo una diferencia entre los radios ecuatorial y polar de
unos 21 km. Esto hace que una persona parada en uno de los polos, est ms cerca del
centro de la Tierra de lo que lo est otra, parada en el Ecuador. La gravedad vale ms en
los polos y menos en el Ecuador. Una misma persona pesa ms en los polos que en el
Ecuador. Adems vara localmente con la composicin del subsuelo, si hay napas de
petrleo, por ejemplo.
Si abandonamos la superficie terrestre y nos elevamos por sobre ella, el valor de
la aceleracin de la gravedad disminuye; deduciremos una expresin que nos permita
calcular el valor que toma | g| en funcin de la altura h, medida con relacin a la
superficie terrestre.
La igualdad (4) se dedujo para la superficie terrestre:

mg = 0
Mm
R
2
(4)
Si al cuerpo testigo m lo alejamos una altura h sobre la superficie terrestre, esta
igualdad se escribir como:
mg
h
= 0
Mm
( R+h)
2
(5)
donde g
h
representa el valor de la gravedad a la altura h. Dividiendo miembro a
miembro las (4) y (5):
g
g
h
=
( R + )
2
R
2

Despejando g
h
: g
h
= g j
R
R+h
[
2
(6)

La gravedad alcanza su mximo valor en la superficie terrestre; disminuye si nos
alejamos de ella, segn la ecuacin (6) y tambin comprobaramos que disminuye si
emprendiramos el viaje al centro de la Tierra que Julio Verne alguna vez imagin,
para terminar valiendo cero justamente en el centro de la Tierra.

Problema 1.
Un satlite artificial orbita la Tierra a 1000 km de altura sobre su superficie.
Determinar su velocidad tangencial y su perodo de revolucin.
Solucin:
- Clculo de g
h
a 1000 km de altura.
g
h
= g _
R
R +
_
2
= 9,8 _
6,378x10
6
7,378x10
6
_
2
= 7,328
m
s
2


- Clculo de la velocidad tangencial.
El peso del satlite es la fuerza centrpeta. Luego:
P
h
= Fc
m g
h
= m

2
R
T
+ h
v = g
h
( R + )
v = 7,328 ( 7,378x10
6
) = 7353
m
s

- Clculo del perodo.
I =
2n
o
donde =

( R+h)

T =
2n( R+h)

=
6,287,378x10
6
7353
= 6304 s = 145m

El movimiento planetario. Leyes de Kepler.
Johannes Kepler fue un astrnomo y filsofo alemn nacido el 27 de diciembre
de 1571 y fallecido en 1630; fue ayudante del astrnomo Tycho Brahe en su
observatorio de Praga. Descubri regularidades importantes en el movimiento de los
planetas, las que expuso en tres enunciados conocidos actualmente como las leyes de
Kepler del movimiento planetario. Vale destacar que el matemtico y fsico ingls Isaac
Newton se bas en las teoras y en las observaciones de Kepler para formular su ley de
la gravitacin universal, como lo detallamos al final de este mdulo.
Las leyes de Kepler hacen una descripcin cinemtica del movimiento
planetario. Son ellas:

1- LEY DE LAS RBITAS.

Todos los planetas describen rbitas elpticas con el Sol en uno de sus focos.


2- LEY DE LAS REAS.

La lnea que une un planeta con el Sol, barre reas iguales en tiempos iguales. Esto
puede expresarse diciendo que la velocidad areolar es constante.

En la figura, si los tiempos entre las posiciones P
1
-P
2
y P
3
-P
4
son iguales,
entonces las reas sombreadas son iguales.

:
T
S
m g
h

m
Planeta
Sol
F
1
F
2

Sol
P
1

P
2

P
3

PRIMERA LEY SEGUNDA LEY
P
4

La figura muestra un planeta que se mueve en su rbita alrededor del Sol. Como
el sistema solar es un sistema aislado, se verifica que
ext
= 0, por lo que el momento
angular (I

) permanece constante. El vector I

tiene direccin perpendicular al plano


determinado por r y por :. Esto implica las siguientes dos importantes consecuencias:
1) Al ser I

constante, su direccin no puede cambiar, lo que impide que los planetas


puedan salirse del plano en el cual se mueven, el plano de la rbita.
2) Dado que |I

| = mrv = constante, a medida que el planeta va recorriendo su rbita,


r cambia obligando a que v tambin cambie. De modo que un planeta no se mueve con
velocidad tangencial de mdulo constante, sno que es mayor cuando pasa ms cerca del
Sol (perihelio) y es menor cuando est ms lejos de l (Afelio).

3 -LEY DE LOS PERODOS.
Si entre el planeta y el Sol, aplicamos la ley de la gravitacin universal, podemos
poner:
F = 0
Mm
R
2

siendo M y m las masas del Sol y del planeta. Esta F es la fuerza centrpeta Fc del
movimiento. Entonces: Fc = F
m
2
r = 0
Mm
R
2

Simplificando las m y reemplazando por
2n
1
queda:
_
2 n
I
]
2
=
0 H
r
3

4n
2
0 H
=
I
2
r
3

Todos los factores del primer miembro son constantes, luego:
1
2

3
= ctc. Esto es lo que
expresa la tercera ley de Kepler.

El cuadrado del perodo T de cualquier planeta en torno al Sol es proporcional al cubo
de la distancia promedio del planeta al Sol.

Si 1 y 2 son dos planetas cualesquiera, podr escribirse:

I
1
2
r
1
3
=
I
2
2
r
2
3
= constontc

El valor de esta constante es:
4n
2
uM
=
4( 3,14)
2
( 6,67x10
-11
) ( 2x10
30
)
= 2,96x10
-19

s
2
m
3



Sol
r
:
m
Planeta
Afelio Perihelio
Energa potencial gravitatoria.
Cuando en el mdulo 7 presentamos el concepto de energa potencial
gravitatoria, dimos la expresin Ep
g
= mgh, supusimos que g era constante; esa
expresin result til, en tanto y en cuanto el cuerpo se elevara un h lo suficientemente
pequeo como para que esa suposicin fuera vlida. Pero ahora que sabemos que g
vara a medida que nos alejamos de la superficie terrestre, si el h es muy grande, los
cambios en g no pueden ser ignorados. La frmula dada de Ep
g
ya no nos servir y
deberemos elaborar una nueva expresin.
En el caso general, cuando dos cuerpos (como los 1 y 2 de la figura) se separan
venciendo la fuerza de atraccin gravitatoria existente entre ellos, hay que tener en
cuenta la variacin de la fuerza, cuyo valor es inversamente proporcional al cuadrado de
la distancia, al momento de calcular el trabajo efectuado, trabajo cuyo valor es igual a la
variacin de la energa potencial del sistema.
La figura muestra dos masas m
1
y m
2
, separadas inicialmente una distancia r.
Supongamos a m
1
fija en el origen del sistema de referencia y a m
2
en la posicin r
1
. La
masa m
1
ejerce sobre m
2
la fuerza:
F
1
= -G
m
1
m
2

2

que por tener sentido hacia la izquierda, es negativa. La fuerza externa F
ext
que debe
ejercerse sobre m
2
para alejarla de m
1
es:

F
ext
= - F
1
= G
m
1
m
2

2

El trabajo realizado por esta fuerza al desplazar a m
2
desde la posicin r
1
hasta la
r
2
es:
W = F
cxt
Jr = 0 m
1
m
2

1

d

2
= 0 m
1
m
2
j
1

1


W = G m
1
m
2
j
1

1

1

2
[
Puesto que el trabajo es igual al incremento de la energa potencial,

W = Ep
r2
Ep
r1
= = G m
1
m
2
j
1

1

1

2
[ (7)
Es habitual tomar como nivel de referencia para el clculo de la Ep, el nivel del
mar; sin embargo aqu, en la teora general de la gravitacin conviene tomar como nivel
de referencia al infinito; o sea Ep = 0 cuando el cuerpo est en el . Si tomamos as a la
referencia, en la (7) es r
2
= y Ep
r2
= 0. Resulta:

-Ep
r1
= = G m
1
m
2
j
1

1

1

[
Y como r
1
representa cualquier distancia, podemos prescindir del subndice y
escribir:
0
r
1

r
F
1

F
cxt


m
2
m
1

r
2
x
Sistema de referencia
Ep = -G
m
1
m
2

(8)

La (8) es la expresin de la energa potencial de una masa m
2
situada a una
distancia r de otra masa m
1
. Qu significa el signo negativo? Que Ep
r
< Ep

.

La energa potencial gravitatoria es siempre negativa.

En el diagrama se muestra la
grfica de Ep
g
= f(r) para el sistema
constituido por la Tierra (masa M y
radio R) y un cuerpo cualquiera de
masa m a distintas distancias.
Ntese que el origen del
sistema de referencia se sita en el
centro de la Tierra.






Rapidez de escape.
La ecuacin (8) nos permite calcular por ejemplo, la Ep de una partcula de masa
m en la superficie de la Tierra (masa M y radio R):

Ep = 0
Mm
R

El trabajo necesario para llevar este cuerpo desde la superficie terrestre hasta el
infinito es 0
Mm
R
. Si a un proyectil que se encuentra sobre la superficie terrestre le
comunicamos ms energa que sta, entonces, despreciando la resistencia de la
atmsfera terrestre, escapara de la Tierra para nunca ms volver. Al suceder esto, su Ec
decrece en tanto que su Ep aumenta, pero su rapidez nunca se reduce a cero. La rapidez
de escape que hace posible que el proyectil no regrese a la Tierra, se obtiene a travs del
principio de la conservacin de la energa. Para ello planteamos:
Ep
1
+ Ec
1
= Ep
2
+ Ec
2
.
donde 1 representa un punto sobre la superficie terrestre y 2 un punto en el infinito.
Reemplazando:
-0
Mm
R
+
1
2
m :
csc
2
= 0 + 0


1
2
m :
csc
2
= 0
Mm
R

Desde aqu se despeja la expresin de la rapidez de escape.

v
esc
= _
2uM
R
= _
2( 6,67x10
-11
) ( 5,98x10
24
)
6,378x10
6
= 11.184
m
s
= 11,18
km
s

Si un proyectil adquiere una rapidez de 11,18 km/s o ms, escapa de la Tierra,
pero si su rapidez es menor de 11,84 km/s retornar, ya que su Ec se agotara a una
distancia finita de la Tierra.
r
Tierra

M
Ep
0
R
0
H m
R

0
H m
r

Debido a la agitacin trmica, las molculas ligeras de la atmsfera superior de
la Tierra pueden alcanzar una rapidez tal que les permita escapar al espacio exterior. El
hidrgeno que alguna vez en el pasado abund en nuestra atmsfera, ya ha desaparecido
de ella. Actualmente el helio, escapa de nuestra atmsfera con una rapidez constante y
se encuentra en vas de desaparecer.
En el Sol, la rapidez de escape es mucho mayor y el hidrgeno no puede
escapar de su atmsfera. En la Luna, la rapidez es tan pequea que prcticamente no
puede retener sobre ella ninguna atmsfera.

Deduccin de la ley de la gravitacin universal.
Newton dedujo la Ley de la Gravitacin Universal, a partir de la tercera ley de
Kepler. Veamos qu pasos sigui.
La tercera ley de Kepler establece que para cualquier planeta del sistema solar se
cumple que:

1
2

3
= k (9)
siendo T de perodo del planeta (tiempo para dar una vuelta completa alrededor del Sol),
r la distancia media del planeta al Sol y k una constante.
Si la trayectoria del planeta alrededor del Sol fuera una circunferencia, sera
T =
2n

siendo v la rapidez del planeta. Reemplazando en la (9):



j
2nr

[
2

3
= k :
2
=
4n
2
k
(10)
Como el planeta describe una rbita circular, recibe de parte del Sol, la accin de una
fuerza centrpeta:
Fc = m
P
[


donde m
P
es la masa del planeta. Reemplazando v
2
por la (10):
Fc = m
P
[
4n
2
k
2
(11)
sta es la fuerza ejercida por el Sol sobre el planeta (proporcional a m
P
). Por el
principio de accin y reaccin, otra fuerza igual y opuesta ser ejercida por el planeta
sobre el Sol; en ella deber aparecer la masa del Sol (m
S
). Para que la simetra del
principio de accin y reaccin se haga patente en la frmula (11), convendr que en ella
aparezca el factor m
S
. Entonces la constante
4n
2
k
de la (11) la escribiremos como Gm
S
,
siendo G una nueva constante que se llamar constante de gravitacin universal.
Entonces la (11) quedar as:
F = G
m
1
m
2

1-2
2



















23
Hidrosttica
La materia se encuentra en la naturaleza en tres estados: slido, lquido y
gaseoso. Hasta aqu hemos estudiado conceptos de la fsica aplicables estrictamente al
estado slido. Los lquidos y los gases presentan algunas caractersticas tales que
justifican un estudio particularizado. Por eso, estas ltimas clases estn dedicadas
exclusivamente a ellos. Una denominacin que incluye tanto a los lquidos como a los
gases es la de fluidos.

Los fluidos desempean un papel crucial en muchos aspectos de la vida
cotidiana. Los bebemos, respiramos, y nadamos en ellos; circulan por nuestro
organismo y controlan el clima. Los aviones vuelan a travs de ellos y los barcos flotan
en ellos. Un fluido es cualquier sustancia que puede fluir.

Se llama HIDROSTTICA a la parte de la mecnica que estudia a los fluidos en
reposo, en condiciones de equilibrio.

El nombre de hidrosttica resulta inapropiado ya que parece referirse
exclusivamente al estado lquido, lo que obligara a utilizar otro trmino (neumosttica)
para los gases; slo se conserva por inercia histrica. El nombre correcto debe ser
esttica de los fluidos.

Densidad ()
Se define a la densidad como la razn entre la masa de un cuerpo y su volumen:

=
m
v
En el S.I. j
kg
m
3
[
Es una propiedad importante de cualquier material. Cuando ste tiene el mismo
valor de densidad en todas sus partes, se dice que es un material homogneo. Cada
sustancia tiene un valor de densidad que le es propia. En la siguiente tabla, las
densidades estn expresadas en
kg
m
3


MATERIAL MATERIAL MATERIAL
Agua pura 1000 Hierro 7800 Plata 10500
Agua de mar 1030 Latn 8600 Platino 21400
aluminio 2700 Mercurio 13600
Hielo 920 Oro 19300

Llamamos densidad relativa a la razn entre la densidad de un material
cualquiera y la del agua; es adimensional. Por ejemplo, la densidad relativa del hierro
vale 7,8.

La medicin de la densidad es una tcnica analtica no destructiva importante.
Por ejemplo, nos permite saber acerca del nivel de carga de la batera de un automvil;
la densidad de la solucin de H
2
SO
4
debe dar 1300
kg
m
3
si la carga est completa, o 1150
kg
m
3
si est totalmente descargada.

El peso especfico () guarda con la densidad, la misma relacin que el peso
guarda con la masa:
Si P = mg = g

Presin (p)
Tal vez la diferencia ms importante entre los slidos y los fluidos est en que
estos ltimos nicamente pueden soportar la accin de fuerzas perpendiculares a su
superficie. Si se aplica una fuerza inclinada sobre la superficie libre de un lquido en
reposo, ella podr descomponerse en una componente tangencial y otra normal; la
primera har resbalar las lminas de lquido superficiales entre s, quebrndose el estado
de reposo. Por eso:

Los fluidos no resisten fuerzas tangenciales o esfuerzos de corte.

Por este motivo se define una magnitud nueva: la presin. Ella representa el
valor de la fuerza normal que acta por unidad de superficie:
p =
P
l
S

Tanto la fuerza como la superficie son cantidades vectoriales que en este caso
siempre tendrn la misma direccin. Por esta razn la presin pierde toda caracterstica
vectorial.

La presin es una cantidad escalar.

UNIDADES DE PRESIN: En el S.I. Pascal (Pa) 1 Pa =
1 N
1 m
2

Otras unidades antiguas, an en uso, y su equivalencia con el Pa:
Bar 1 bar = 10
5
Pa
Milibar 1 mbar = 100 Pa
Atmsfera 1 atm = 101.325 Pa
mmHg o Torr 1 mmHg = 133,3 Pa

Cuando se desea aumentar el valor de una presin, se dispone de dos caminos:
aumentar F o disminuir S. Generalmente esta segunda opcin es la ms prctica.
Citar aqu los ejemplos: 1- cortar un pan de manteca con un cuchillo afilado.
2- clavar un clavo de punta y de cabeza
3- caso del faquir
4- La enorme presin (30 kgr/cm
2
) que haca la pa sobre el
disco, en los tocadiscos antiguos.

La presin dentro de una masa lquida.
Recibe el nombre de manmetro, todo instrumento capaz de medir presiones.
Supongamos tener un recipiente conteniendo agua y un manmetro. Experimentalmente
es posible hacer las siguientes comprobaciones:

1- Si recorremos con el sensor del manmetro puntos de un plano horizontal a cualquier
profundidad dentro del lquido del recipiente, mediremos la misma presin en todos sus
puntos.

2- Si recorremos con el sensor del manmetro puntos de un plano vertical, mediremos
presiones mayores conforme aumente la profundidad. Es decir que a peso especfico
constante, la presin es proporcional a la profundidad.

3- Si recurrimos ahora a varios recipientes, donde en cada uno hemos puesto un lquido
diferente (con una densidad distinta), tomando las presiones en cada recipiente, siempre
a igual profundidad, mediremos valores diferentes. Se podr establecer que tales
valores, a profundidad constante, son proporcionales a las densidades.

Todas estas observaciones nos conducen a que la presin en el interior de una
masa lquida es proporcional simultneamente al peso especfico y a la profundidad:
p
H
= gh (1)
Esta presin se llama presin hidrosttica. Alcanza su mximo valor en el fondo
del recipiente, y para un lquido determinado, su valor depende nicamente de la
profundidad h; es independiente de la masa de lquido. Esta circunstancia se conoce con
el nombre de paradoja hidrosttica.

Si a la presin hidrosttica, que es la generada por el lquido, le agregamos la
presin atmosfrica, ejercida por el aire sobre la superficie libre del lquido, obtenemos
la presin total, que es llamada presin absoluta:
p
abs
= p
atm
+ gh (2)
p
H
pa
bs

h h
pa
tm

La expresin anterior es conocida con el nombre de teorema general de la
hidrosttica. El grfico de arriba muestra cmo la presin crece linealmente con la
profundidad, hecho que comprobamos cuando nos sumergimos en aguas profundas; all
los odos nos indican que la presin aumenta rpidamente al aumentar la profundidad.

Si en la expresin (2): p
abs
= p
atm
+ gh ordenamos trminos:
p
abs
- p
atm
= gh
la diferencia de presiones del primer miembro recibe el nombre de presin
manomtrica; representa el exceso de presin existente con respecto a la presin
atmosfrica. El motivo del nombre est en que sta es normalmente la presin que
miden los manmetros. As por ejemplo, si la presin dentro de un neumtico es igual a
la presin atmosfrica, el neumtico est desinflado: la presin debe ser mayor que la
atmosfrica para poder sostener al vehculo, as que la cantidad significativa es la
diferencia entre las presiones interior y exterior. Un manmetro mide justamente dicha
diferencia; entonces:
p
abs
- p
atm
= p
manomtrica
.

Como se ve, la presin manomtrica es igual a la presin hidrosttica:
p
man
= p
H
.

Problemas.
1- Un barril contiene una capa de aceite (
ac
= 600 kg/m
3
) de 12 cm de espesor, sobre 25
cm de agua. a) Cunta presin manomtrica hay en la interfase aceite-agua? b) dem en
el fondo del barril.
Solucin:
a) p
man A
=
ac
gh
ac
= 6009,80,12 = 705,6 Pa
b) p
man B
= p
man A
+
ag
gh
ag
= 705,6 + 10009,80,25 = 3155,6 Pa

2- En el manmetro de la figura, a) Cunta presin absoluta hay en la base del tubo en
U? b) Y en el tubo abierto, 4 cm
debajo de la superficie libre? c)
Cunta presin absoluta tiene el aire
del tanque? d) Cunta presin
manomtrica tiene el aire de la ampolla
en Pa?
Solucin:
980 milibares = 98.000 Pa
a) p = p
atm
+ gy
2

p = 98.000 + 13.6009,80,07 =
107.330 Pa

b) p = p
atm
+ g(y
2
y
1
) = 98.000 + 13.6009,80,04 = 103.331 Pa

c) La p
abs
calculada en b) es la misma p
abs
que hay en la superficie libre de Hg en la
rama de la izquierda. Por lo tanto la respuesta es 103.331 Pa.

d) p
man
= p
abs
p
atm
= (103.331 98.000)Pa = 5.331 Pa

3- Un cortocircuito deja sin electricidad a un submarino que est 30 m bajo el nivel del
mar. Para escapar, la tripulacin debe empujar hacia afuera una escotilla en el fondo,
Aire
y
1
= 3 cm
y
2
= 7 cm
Patm = 980 milibares
Hg
que tiene un rea de 0,75 m
2
y pesa 300 N. Si la presin interior es de 1 atm, cunta
fuerza hacia abajo se debe ejercer sobre la escotilla para abrirla?
Solucin:

F = F
ext
F
int
P = [p
ext
p
int
]S P
= [(
ag
gh + p
atm
) (p
atm
)]S P
=
ag
ghS P = (10309,8300,75 300)N
= 226.815 N

Vasos comunicantes.
A- CON UN MISMO LQUIDO.
Hemos estudiado que dentro de una misma masa lquida,puntos a igual
profundidad h, tienen la misma presin. Esta es una frase tpicamente reversible, donde
si permuto las posiciones de las palabras profundidad y presin, la nueva afirmacin,
tambin es vlida:
Puntos a igual presin, deben estar a la misma profundidad.

Esto puede comprobarse en el dispositivo de la figura, donde el lquido alcanza
la misma altura en todas las columnas, sin
importar cul sea su forma o tamao. Esto
explica tambin por qu la superficie libre de
los lquidos siempre est perfectamente
nivelada. Si uno de los vasos de la figura
tuviera una altura de lquido diferente a la de
los dems vasos, la presin en el fondo sera
diferente a la de los dems vasos, lo que provocara una circulacin de lquido dentro
del recipiente, que slo cesara cuando tales presiones se igualen.

En los vasos comunicantes el nivel alcanzado por el lquido en reposo es el mismo en
todos los recipientes, independientemente de su tamao y forma.

B- CON DOS LQUIDOS NO MISCIBLES.
Si se introducen dos lquidos no miscibles, como
por ejemplo A = agua y B = mercurio dentro del tubo en
U de la figura, los niveles de las superficies libres de
ambas ramas son diferentes. No se cumple el lema
anterior, porque ahora ya no tenemos un nico lquido.

Los puntos 1 y 2 estn al mismo nivel, el 1 en la
interfase A-B y el 2 en la otra rama, dentro de B. Los
dos puntos estn dentro de un mismo lquido y a un
mismo nivel; por lo tanto: p
1
= p
2

Desarrollando: p
atm
+
A
gh
A
= p
atm
+
B
gh
B
Ordenando:

h
A
h
B
=
p
B
p
A


Las alturas alcanzadas por las superficies libres de los dos lquidos medidas respecto
del plano horizontal que pasa por la superficie de separacin entre ambos, son
inversamente proporcionales a sus pesos especficos.
escotilla
F
int
F a aplicar
F
ext
Peso
Lq. A
Lq. B
h
A

h
B

p
atm
p
atm

1 2
Principio de Pascal.

La sobrepresin ejercida sobre un lquido en equilibrio se transmite a todos los puntos
de la masa lquida con igual valor.

Se entiende por sobrepresin a la
diferencia entre la presin final y la presin
inicial. Las presiones que indican los man-
metros en la figura, son diferentes, ya que estn
ubicados a distintas alturas; no obstante, las
sobrepresiones son iguales.

El cientfico francs Blas Pascal recono-
ci este hecho en 1653. La principal aplicacin
de este principio est en la prensa hidrulica, un
dispositivo que permite multiplicar la fuerza. De hecho, Pascal construy la primera
prensa hidrulica.

I los mbolos estn a igual nivel, entonces los
puntos 1 y 2 estn a la misma presin:
p
1
= p
2

Reemplazando,

P
1
S
1
=
P
2
S
2
F
2
= F
1
[
S
2
S
1

Siendo el cociente del parntesis mayor que 1, en esa
relacin se ver multiplicada la fuerza. Luego F
2
> F
1
.

Los elevadores hidrulicos para automviles, los frenos hidrulicos, las
compactadoras y las estampadoras son ejemplos de dispositivos basados en este
principio.

La fuerza de empuje (F

) y su punto de aplicacin.
Supongamos tener un recipiente de forma prismtica, conteniendo un lquido;
cada una de las caras laterales del recipiente soporta una fuerza distribuida ejercida por
el lquido, en contra de ellas. La resultante de esa fuerza distribuida recibe el nombre de
fuerza de empuje (E

):
E = p
med
S
donde como p no es constante en toda la cara lateral sino que se incrementa linealmente
con la profundidad, debemos poner una p
media
tal que p
med
=
1
2
p g .
Por otra parte, si se trata de una cara rectangular, ser: S = hl. Reemplazando:
E =
1
2
p g h
2
l
Las coordenadas del punto de aplicacin de esta fuerza son:
Centro de empuje: (
1
2
l;
2
3
)

Principio de Arqumedes.
Arqumedes naci en Grecia en el -278 y muri en el -212 cuando un
destacamento de soldados romanos irrumpi en su casa y le dio muerte. Vivi en
F S
p = 4
1 - 5
2 - 6
3 - 7
4 - 8
F
1
F
2

S
1
S
2

1 2
Siracusa, capital de la colonia griega en Sicilia. Descubri el principio de la palanca y
sus diversas aplicaciones, las que produjeron gran sensacin en el mundo antiguo.
Pero probablemente su descubrimiento ms importante haya sido su ley sobre la
prdida aparente de peso que sufren los cuerpos sumergidos dentro de un fluido.

Desde muy antiguo era un hecho conocido pero
inexplicable que cuando se sumerge un cuerpo en agua, parece
pesar menos. Si P = mg y tanto m como g se mantienen
invariablemente constantes, por qu disminua P?
Fue Arqumedes quien trajo la respuesta; el lquido aplica
al cuerpo una fuerza en la misma direccin del peso y con
sentido opuesto, de modo que el mdulo de la
resultante se reduce. Actualmente damos a esta fuerza
los nombres de fuerza de flotacin o de empuje.

E = P - P
ap


Pero Arqumedes fue ms all. Encontr que el valor de la fuerza de
flotacin o de empuje (mdulo de E

) poda calcularse de
otra manera. Si se llena el recipiente de la figura de la derecha
hasta enrasar con su pico de derrame y se sumerge luego el
cuerpo, recogiendo el lquido derramado, el peso de dicho lquido
derramado es el valor de la fuerza de flotacin:
P
lq. derramado
= E
Por lo tanto:
E =
l
gV
cs
.
El enunciado del principio de Arqumedes es:

Si un cuerpo est parcial o totalmente sumergido en un fluido, ste ejerce una fuerza
hacia arriba sobre el cuerpo, igual al peso del fluido desplazado por el cuerpo.

Problemas.
1- Una pesa metlica de 400 N tiene un volumen de 5x10
-3
m
3
. Por medio de una
cuerda, se la suspende dentro de un lquido de 760 kg/m
3
de densidad. Calcular los
valores a) del empuje E; b) de la tensin T en la cuerda.
Solucin:
a) E =
l
gV
cs
.= 7609,85x10
-3
= 37,24 N
b) T = P E = (400 37,24)N = 362,76 N

2- Un objeto pesa 500 N en el aire y 450 N cuando est totalmente sumergido
en agua. Hallar a) el volumen de la pieza; b) la densidad de la aleacin.
Solucin:
E = P Pa = (500 450)N = 50 N
a) E =
l
gV
cs
. Vcs =
L
p
l
g
=
50
10009,8
= 5,1x10
-3
m
3

b) p
c
=
m
c
v
c
=
P
c
gv
c
=
500
9,85,110
-3
= 10.000
kg
m
3


3- Una plancha de hielo flota en un lago de agua dulce. Cul es el mnimo volumen
que debe tener para que una mujer de 45 kg pueda pararse sin mojarse los pies?
P
P
ap

P
E
P
E
T
Solucin:
Condicin de flotacin: P = E siendo P = P
hielo
+ P
mujer
.

h
gV
h
+ 459,8 N =
l
gV
h

gV
h
(
l

h
) = 441 N
V
h
=
441 N
9,8( 1000-920)
N
m
3
,
= 0,5625 m
3


Tensin superficial.

Es la propiedad que tiene la superficie libre de un lquido (o la superficie de interfase
entre dos lquidos) por la cual se comporta como una membrana en tensin.

Este fenmeno se debe a que las molculas del lquido ejercen fuerzas de
atraccin entre s. La fuerza neta sobre una molcula dentro del volumen del lquido es
cero, pero una molcula en la superficie es atrada hacia el volumen. Por esta razn, el
lquido tiende a reducir al mnimo su rea superficial, tal como lo hace una membrana
estirada.

Algunos ejemplos que ilustran este fenmeno son:

1- Algunos objetos como un clip o un alfiler pueden descansar sobre la superficie libre
del agua, a pesar de tener una densidad muy superior a la del agua.

2- Al introducir una brocha o un pincel en el agua, todas sus cerdas se unen en un haz.

3- Al pasar un peine por debajo del chorro de agua de una canilla, se forma una pelcula
entre diente y diente. Este mismo fenmeno es el que mantiene el cabello bien peinado
cuando est mojado.

4- Las gotas de lluvia en cada libre tienen forma esfrica (no de lgrima), porque una
esfera tiene menor rea superficial para un volumen dado que cualquier otra forma.

5- La formacin de la gota en un gotero.

6- Las pompas de jabn que hacemos soplando una pelcula de solucin jabonosa
extendida sobre un aro de alambre.

Para hacer un buen lavado de la ropa, es necesario que el agua pueda pasar a
travs de los pequeos espacios que existen en la trama del tejido de la ropa, pero ello es
dificultado por la tensin superficial. Para reducirla, se emplea agua caliente y jabn o
detergentes; ambas cosas ayudan a reducir la tensin superficial.

La formacin de meniscos en la superficie libre de lquidos en tubos, o en
contacto con la pared del recipiente, tambin es consecuencia de la tensin superficial.
Finalmente, si se escucha el sonido que produce el chorro de agua fra y el de agua
caliente al caer sobre la pileta, se notar que esta ltima no golpea con la misma fuerza
que la fra, pues su tensin superficial es menor.

Capilaridad.
Al estudiar vasos comunicantes vimos que el lquido alcanzaba el mismo nivel
en todos los vasos. Del mismo modo, si introducimos verticalmente un tubo dentro de
un recipiente con lquido, el nivel de la superficie libre es el mismo dentro del tubo que
afuera. Sin embargo, si el tubo es muy delgado, es un capilar (delgado como un cabello)
observaremos cosas diferentes. Segn de qu lquido se trate, el lquido sube o baja
dentro del tubo capilar, respecto del nivel en el recipiente. Lo primero ocurre cuando el
lquido es miscible, o sea que moja las paredes del tubo o del recipiente; lo segundo en
cambio, sucede cuando el lquido es no miscible. Este fenmeno se conoce con el
nombre de capilaridad.

Algunos ejemplos que ilustran este fenmeno son:

1- El papel secante, absorbe la tinta derramada, y ella se extiende sobre el papel. Ello se
debe a que las fibras del papel actan como delgados tubos, promoviendo el fenmeno
de la capilaridad.

2- Por el mismo motivo, una gota de caf derramada sobre un terrn de azcar, se
extiende por todo el terrn.

Los fenmenos de capilaridad se contradicen con lo estudiado en la hidrosttica.
La ley de Jurin da una expresin matemtica que describe este fenmeno. Segn ella:
h =
21cos 0
pg

donde h es la altura de lquido (en el ascenso o descenso) dentro del tubo capilar.(m)
T es el valor de la tensin superficial. [
N
m

0 es el ngulo de contacto
r es la medida del radio del tubo capilar. (m)
es la densidad del lquido. [
kg
m
3

g es la aceleracin de la gravedad [
m
s
2


La altura o ascenso capilar alcanzado por un lquido de tensin superficial conocida,
en un tubo capilar, es inversamente proporcional al radio y a la densidad del lquido, a
la vez que es directamente proporcional a su tensin superficial.

Generalmente la ley de Jurin se usa para conocer el valor de la tensin
superficial, despejando T de su frmula.




































24
Hidrodinmica I
(LQUIDOS IDEALES)

Desde hace un tiempo circula por internet un mail que cuenta una interesante
historia que en este momento resulta ideal como introduccin al tema de la clase de hoy.
Por eso lo voy a leer, y como vern enseguida, se trata de una gansada; dice:

La prxima temporada, cuando veas los gansos emigrar dirigindose hacia un
lugar ms clido para pasar el invierno, fjate que vuelan en forma de V, de V corta. Tal
vez te interese saber el por qu lo hacen en esa forma. Lo hacen porque al batir sus alas,
cada pjaro produce un movimiento en el aire que ayuda al pjaro que va detrs de l.
Volando en V, la bandada de gansos aumenta por lo menos un 70% su capacidad de
vuelo, en comparacin con la de un pjaro que vuela solo. Cada vez que un ganso sale
de la formacin, siente inmediatamente la resistencia del aire; se da cuenta de la
dificultad de hacerlo solo y rpidamente vuelve a la formacin para beneficiarse del
compaero que va adelante.

Cuando el lder de los gansos se cansa, se pasa a uno de los lugares de atrs y
otro ganso toma su lugar. Los gansos que van atrs graznan para alentar a los que van
adelante a mantener la velocidad. Finalmente cuando un ganso se enferma o cae herido
por un disparo, otros dos gansos salen de la formacin y lo siguen para apoyarlo y
protegerlo.
Hasta aqu el relato. Justamente cuestiones como sta, que se ocupan del
movimiento de un cuerpo dentro de un fluido, o del movimiento de los fluidos, como las
aguas de un ro, o los remolinos del humo de un cigarrillo en el aire, son las que trata la
hidrodinmica. Encontraremos aqu una respuesta a preguntas tales como por qu
vuelan los aviones? o cmo hace un jugador de ftbol para imprimir a la pelota una
trayectoria curva, y as eludir la barrera y hacer un gol.

HIDRODINMICA es la parte de la fsica que se ocupa del estudio de los fenmenos
vinculados con los fluidos en movimiento.

Como hicimos cada vez que comenzamos un tema nuevo, definiremos a
continuacin una serie de palabras y frases de uso comn en este tema:

En lo que hace al movimiento de los fluidos, ste puede ser de dos formas:
estacionario o no estacionario. Depende de si el vector velocidad : del fluido, en un
punto cualquiera dentro del espacio atravesado por el fluido, se conserva constante en el
tiempo, en todos sus aspectos (mdulo, direccin y sentido) o no. De manera que:
Es estacionario si : = {( x; y; z)
Es no estacionario si : = {( x; y; z; t)

El primero ocurre cuando el fluido se mueve con baja velocidad y se llama
tambin rgimen laminar. El segundo se caracteriza por la presencia de remolinos,
como en los rpidos de un ro, y el : cambia en forma errtica instante a instante y de
punto a punto; a este movimiento tambin se lo llama de rgimen turbulento.

Ambos casos pueden ser comparados con el movimiento de la gente; el rgimen
laminar es como la marcha de los soldados que avanzan en forma ordenada a mientras
desfilan; en cambio el rgimen turbulento es como el de los peatones en una terminal
ferroviaria en hora pico.

Lnea de corriente.
Sean A, B y C tres puntos dentro del recorrido de
un fluido en movimiento. En el rgimen estacionario los
: en cada uno de ellos permanece constante en el
tiempo. La lnea envolvente que hace tangencia con cada
uno de estos :, recibe el nombre de lnea de corriente.

Tubo de corriente.
La parte de lquido limitada por las lneas de corriente, recibe el nombre de tubo
de corriente o filete lquido o vena lquida. Como : es en cada punto tangente a las
lneas de corriente, lo ser tambin a las superficies del tubo de corriente, por lo que las
partculas del lquido no cruzan durante su movimiento las paredes del tubo de
corriente.

Fluido ideal o perfecto.
Para que un fluido sea ideal o perfecto, debe reunir dos requisitos:
a) ser incompresible, es decir que su densidad sea constante, tanto en el espacio
como en el tiempo.
b) ser no viscoso, es decir que carezca de fuerzas de rozamiento interno.
A
B
C
:
A

:
B

:
C

En la clase de hoy estudiaremos la hidrodinmica de los fluidos ideales.

La ecuacin de la continuidad.
Sea un fluido ideal circulando dentro de
un tubo de corriente y consideremos dos lugares
(1) y (2). En un t cualquiera no puede
acumularse ni desacumularse fluido entre las
secciones A
1
y A
2
, ya que de suceder tal cosa, el
fluido variara su densidad, lo que no puede
suceder por tratarse de un fluido ideal. Por lo
tanto, en todo momento la cantidad que ingresa a travs de A
1
debe ser igual a la que
sale por A
2
. Se podr poner:

[
voI
t

1
= [
voI
t

2
v
1
A
1
= v
2
A
2
= Q (caudal) = constante
Esta expresin es conocida con el nombre de ecuacin de la continuidad; la
constante Q recibe el nombre de gasto o caudal y se expresa en m
3
/s.

Esta ecuacin nos muestra que para una misma vena lquida, las velocidades son
inversamente proporcionales a las medidas de las secciones. Si entre dos secciones la
velocidad vara, entonces tenemos una aceleracin. En el
mismo sentido de la aceleracin tenemos una fuerza;
esta fuerza es provocada por una diferencia de presiones.
En la figura derecha:
A
1
> A
2

v
1
< v
2

p
1
> p
2


A mayor seccin corresponde mayor presin y menor velocidad.

Teorema de Bernoulli.
Este teorema se constituye en la ley fundamental de la mecnica de los fluidos;
puede deducirse a partir del teorema del trabajo y la energa. Daniel Bernoulli (1700
1782), naci en Holanda, aunque su verdadera nacionalidad es Suiza; fue profesor de
Matemtica en la Academia Rusa de San Petersburgo; en 1738 dedujo este teorema que
hoy lleva su nombre y que relaciona la presin con la velocidad y la altura para un
fluido circulando en rgimen estacionario dentro de una tubera no horizontal. Este
teorema NO SE CUMPLE en el rgimen no estacionario o turbulento.

A continuacin lo deduciremos para el caso particular de tener un fluido ideal.
La figura de la prxima pgina muestra una vista en seccin de un cierto tramo de
tubera; se destaca una pequea porcin de fluido a la que le haremos su seguimiento
durante su avance a lo largo de la tubera. Los nmeros (1) y 2) indican dos lugares
cualesquiera en la tubera, cada uno caracterizado por tener valores de h, v y A propios.
Supondremos tambin que el fluido se mueve con sentido desde (1) hacia (2).

Aplicaremos la expresin: W
F ext
= Em = Em
2
Em
1

Nos ocuparemos previamente del primer miembro de la expresin de arriba. El
cilindro de lquido de la figura soporta dos fuerzas: una sobre su tapa inferior, ejercida
:
1

:
2

A
1

A
2

(1)
(2)
V
1

V
2
a
A
1

A
2

por el lquido que viene atrs y que realiza
un trabajo positivo desplazando al cilindro
desde 1) hasta (2). La otra fuerza acta
sobre su cara superior y es ejercida por el
lquido que est adelante; el trabajo de esta
fuerza es negativo ya que los sentidos de
dicha fuerza y del desplazamiento son
opuestos. Podemos poner:

W
F ext
= W
Finf
- W
Fsup
.

= F
n]
Jl F
sup

B
C
A
Jl

Desglosando integrales:

W
F ext
= j p A Jl
B
A
+ p A Jl
C
B
[ j p A Jl
C
B
+ p A Jl

C
[
Las integrales del 2 y 3 trmino se cancelan por ser iguales y opuestas; queda:
W
F ext
= p A Jl
B
A
p A Jl

C

Por ser pequeas las respectivas distancias AB y CD, las presiones y reas pueden
suponerse constantes sin error apreciable, con lo que resulta:

W
F ext
= p
1
V p
2
V
Retornando a la frmula del principio y desarrollando cada trmino, ahora
podemos poner:
p
1
V p
2
V = [ mgh
2
+ mv
2
2
] [mgh
1
+ mv
1
2
]

mgh
1
+ mv
1
2
+ p
1
V = mgh
2
+ mv
2
2
+ p
2
V

La igualdad anterior nos dice que la suma de los tres trminos de esta expresin
arroja el mismo resultado, cuando se los calcula para el punto (1) que cuando se los
calcula para el punto (2). Y teniendo en cuenta que estos puntos han sido elegidos
arbitrariamente, deber interpretarse que dicha suma tiene un resultado constante en
cualquier seccin de la tubera donde se la calcule. Por lo tanto, generalizando para una
seccin cualquiera podremos poner:

mgh + mv
2
+ pV = Cte

Dividiendo por V (volumen) todos los trminos de la igualdad:
m g
I
+
m
2 I
: + p = Ctc
Y como
m
v
= p queda finalmente:
pg +
1
2
p :
2
+ p = Ctc

sta es la expresin del teorema de Bernoulli. Cada uno de sus trminos
representa una presin; el primero es la presin hidrosttica, el segundo la presin
cintica y el tercero a la absoluta (no manomtrica), tambin llamada presin esttica.
La suma de los dos primeros trminos suele recibir el nombre de presin
hidrodinmica.
v
2

F
2

(2)
Flujo
v
1

F
1

h
1

h
2

A
1

A
2

(1)
A
B
C
D
El teorema de Bernoulli puede enunciarse as:

En todo fluido ideal en rgimen estacionario, la suma de las presiones hidrosttica,
cintica y esttica, es constante.

Para una mejor comprensin del Teorema de Bernoulli, consideraremos a
continuacin distintos casos a partir de suponer que al orificio de salida de un tanque, se
le ha conectado un tubo largo. Supondremos que el nivel de lquido dentro del tanque se
mantiene (o es mantenido) constante.

1 CASO: LA LLAVE L AL FINAL DEL TUBO LARGO, EST CERRADA.
En distintos lugares a lo largo del tubo se han colocado unos tubos verticales,
largos y delgados, que van a servir para medir la presin; por eso se los llama tubos
manomtricos.

Si la llave L est cerrada, no hay circulacin de lquidos; no se trata de un
problema de hidrodinmica sino de hidrosttica; todo el conjunto de la figura se
comporta como un vaso comunicante, y como hemos estudiado, en todos los vasos el
lquido alcanza el mismo nivel.

Cul es la presin que miden
estos tubos manomtricos, tambin
llamados piezomtricos? Revisemos la
ecuacin de Bernoulli: su primer trmino
proviene de la energa potencial, por lo
que solo aparece cuando existe un h en
la tubera; pero aqu la tubera es horizontal y h = 0. El primer trmino de la ecuacin
de Bernoulli es nulo.

Su segundo trmino contiene la velocidad y proviene de la energa cintica; en el
caso que estamos considerando, el lquido no tiene movimiento y por lo tanto el
segundo trmino de la ecuacin de Bernoulli tambin es nulo.

Solamente nos queda el tercer trmino de la ecuacin de Bernoulli, el de la
presin esttica o absoluta. Por lo tanto, la conclusin es:

Los piezmetros miden presiones estticas o absolutas.

2 CASO: LLAVE L ABIERTA; TUBERA HORIZONTAL DE SECCIN
CONSTANTE.

En el caso anterior, el nivel de los piezmetros era el mismo nivel del tanque;
ahora ese nivel ha bajado por igual en todos los piezmetros. Qu interpretacin le
damos a la lectura de los piezmetros? Recurrimos nuevamente a la ecuacin de
Bernoulli; su primer trmino contina siendo nulo, porque la tubera es horizontal y h
= 0. Pero ahora el segundo trmino no es nulo porque hay energa cintica. Pero como
la suma de los 3 trminos de la ecuacin de Bernoulli debe mantenerse constante, la
presin cintica que aparece lo hace a expensas de la presin esttica.

Como los piezmetros
miden la presin esttica y ella ha
disminuido, el nivel de lquido en
los piezmetros es menor. En la
figura, la altura (a) mide la presin
esttica y la altura (b) mide la
presin cintica.


3 CASO
TUBERA HORIZONTAL DE SECCIN VARIABLE.

El caudal Q a travs de cualquier
seccin debe ser el mismo, ya que si
no fuera as, habra acumulacin o
desacumulacin de lquido entre dos
secciones, lo que no puede suceder
por ser el lquido incompresible. Si en
la tubera de la figura llamamos A
1
y
A
2
a las secciones mayor y menor,
deber cumplirse que
A
1
v
1
= A
2
v
2

Si A
1
> A
2
v
2
> v
1

Si existe variacin en la velocidad, entonces hay aceleracin entre las secciones
(1) y (2); la aceleracin est representada por un vector que va de izquierda a derecha.
Como donde hay aceleracin hay fuerza, sta tambin va de izquierda a derecha; en un
lquido, la fuerza se manifiesta como una presin. Resulta entonces que:

A mayor rea de seccin corresponde mayor presin esttica o absoluta.

Los tubos piezomtricos de la figura confirman esta observacin; recordar que
ellos slo miden la presin esttica; la diferencia entre el nivel del lquido en los
piezmetros y el tanque mide la presin cintica (2 trmino de la ecuacin de
Bernoulli) y ella es mayor en la seccin (2) donde v es mayor. El 1 trmino de la
ecuacin de Bernoulli contina siendo nulo por ser la tubera horizontal.

Las conclusiones obtenidas que relacionan seccin con velocidad y
presin, nos conduce tambin a que:

(b)
(a)
1
2
v
1

v
2

a
F
A
1
A
2

Las presiones estticas son inversamente proporcionales a las velocidades.
4 CASO: TUBERA HORIZONTAL DE SECCIN CONSTANTE.

La novedad va a consistir en que
introduciremos al lado del tubo
piezomtrico, otro tubo abierto en ambos
extremos, pero acodado en su parte
inferior donde adems su calibre se
reduce; adems se lo instala de modo que
su boca enfrente a la corriente de lquido
por la tubera. Un tubo como ste recibe
el nombre de tubo de Pitot. El lquido que
penetra dentro del Pitot, sube por dos
razones: una, la presin absoluta p, la
misma que mide el piezmetro; otra, por
efecto de la velocidad del lquido.

Entonces, el nivel del lquido dentro de un Pitot ser mayor que en el
piezmetro, y si la tubera es horizontal, el nivel del Pitot debe ser igual al nivel del
tanque. La figura ilustra lo dicho.

La presin cintica en una seccin de la caera, se calcula multiplicando la diferencia
de niveles entre el Pitot y el piezmetro, por el peso especfico del lquido.

Lo recuadrado se expresa matemticamente as:
P
cintica
= gh

Aplicaciones del Teorema de Bernoulli.

1- SALIDA DE UN LQUIDO POR UN ORIFICIO.
Nos proponemos calcular la velocidad con que
sale el lquido por el orificio, en un tanque como el que
muestra la figura. La expresin a la que llegaremos, es
conocida con el nombre de Teorema de Torricelli.

Elegiremos un punto (1) en la superficie libre del
lquido en el tanque y un punto (2) en el punto medio del
orificio de salida. Nuestro nivel de referencia ser el del
punto (2). Aplicando el Teorema de Bernoulli a los
puntos (1) y (2), ponemos:

gh
1
+ v
1
2
+ p
1
= gh
2
+ v
2
2
+ p
2

donde v
1
= 0 y h
2
= 0. Adems p
1
= p
2
= p
atm
se cancelan. Nos queda:

gh
1
= v
2
2


Simplificando y despejando v
2
: v
2
= 2 g
1

h
Pitot Piezmetro
h
1

1
2
Siendo h
1
la altura del nivel de lquido respecto del punto medio del orificio. Observar
que la frmula obtenida para la velocidad es la misma frmula de velocidad para una
cada libre. El Teorema de Torricelli se enuncia as:

La velocidad de salida de un lquido por un orificio practicado en una pared plana y
delgada, es la misma que adquirira cayendo libremente en el vaco, desde la
superficie libre de nivel hasta el centro del orificio.

2- LAS ECUACIONES DE LA HIDROSTTICA.

Las ecuaciones de la hidrosttica no son ms que un
caso particular del Teorema de Bernoulli, para cuando v = 0 en
todos los puntos. Por ejemplo, la expresin de la presin en un
punto dentro de una masa lquida, como el (2), puede ser
deducida aplicando el Teorema de Bernoulli entre los puntos
(1) (en la superficie libre) y (2) en un lugar cualquiera. Nuestro
nivel de referencia ser el del punto (2).

gh
1
+ v
1
2
+ p
1
= gh
2
+ v
2
2
+ p
2

Los trminos con v son nulos; h
2
= 0; adems p
1
= p
atm
. Nos queda:

p
2
= p
atm
+ gh

3- PROPULSIN POR ACCIN Y REACCIN.
Si se coloca un recipiente lleno de agua, con un
orificio abajo, sobre la plataforma de un carrito, como
muestra la figura, ste se pondr en movimiento como
una consecuencia del principio de accin y reaccin.
Mientras el chorro es despedido hacia la derecha (accin),
el carrito es impulsado hacia la izquierda (reaccin).

El funcionamiento de los motores reactivos y los
cohetes se basan en la reaccin del chorro de gas que se expulsa. El movimiento
reactivo no necesita de la atmsfera para su realizacin.

La fuerza que mueve al carrito es la fuerza de reaccin. Cmo se calcula dicha
fuerza? Llamaremos A al rea del orificio, a la densidad del lquido que fluye, y v a la
velocidad de salida del lquido por el orificio. De modo que la masa expulsada puede
expresarse como:
Masa = Avt
La cantidad de movimiento p trasmitida por el recipiente al lquido que sale es:
p = m : = ( p A : t) :
Por el principio de accin y reaccin el recipiente recibe del lquido que sale, (en
el transcurso de ese t) una cantidad de movimiento igual y contraria: -p. Visto del lado
del recipiente, podemos poner que:
I

= p = p
]
p
0
= -p 0 = p F

t = p
F

=
-( puvAt)
At
= pA: : |F

| = pA| :|
2

h
1

1
2


F

reaccin

-p p
Reemplazando v por la frmula del teorema de Torricelli de la velocidad de
salida de un lquido por un orificio: v = 2g, queda:
F = 2gAh
Donde gh = p
hidrosttica
. Finalmente:
F (de reaccin) = 2Ap
H
.

4- Medicin de velocidades en una tubera, con un tubo
Pitot.

Aplicando el Teorema de Bernoulli:

gh
1
+ v
1
2
+ p
1
= gh
2
+ v
2
2
+ p
2


Si la tubera es horizontal como en el
dibujo, entonces h
1
= h
2
y los primeros trminos de
ambos miembros se simplifican. Por otra parte, en
la boca del Pitot, v
1
= 0 ya que de otro modo estara entrando lquido en el tubo. Por lo
tanto, la ecuacin de Bernoulli se reduce a:
p
1
p
2
= v
2
2


El valor de p
1
p
2
queda medido por el desnivel h del mercurio, de modo que:

P
1
p
2
=
Hg
hg

Igualando los segundos miembros: v
2
2
=
Hg
hg

Finalmente, despejando v2: v
2
= _
2gpi
Hg
p

La primera raz contiene nicamente factores constantes, o sea que tiene un valor
fijo, que denominaremos K. Entonces la expresin final ser:

v
2
= K
Clculo de la constante K, para el caso que el lquido que circula fuera agua:

K =
_
2gp
Hg
p
cguc
= _
29,813.600
1.000
= 16,32
m
1/ 2
s


5- Trompa de agua
La figura muestra un dispositivo
simple que permite extraer aire otro gas
de un recinto, produciendo un vaco
moderado. El chorro de agua ingresa por
arriba y sale por abajo a un tubo
comunicado con la atmsfera. El
estrechamiento obliga al agua a que gane
velocidad, por lo que se origina una zona
de baja presin que se localiza en la cmara
de la bomba. Esta cmara est comunicada
v
2

2
1
h
Hg
Pitot Piezmetro
Aspiracin
agua
Cmara de
La bomba
A la atmsfera
p
atm

con el tubo de aspiracin, haciendo posible evacuar el aire otro gas hasta una presin
de
1
7
de la atmosfrica (unos 100 mm Hg). El aire que se extrae se captura por el chorro
de agua y se expulsa a la atmsfera.

6- Hechos paradojales.
El Teorema de Bernoulli permite explicar una serie de hechos paradojales tales
como:
a) Las pelotas de ping-pon de la figura se aproximan cuando se hace pasar un chorro de
aire entre ambas. La velocidad del aire hace que aumente la presin cintica, obligando
a que la presin absoluta disminuya generando una zona de depresin. Lo mismo puede
hacerse con la llama de dos velas, soplando con un tubo entre ambas: las llamas se
aproximan.

b) Otros casos son el pulverizador y el carburador del automvil, que funcionan basados
en este principio.

Problemas.
1- Por una tubera de 10 cm de dimetro circula un
lquido ideal en el que se introduce un tubo
piezomtrico y un tubo Pitot. La diferencia de nivel
entre los meniscos es h = 10 cm. Determinar el
caudal.
Solucin:
p
cintica
=
1
2
p :
2
= p g h
v = 2 g = 2 9,8 0,10 = 1,40
m
s

Q = Av = r
2
v = 3,14(0,05 m)
2
1,40 m/s = 0,010995 m
3
/s = 11 litros/s

2- El tubo de la figura tiene una seccin transversal de
A
A
= 36 cm
2
en las partes anchas, y de A
B
= 9 cm
2
en el
estrechamiento. Cada 5 segundos salen del tubo 27 litros
de agua. Calcular: a) las velocidades v
A
y v
B
(en m/s), b)
La diferencia de presiones p (en Pa) c) La diferencia
de alturas h (en m).
DATOS:
agua
= 1000 kg/m
3

Hg
= 13.600 kg/m
3

Solucin:
Presin
normal
Chorro de aire
Presin
normal
Zona de presin
reducida.
h
A
B
h
Caudal: Q =
27 Itos
5 s
5,4
Itos
s
= 0,0054
m
3
s

Q
A
= Q
B
Luego, v
A
A
A
= v
B
A
B
.

a) v
A
=

A
A
A
=
0,0054
36x10
-4
= 1,5
m
s

v
B
=

B
A
B
=
0,0054
9x10
-4
= 6,00
m
s


b) p
A
+
1
2
p :
A
2
= p
B
+
1
2
p :
B
2

p
A
p
B
=
1
2
p ( :
B
2
:
A
2
) =
1
2
[1.000
kg
m
3
( 6
2
1,5
2
)
m
2
s
2
= 16.875 Po

c) p
A
p
B
= gh Luego: h =
( p
A
- p
B
)
p
Hg
g
= 0,124 m

























































25
Hidrodinmica II
(LQUIDOS REALES)

La clase pasada hemos estudiado el Teorema de Bernoulli, una herramienta
valiosa en la hidrodinmica. Sin embargo no debemos olvidar que ella solamente en
vlida cuando el flujo de lquido es laminar (rgimen estacionario), y cuando se tiene un
lquido ideal, es decir incompresible y sin viscosidad. Limitaciones incluso
contradictorias ya que en la prctica no puede asegurarse el flujo laminar si no se tiene
un poco de viscosidad.

Los lquidos reales tienen siempre algo de viscosidad y esta clase estar
dedicada a estudiar la hidrodinmica de ellos. Cuando un lquido real circula por una
caera, las fuerzas de rozamiento entre el lquido y las paredes de la caera existen y
son inevitables. Estas fuerzas realizan un trabajo a expensas de la energa que posee el
sistema, y que finalmente es entregado al medio externo en forma de calor. Adems del
rozamiento contra las paredes, existen rozamientos dentro del fluido, entre lminas o
porciones contiguas de lquido en movimiento relativo; estos rozamientos internos son
una causa adicional de prdida de energa.

Debido a ello el perfil real de las presiones en el caso que venamos
ejemplificando, de un tanque con un tubo de salida horizontal, ser como el que muestra
la figura. La presin que se pierde por rozamiento, crece con la longitud de la caera, y
se la llama prdida de carga. En los oleoductos es necesario cada cierta distancia,
instalar compre- sores para contrarrestar la prdida de carga.
Viscosidad ()
Llamamos viscosidad a la friccin interna en un fluido. La figura muestra una
placa slida A, apoyada sobre un fluido; para que ella se mueva con velocidad relativa
constante respecto de otra placa slida B, se necesita aplicarle a A una fuerza F en
forma permanente, con un valor tal que contrarreste los rozamientos internos. Podemos
imaginarnos al fluido, como un conjunto de lminas lquidas horizontales, una arriba de
otra, a lo largo de todo el espesor e. Al moverse el fluido hacia la derecha, aparecen las
fricciones internas, entre las lminas contiguas en contacto. De todas estas lminas
lquidas, tanto aquella que est directamente en contacto con A, como la que lo est con
B, permanecen adheridas a las
superficies respectivas y no se
mueven con relacin a ellas.
Mientras tanto en las restantes
lminas se observa toda una
distribucin de velocidades, dando
un perfil como el que se muestra en
la figura. As, una porcin de lquido
que inicialmente tuviera forma de
cubo, como la C, terminara
adquiriendo una forma de rombo, como la D.

De qu depende el valor de la fuerza F? Depende:
a) del espesor de fluido e, con quien resulta inversamente proporcional, para una v dada.
b) del rea A de la placa A con quien es directamente proporcional.
c) de la velocidad v con que se mueve la placa A, con quien es directamente
proporcional.

Luego, F = A

c

Donde (eta) es la constante de proporcionalidad; se llama coeficiente de viscosidad y
su valor depende de dos cosas: del fluido y de la temperatura. Para encontrar las
unidades en que se expresa , se la despeja:
=
Pc
A

y se reemplaza. As resulta, en el S.I. que las unidades son:
Ns
m
2

En la prctica es habitual trabajar con submltiplos de esta unidad: el poise que es la
dcima parte del
Ns
m
2
y el centipoise, que es la milsima parte del
Ns
m
2
.

Hay fluidos ligeros (que escurren con facilidad) como el agua y el alcohol
etlico, y hay fluidos espesos (que escurren con dificultad) como la miel o una pintura.
Prdida
de carga
SLIDO A
SLIDO B
C D e
v
A
= cte
F


h
v
Los coeficientes de viscosidad de todos los fluidos dependen notablemente de la
temperatura. En los gases, a mayor temperatura corresponde mayor viscosidad, pero en
los lquidos es al revs: a mayor temperatura corresponde menor viscosidad.

TABLA DE VISCOSIDADES (en centipoise)
AGUA GLICERINA
t (C) t(C)
0 1,78 2,8 4220
20 1,00 14 1390
100 O,28 27 490

Velocidades del fluido.
En un lquido viscoso que circula dentro
de un tubo, la rapidez es distinta en distintos
lugares de la seccin. Junto a las paredes del tubo
la capa de fluido se adhiere y su velocidad es
nula. La velocidad mxima se alcanza en el eje
del tubo. Si el flujo es laminar, el perfil de
velocidades es parablico.

La figura muestra un tubo en corte, por el
cual circula un fluido hacia la derecha y se
muestra la forma del perfil de velocidades en una seccin. Llamando con p
1
y p
2
a las
presiones sobre el lquido que circula por la caera de radio R, en dos secciones A
1
y
A
2
, separadas por una distancia L, se puede deducir una expresin que permite calcular
la velocidad v del lquido, a una distancia r del eje de la caera. La frmula es:

v(r) =
p
1
- p
2
4qL
( R
2
r
2
)
que es la ecuacin de una parbola del tipo x = ky
2
como la representada por el perfil
de velocidades de la figura.

VELOCIDAD MXIMA.
La velocidad mxima en un lquido circulando dentro de un tubo se encuentra en
el eje, donde r = 0. Si en la frmula de la velocidad [v = f(r)] hacemos este reemplazo,
nos queda:
v
mx
=
p
1
- p
2
4qL
R
2

VELOCIDAD MEDIA.
La velocidad media (v
m
) puede ser tomada como un promedio de velocidades:
v
m
=

mx
+ 0
2
v
m
=
p
1
- p
2
8qL
R
2

Ley de Poiseuille.
Poiseuille dedujo la ecuacin que permite calcular el valor del caudal (Q) de
lquido en la tubera. De acuerdo con la ecuacin de la continuidad, Q = v
m
A donde v
es la que habra en todos los puntos de la seccin, como si el perfil de velocidades fuera
rectangular y A representa al rea de la seccin; si ella es circular, A = R
2
.
Reemplazando se obtiene la ecuacin de Poiseuille:
Q =
n( p
1
- p
2
)
8qL
R
4


p
1
p
2

1 2
R
r
L
v
mx

v(r)
El nmero de Reynolds.
Cuando la rapidez de un lquido en un tubo supera cierto valor (velocidad
crtica) el flujo deja de ser laminar (con perfil de velocidades parablico como se
estudi) y se convierte en turbulento, (un movimiento irregular con vrtices y
torbellinos).

Se define el nmero de Reynolds (Re) como la relacin: Re =
dp
q

donde d = dimetro del tubo; v = velocidad media del lquido; = densidad; =
viscosidad.

Independientemente del sistema de unidades que se utilice, el nmero de
Reynolds es siempre adimensional.

Experimentalmente se comprueba que mientras sea Re < 2000, el flujo es
laminar y que si Re > 3000, el flujo es turbulento. Para Re comprendidos entre 2000 y
3000 el flujo es inestable y pasa errticamente de una forma a otra.

El valor de la velocidad crtica se obtiene despejando v de la expresin del
nmero de Reynolds y tomando como valor de Re, el de 2000:

2.000 =
d
cr
p
q
siendo d = 2r y despejando v
crtica
queda:
v
CRTICA
=
1.000q
p


Problemas.
1- Una tubera por la que circula un lquido de = 800 kg/m
3
y = 1 poise, tiene un
radio de 10 cm. Cul ser el caudal al alcanzarse la velocidad crtica?
Solucin:
Reducciones al MKS: = 0,1
Ns
m
2
r = 0,1 m
-Clculo de la velocidad crtica:
v
crtica
=
1000qg
p
=
10000,19,8
8000,1
= 12,25
m
s

-Clculo del caudal:
Q = v
cr
A = 12,25[(0,1)
2
] = 0,38
m
3
s
= 380 litros/s

2- Por una tubera de 0,40 m de dimetro y 10 m de longitud circula un lquido de
viscosidad = 1 poise. Si el gasto es Q = 6,28 litros/s cul es la diferencia de presin
entre los extremos de la tubera?
Solucin:
Reducciones al MKS: r = 0,20 m l = 10 m = 0,1
Ns
m
2
Q = 0,00628
m
3
s

De Q =
n( p
1
- p
2
)
8qL
R
4
p
1
p
2
=
8q I
nR
4
= 10 Pa

Movimiento de slidos dentro de fluidos.
Cuando existe un movimiento relativo entre un slido y el fluido que lo
circunda, actan sobre el slido fuerzas, las cuales tienen por resultante a R, en la figura
1.
La fuerza R siempre puede considerarse
descompuesta en dos: Q (con la direccin del movimiento
del slido, o bien del fluido) y P (perpendicular a la
anterior). De modo que P y Q son las componentes de R.
Ellas reciben los siguientes nombres:
P = Fuerza de sustentacin o fuerza ascensional.
Q = Resistencia frontal.
En la figura 1, el cuerpo tiene una forma cualquiera, y
ntese adems que el sentido de la fuerza Q es opuesta al
del movimiento del slido.

Si el slido fuera simtrico respecto de la direccin del movimiento (eje x), slo
existira la resistencia frontal Q, y la fuerza de sustentacin P sera nula.

Para que un slido se pueda mover sin
resistencia frontal (Q = 0) se necesita que el medio sea
un fluido perfecto, es decir que carezca de viscosidad,
en tal caso el fluido deslizar libremente junto a la
superficie del slido, rodendolo por completo. Este es
el caso que se ilustra en la figura 2. En ella se muestran
las lneas de corriente de un lquido perfecto alrededor
de un cilindro de longitud infinita; se aprecia una
perfecta simetra de las lneas de corriente, tanto
respecto de un eje horizontal como de un eje vertical.

Como consecuencia de esta simetra, las presiones en puntos tales como A y B
son iguales (y mayores que en un flujo no perturbado, ya que las velocidades junto a
dichos puntos es menor). En cambio en puntos tales como C y D, las presiones son
tambin iguales, (pero menores que en el flujo no perturbado, ya que las velocidades
junto a dichos puntos es mayor). En consecuencia, la resultante de las fuerzas de presin
en la superficie del cilindro, es igual a cero. Para slidos de otras formas obtendremos
tambin el mismo resultado. Este resultado es debido exclusivamente al hecho de
tratarse de un fluido perfecto.

Consideraremos ahora el caso del movimiento de un slido dentro de un lquido
viscoso; los fenmenos ocurrirn ahora de manera completamente diferente. Ahora una
capa muy delgada del lquido se adhiere a la superficie del cuerpo y se mueve junto con
l como un todo nico, llevndose tras de s las siguientes capas. A medida que se alejan
de la superficie del cuerpo, la velocidad de las capas disminuye y, por fin, a cierta
distancia de la superficie, el lquido resulta estar prcticamente no perturbado por el
movimiento del slido. Puede decirse que el slido se encuentra rodeado por una capa
de lquido en la que hay un gradiente de velocidad; esta capa se denomina capa lmite.
En ella actan las fuerzas de rozamiento que, al fin de cuentas, resultan aplicadas al
slido y provocan el surgimiento de la resistencia frontal. Pero la cosa no termina aqu;
la presencia de la capa lmite vara radicalmente las corrientes de lquido alrededor del
slido. El rodeo completo se hace imposible. La accin de la fuerza de rozamiento sobre
la capa lmite provoca la separacin de sta de la superficie del slido, en la parte
posterior. Se crean all torbellinos o vrtices (ver figura 3). Los torbellinos son
arrastrados por el flujo y se extinguen gradualmente a consecuencia del rozamiento; as
la energa cintica de los torbellinos se convierte en calor cedido al lquido.
P
Q
R
Figura 1
C
A B
D
Figura 2
La presin en la zona de torbellinos es menor, por lo que la resultante de las
fuerzas de presin no ser nula y ser
una de las causantes de la fuerza Q
(resistencia frontal). As pues,
resumiendo, la resistencia frontal Q es
el resultado de dos causas: la
resistencia de rozamiento y la fuerza de
presin. El valor de esta ltima
depende en alto grado de la forma del
slido; por este motivo a las fuerzas de
presin, tambin se las llama
resistencia de la forma. As fuerza o
resistencia de presin es mnima en
cuerpos con forma de gota (ver figura
4). Al fuselaje de los aviones, a sus alas, a la carrocera de los automviles, etc se
procura darles la forma de gota por este motivo.

El nmero de Reynolds (Re) es un buen indicador de la
relacin de magnitud que hay entre las resistencias de
rozamiento y las resistencias de forma. Con Re pequeos el
papel fundamental lo desempea la resistencia de rozamiento y
la otra puede ser ignorada. Al aumentar Re la importancia de la
resistencia de forma crece ms rpido que la otra. Con Re grandes predomina la
resistencia de forma.


Fuerza de sustentacin.
Para el valor de esta fuerza, la viscosidad del
lquido no tiene gran importancia. La figura 5 muestra
las lneas de corriente de un lquido perfecto rodeando
un semicilindro. Como el rodeo es completo, las lneas
de corriente sern simtricas con respecto del eje
vertical CD; pero son asimtricas respecto del eje
horizontal AB. Arriba de C la densidad de lneas es
mayor, por lo que la presin en C es menor que la
presin en D, con lo que surge la fuerza de
sustentacin.

Ahora podemos explicar por qu vuela un avin; la seccin de corte del ala de
un avin se asemeja en forma al semicilindro de la figura 5. Las lneas de corriente por
arriba del ala tienen mayor longitud que las que pasan por abajo, lo que obliga a que la
velocidad del aire sea mayor arriba del ala que abajo. Como a mayor velocidad
corresponde menor presin, surge la fuerza de sustentacin. Se cumple que:
P = Ap
Donde P es la fuerza de sustentacin; A es el rea del ala y p es la diferencia de
presiones. Por el teorema de Bernoulli, p = p
D
p
C
=
1
2
p ( :
C
2
:

2
)

El efecto Magnus.
C
A B
D
Figura 3
Mayor
presin
Fuerza de presin
Menor
presin
Figura 4
Fuerza de
sustentacin
C
A D B
Figura 5
Se conoce con el nombre de efecto Magnus, al fenmeno por el cual una pelota
en rotacin describe una trayectoria curva al desplazarse por el aire.

Para analizar este fenmeno, consideraremos las figuras 6. La 6-a muestra una
pelota que gira alrededor de un eje vertical; como consecuencia del rozamiento de la
pelota con el aire que la rodea, la pelota arrastra en su rotacin a una delgada capa de
aire. La figura 6-b muestra la misma pelota quieta (sin rotar), inmersa en una corriente
de aire que va de izquierda a derecha. Como lo que cuenta es el movimiento relativo, lo
mismo da suponer al aire en reposo y la pelota movindose con traslacin pura de
derecha a izquierda. La figura 6-c muestra el efecto resultante en el caso en que
ocurrieran simultneamente los dos movimientos anteriores, o sea que la pelota tuviera
rotacin y traslacin. La velocidad real del aire que la rodea, en cada punto del espacio,
se obtendr componiendo las velocidades de las figuras 6-a y 6-b. As, en la parte
superior del diagrama (por arriba de la pelota) ambas velocidades tienen el mismo
sentido, se suman y resulta una regin de alta velocidad y de acuerdo con Bernoulli, una
regin de baja presin. En la parte inferior del diagrama (por debajo de la pelota) las
velocidades de 6-a y 6-b son opuestas; se restan y resulta una regin de baja velocidad,
pero de alta presin. Finalmente, las diferencias de presin por arriba y debajo de la
pelota que rota, obligan a que sta siga la trayectoria curva mostrada en la figura 6-d.

Baja presin
Alta presin
(a) (b) (c) (d)
Figura 6

Você também pode gostar